MBE KAPLAN--EVIDENCE Flashcards Preview

Texas Bar Exam > MBE KAPLAN--EVIDENCE > Flashcards

Flashcards in MBE KAPLAN--EVIDENCE Deck (175)
Loading flashcards...
1
Q
  1. A customer at a fish market was leaving the store after purchasing an assortment of shrimp, oysters, and scallops. He was walking along the sidewalk in front of the store when he slipped on a piece of eel. He brought suit against the owner of the market claiming that he suffered leg and back injuries. The owner, although admitting that the customer was injured by slipping on the eel, denied negligence and claimed that the customer was contributorily negligent.
    At trial, the owner calls a witness to testify that before the fall he heard someone call out to the customer, “Watch it, buddy, you’re going to step on that piece of fish.”
    The witness’s testimony is
    (A) admissible, because it is relevant as to the customer’s contributory negligence.
    (B) admissible, as a statement of the declarant’s present state of mind.
    (C) inadmissible, as hearsay not within any exception.
    (D) inadmissible, because it was not a spontaneous utterance.
A

i. (A) Proof of utterances and writings may be made with an almost infinite variety of other purposes, not resting for their value upon the veracity of the out-of-court declarant and, hence, falling outside the hearsay classification. Here, choice (A) is correct because the witness’s testimony rests on the non-hearsay ground of evincing knowledge to show circumstantially that the customer had notice or awareness that he was about to step on the fish. Thus, the customer’s knowledge is sought to be used as the basis for a further inference that he was contributorily negligent in failing to heed the warning. Choice (B) is incorrect because the “state of mind” hearsay exception covers only statements that reveal the present mental attitude or physical condition of the declarant. Here, the statement of the declarant reveals nothing about the declarant’s own mental state or physical condition. Choice (C) is not the best answer, because choice (A) identifies a non-hearsay use for the statement. Choice (D) is a trap. The words “spontaneous utterance” tempt one to choose the excited utterance exception of FRE 803(2). The excited utterance exception requires that the declarant speak while under the stress or excitement of a startling event. There is no information in the fact pattern about the declarant’s state of mind.

2
Q
  1. While shopping at a grocery store, a customer tripped over a broken tile, fell, and suffered a concussion. A few months after the accident, the customer’s attorney deposed a store employee. In the deposition, the employee testified, “I’d been telling the store manager for years to get that broken tile fixed, but he wouldn’t do it.” The employee died in an automobile accident after being deposed.
    At trial, the deposition should be
    (A) admitted, as a dying declaration.
    (B) admitted, as former testimony.
    (C) not admitted, because it is hearsay not within any exception.
    (D) not admitted, because the employee is not available for cross-examination.
A
  1. (B) Under FRE 804(b) (1), testimony given as a witness at another hearing of the same or different proceeding, or in a deposition taken in compliance with law in the course of the same or another proceeding is admissible as a former testimony hearsay exception. McCormick points out that the courts and textwriters generally favor the wider admission of former testimony because of its elements of special reliability, such as the oath, the solemnity of the occasion, and in the case of transcribed testimony, the accuracy of reproduction of the words spoken. Choice (A) is incorrect because the dying declaration hearsay exception of FRE 804(b)(2) requires that the hearsay statement be (1) made while the declarant’s death was imminent; and (2) concern the cause or circumstances of what the declarant believed to be impending death. Here, the statement has nothing to do with the declarant’s death. Choice (C) is incorrect. The answer meets the definition of hearsay as an out-of-court statement offered for the truth of the matter asserted, and it fits the admissibility criteria of FRE 804(b)(1). Choice (D) seems like an attractive answer, but it is deceptive:
    if testimony meets the requirements of FRE 804(b)(1), it is not necessary for the declarant to be present at triaL for cross-examination. The opportunity for cross- examination would have occurred at the deposition.
3
Q
  1. A plaintiff was hit and injured by a falling branch on a sidewalk in front of a convenience store. The plaintiff calls a woman to testify. The woman proposes to testify that a witness who is now deceased told the woman that when he was walking past the convenience store one week before the plaintiff was hit by the branch, the witness saw an employee of the convenience store shoveling snow from the sidewalk in front of the store.
    The woman’s testimony is
    (A) admissible, because the witness is unavailable to testify.
    (B) admissible, as relevant for the limited purpose of proving that the convenience store owner retained control over the sidewalk where the accident occurred.
    (C) inadmissible, because it is hearsay not within any recognized exception.
    (D) inadmissible, because of the Dead Man’s Statute.
A
  1. (C) According to FRE 801(c) hearsay is defined as a “statement, other than one made by the declarant while testifying at the trial or hearing, offered in evidence to prove the truth of the matter asserted.” The woman is attempting to testify to what the witness alLegedly told her. Choice (C) is correct because the witness’s statement is (1) an out-of-court statement that is (2) offered for the truth of the matter asserted. The statement does not, however, fit within any recognized hearsay exceptions. This is an extremely popular Multistate Evidence example because the test maker knows many students will incorrectly choose choice (B). This choice would be correct if the woman were testifying that she saw an employee of the convenience store shoveling the snow. Rather, the woman is proposing to testify to what the witness alLegedly told her he saw. Since it is being offered to prove the truth (i.e., the convenience store owner retained control over the area in front of his store) of the matter asserted, it is inadmissible hearsay. Choice (A) is incorrect because, regardLess of the fact that the witness is unavailable to testify, his out-of-court statement fails to satisfy any recognized hearsay exceptions. Although interesting, choice (D) is wrong because Dead Man’s Statutes apply only to a limited circumstance: they prohibit claimants against estates from testifying about conversations or transactions with the decedent.
4
Q
  1. A defendant is on trial for attempting to cash a forged check at a bank. A teller at the bank is called to testify.
    The teller testified that she was on duty when the defendant came to her station and handed her a check drawn on the account of another customer. She recognized the forgery because she knew the other customer’s signature since he was one of her regulars. The teller further testified that after becoming suspicious, she placed the check down on the counter and turned to talk to her supervisor. As she did so, the defendant picked up the check and left the bank before she could say anything. The check that the defendant presented at the bank was not recovered by law enforcement authorities and was not offered as evidence.
    The teller’s testimony regarding the forged signature on the check is
    (A) inadmissible, because the witness is at fault in allowing the loss of the original by failing to secure the check.
    (B) inadmissible, because it is not possible for the jury to compare the signature on the check since it was not presented as evidence.
    (C) admissible, because it is rationally related to the witness’s perception.
    (D) admissible, because the witness was familiar with the alleged victim’s handwriting.
A
  1. (D) Under FRE 901 (b)(2), authentication or identification as a condition precedent to admissibility is satisfied by “Non-expert opinion as to the genuineness of handwriting, based upon familiarity not acquired for purposes of the litigation.” The teller’s testimony recognizing the forgery will be admissible because she was familiar with the other customer’s signature, since he was one of her regulars. Choice (D) is correct. FRE 701 provides for lay opinion that is rationally based on the perception of the witness. Included within the proper scope of lay testimony is familiarity with handwriting. Nonetheless, choice (D) is a more specific answer than choice (C). Choice (B) is incorrect because comparison of handwriting specimens by the jury is not required as a requisite for admissibility, although it is one way to authenticate a person’s handwriting. Choice (A) alludes to FRE 1004, which allows admissibility of other evidence of contents (i.e., the teller’s testimony) where the original is lost or destroyed, unless the proponent lost the original in bad faith. Choice (A) is incorrect because the teller was not acting in bad faith when the defendant took the check from her possession and ran out of the bank.
5
Q
  1. A plaintiff was the varsity basketball coach of the boys’ team at a high school. After the second game of the season, the plaintiff was suddenly dismissed as the coach for undisclosed reasons. After the dismissal was made public, the local newspaper ran an article discussing the plaintiff’s tenure as coach. The article, which was written by the sports editor, stated that “The plaintiff was a bad role model and morally unfit for the coaching job.”
    The plaintiff brought suit against the newspaper for defamation. At trial, the plaintiff called a player on the basketball team to testify that “the plaintiff was a good role model and respected by the players.” In presenting its evidence, the newspaper called a witness to testify. He proposes to testify that he saw the plaintiff give beer to players on three occasions after victories. Upon objection by the plaintiff’s attorney, witness’s testimony is
    (A) admissible, for impeachment purposes only.
    (B) admissible, as substantive evidence.
    (C) inadmissible, because specific instances of conduct is an improper method of proving character.
    (D) inadmissible to prove character, but admissible for the purpose of showing intent or common scheme.
A
  1. (B) Choices (C) and (D) are wrong because FRE 405(b) provides that “in cases in which character or a trait of character of a person is an essential element of a charge, cLaim or defense, proof may be made by specific instances of that person’s conduct.” The key question is whether the witness’s testimony is limited for impeachment or may be admitted substantively as proof of character. According to McCormick, a person’s character may be a material fact that, under the substantive law, determines rights and liabilities of the parties. For example, in an action of defamation for a publication to the effect that plaintiff’s character is bad, the publisher may raise the defense that the statement is true. Evidence, pg. 551. Similarly, the plaintiff is suing the newspaper for defamation based upon an article describing “The plaintiff as a bad role model.” The testimony by the witness should be admissible substantively because it relates to the truth of the assertion, which, in turn, would provide the newspaperwith a valid defense. Choice (B), therefore, trumps choice (A). Note, too, that choice (A) incorrectly states that the witness’s testimony is admissible only for purposes of impeachment.
6
Q
  1. A plaintiff sued a defendant for injuries suffered in an automobile accident that occurred six months ago. The plaintiff claimed that the defendant drove through a red light and hit her as she was crossing the street. The posted speed limit at the scene of the accident was 25 m.p.h. In his case-in-chief, the defendant testified that the traffic light was green and he was driving carefully at 20 m.p.h. The plaintiff’s attorney did not cross-examine the defendant. Rather, she called a police officer who testified that the defendant told him immediately after the accident that he was going 30 m.p.h.
    The police officer’s testimony is
    (A) a prior inconsistent statement and, therefore, admissible to impeach.
    (B) hearsay, but admissible as a prior inconsistent statement for any purpose.
    (C) admissible, as offered against the defendant.
    (D) inadmissible, because the accident report is the best evidence of what the defendant told the police officer.
A
  1. (C) Choice (C) is the correct answer because the defendant’s statement to the police officer (that he was going 30 m.p.h.) is admissible against him as an admission. By definition, admissions are the words or acts of a party-opponent or his predecessor or representative, offered as evidence against him. This is a popular Multistate example because the test maker is distinguishing between statements (or admissions) made to a police officer and statements contained in a police report. As a general rule, statements in a police report that are attributed to witnesses or to prove conclusions of the investigating officer are inadmissibLe hearsay. Choice (A) is technically correct because FRE 613 permits the use of extrinsic evidence of prior inconsistent statements, but choice (C) is a better choice because an admission can be used substantively and is, therefore, stronger than evidence that is used only to impeach. Choice (B) is incorrect because an admission is excluded from the hearsay rule by FRE 801(d)(2). Choice (D) is incorrect because the best evidence rule, FRE 1002, applies only if the contents of a writing, recording, or photograph are in dispute.
7
Q
  1. One year ago, while driving through an intersection, the defendant’s car collided with the plaintiff’s car. The plaintiff sued, claiming that the defendant ran a red light. In her case-in-chief, the plaintiff testified that immediately after the accident, a bystander, who was a local resident, screamed at the defendant, “You jerk.. . you drove through the red light.” In his case-in-chief, the defendant now calls a police accident investigator to testify that the plaintiff told him that she “didn’t see the color of the light when she entered the intersection because she was dialing a number on her cell phone.”
    The investigator’s testimony should be
    (A) admitted, because the plaintiff’s inconsistent statement has bearing on her credibility.
    (B) admitted, because the accident investigator gathered the information in the course of his official duties.
    (C) excluded, because the defendant failed to lay a foundation, thus not giving the plaintiff an opportunity to deny or explain her statement.
    (D) excluded, because the plaintiff’s statement to the investigator is hearsay.
A
  1. (A) In her case-in-chief, the plaintiff claimed that the defendant drove through a red light and hit her as she was crossing the street. So obviously the plaintiff must be contending that the traffic light was green (in her favor) when she proceeded to cross the intersection. As a consequence, her statement to the investigator that she “didn’t see the color of the light when she entered the intersection because she was dialing a number on her cellphone” may be receivable in two ways: (1) as an admission by a party under FRE 801(d), and (2) as an inconsistent statement to impeach the witness under FRE 613. Choice (A) is, therefore, correct. Choice (B) is a distracting answer and is incorrect. The words “official duties” might cause a test- taker to think of FRE 803(8), the public records hearsay exception. This exception, however, applies only to “matters observed” by public officials in the course of their duties and requires personal knowledge by the official of the matter observed. Here, the investigator has no personal knowledge of the light’s color at the time of the accident. Choice (C) is the wrong answer because, from the facts, it appears that the defendant has laid a sufficient foundation to introduce the plaintiff’s statement as either an admission or to impeach. The plaintiff will be afforded an opportunity to cross-examine the officer about the statement, if she so chooses. Choice (D) is incorrect because an admission is not hearsay under Rule 801(d).
8
Q
  1. A football player sued a football team for breach of contract. He was a star player for the team before suffering a serious hip injury. Although the player attempted to continue playing for the team, he failed a physical examination and was cut from the team roster. After failing his physical, the team discontinued making contract payments to the player as per the terms of his employment contract. In his lawsuit, the player claimed that he was physically fit to continue playing but that he was cut from the team because they wanted to stop making contract payments.
    At trial, the team called its physician, an orthopedist, to testify that the player’s hip injury was, in fact, career-threatening and, in his opinion, sufficiently serious to warrant failing his physical. The team physician stated during his testimony that his diagnosis was based, in part, upon an examination of the player’s x-rays and review of an MRI test conducted by another doctor. Neither the x-rays nor the MRI test results have been admitted into evidence.
    In all likelihood, the team physician’s testimony should be
    (A) admissible, provided that the other doctor testifies as to the validity of the MRI test results.
    (B) admissible, provided the team physician based his opinion on data reasonably relied upon by other orthopedic physicians in the field.
    (C) inadmissible, because neither the x-rays nor the MRI test results had been admitted into evidence at the time he formulated his opinion.
    (D) inadmissible, because the team physician’s opinion was not based in part upon his own firsthand observation of the player’s physical condition.
A
  1. (B) FRE 703 deals with the bases of opinion testimony by experts. An expert may base her opinion on facts or data perceived by or made known to her at or before the hearing. These facts or data need not themselves be admissible in evidence if they are of a type reasonably relied upon by experts in the particular field in forming opinions or inferences upon the subject. Therefore, an expert opinion need not be based on firsthand observation—this is why choice (D) is incorrect—and instead may be based on presentation of data to the expert outside of court. The justification for this rule is based on an attempt to bring judicial practice in line with the out-of-court practice of experts themselves, who base their own life-and-death diagnoses in reliance on information from numerous sources, including statements of patients and relatives, reports and opinions of other doctors and technicians, hospital records, and x-rays. FRE 703, Advisory Committee’s notes. As Long as the team physician’s diagnosis was based on data reasonably relied upon by other orthopedic physicians in the field, it will be admissible. Choice (B) is, therefore, correct. The x-rays and the MRI test results need not be admitted in evidence at trial, so choice (C) is incorrect. Choice (A) is incorrect because the MRI test results need not be validated, provided that such data are reasonably relied upon by experts in the field.
9
Q
  1. In an action for malpractice, a plaintiff sued a doctor, claiming that the latter failed to properly diagnose his emphysema condition. The plaintiff contends that he went to the doctor for a medical examination after experiencing chest pains and shortness of breath. However, the doctor found no evidence of the disease and advised him that he was probably suffering from indigestion. Thereafter, the plaintiff was examined by another doctor who discovered that the plaintiff in fact was suffering from a chronic case of emphysema.
    At trial, the plaintiff calls the other doctor as his first witness. The other doctor proposes to testify the x-rays of the plaintiff’s lungs showed that his respiratory organs were marked by distension, which was characteristic of emphysema. Upon objection by the defendant’s attorney, the testimony should be
    (A) admitted, because a doctor is properly qualified as an expert in medical matters.
    (B) admitted, because the other doctor followed accepted medical practice in arriving at his opinion.
    (C) excluded, because his opinion is based upon facts not in evidence.
    (D) excluded, because the x-rays are the best evidence to show the distension of the plaintiff’s lungs.
A
  1. (D) Here, it is necessary to distinguish that the other doctor is proposing to testify “that the x-rays showed a distension.” X-rays come within the best evidence rule, FRE 1002, which applies to the contents of writings, recordings, and photographs; according to FRE 1001 (2), x-rays are considered photographs. Choice (D) is, therefore, correct. Choice (A) is incorrect because there is no evidence in the fact pattern that the other doctor was tendered as an expert under FRE 702; doctors are not automatically expert witnesses at trial. There is not enough information in the fact pattern to determine whether the other doctor followed acceptable medical practices, so choice (B) is a wrong answer. Choice (C) is incorrect because, assuming that the other doctor was qualified as an expert witness, FRE 703 would permit him to base his conclusions on facts not in evidence.
10
Q
  1. A driver was prosecuted for speeding. It is conceded that the speed limit on the stretch of the highway in question at the time of the alleged speeding was 55 m.p.h. As part of its case-in-chief, the prosecution called a police officer who testified that at about 11:30 a.m. one mile east of the town line, he set up a radar apparatus. The officer testified that, with the use of tuning forks, he found the radar apparatus was accurate. The officer also stated that he had five years of experience in operating such radar devices. Furthermore, the officer testified that the radar’s mechanism for recording the speed of vehicles by means of an ink line drawn mechanically on a roll of paper was also functioning properly. Upon further questioning, the officer stated that at about 1:30 p.m., the needle of the speed indicator dial of the radar apparatus showed the driver’s car passing his location at 65 m.p.h.
    If the driver’s attorney makes an objection to the introduction of the radar finding that the driver was traveling at a speed of 65 m.p.h., the court will most likely
    (A) sustain the objection, because the radar results are not conclusive evidence of speeding.
    (B) sustain the objection, because radar is not a technique recognized by the scientific community.
    (C) overrule the objection, because the court will take judicial notice of the accuracy of such tests.
    (D) overrule the objection, because the radar results would be construed as an admission against interest.
A
  1. (C) The majority of courts are generally in accord in holding that expert testimony is no longer needed to establish the scientific validity of radar. In People v. MacLaird, 264 Cal. App. 2d 972 (1968), the trial court required the jury to take judicial notice of the validity and accuracy of radar devices. Similar results have been reached in People v. Magri, 3 NY. 2d (1958); United States v. Preos, 156 F. Supp. 200 (D.C. Md. 1957); and State v. Dantonio, 115 A.2d 35 (1955). These are among a few of the earliest cases in which the court has taken judicial notice of the validity of radar devices. Choice (A) is incorrect, given the general acceptance of radar results by courts and the scientific community. Choice (B) is an excellent example of an answer choice that cannot be correct if any other answer choice is correct. In order for choice (B) to be correct, it would have to be true that the scientific community had rejected radar. Since case law (and, for that matter, common sense) tells us that expert testimony is no longer needed to establish radar’s validity, choice (B) cannot be correct. Finally, choice (D) is incorrect. An admission is a statement made by a party-opponent. Observations of a party’s actions made or recorded by other people or by scientific instruments are not admissions.
11
Q
  1. In January, while he was driving on the street, a defendant lost control of his car, hitting and killing a small child. The child’s parents sued the defendant for causing the wrongful death of their child. At trial, the parents’ attorney calls a bystander to testify that the defendant’s car was traveling well over the 25 m.p.h. speed limit on that street.
    Upon objection by the defendant’s attorney, the trial judge would rule the bystander’s testimony
    (A) admissible, because the bystander’s opinion is based on his own perception.
    (B) admissible, as a declaration of existing state of mind.
    (C) inadmissible, as hearsay not within any of the exceptions.
    (D) inadmissible, because a lay witness is not qualified to testify about the speed of a vehicle.
A

ii. (A) Choice A is correct, because there are particular situations, such as estimating in miles per hour the speed of a moving vehicle, in which opinions of lay witnesses are generally admissible. Other such situations under FRE 701 in which lay opinions are admissible include (a) the general appearance or condition of a person; (b) the state of emotion of a person; (c) matters involving sense recognition; and (d) identifying the likeness or identity of handwriting. Choice (B) is inapplicable to this set of facts. An estimation of speed is not a “declaration of then-existing mental, emotional, or physical condition” within the meaning of FRE 803(2), the “state of mind” hearsay exception. Choice (C) is incorrect. The witness is testifying about his personal observation. No out-of-court statements are being introduced. Finally, choice (D) is the wrong answer because, under FRE 701, lay opinion witnesses are qualified to testify about such things as the speed of a moving vehicle.

12
Q
  1. A city entered into a small-cost construction contract with a construction company whereby the latter obligated themselves to perform certain minor sewer repair work. Two months after completion of the work, the company sued the city for its failure to pay under the terms of the contract.
    At trial, the company’s construction supervisor who personally oversaw all the repair work was called to testify. He sought to testify about the amount of sewer pipe used, the number of hours required to complete the job, and the number of workers employed by the company during the construction repair work.
    The defendant’s attorney objected on the grounds that the company routinely recorded such factual information in their log books, which were in the company’s possession.
    As a consequence, the supervisor’s testimony should be ruled
    (A) admissible, because it is based on firsthand knowledge.
    (B) admissible, because it is a report of regularly conducted business activities.
    (C) inadmissible, because such testimony would be violative of the best evidence rule.
    (D) inadmissible, because no evidence was introduced to show that the original log records are not available for examination.
A
  1. (A) Choice (C) is incorrect because the original document rule (otherwise known as the best evidence rule) applies to those situations where in proving the terms of a writing, and the terms are material, then the original writing must be produced unLess it is shown to be unavailable for some reason other than the serious fault of the proponent. First, there are certain writings that the substantive law (e.g., the Statute of Frauds, the parol evidence rule) endow with a degree of either indispensability or primacy. Transactions to which substantive rules of this character apply tend naturally to be viewed as written transactions and writings embodying such transactions (e.g., deeds, contracts, judgments, etc.). Contrasted with the above-described types of writings are those, essentially unlimited in variety, that the substantive law does not regard as essential or primary repositories of the facts recorded. In this regard, log books would be viewed as a writing of this latter sort in that they happen to record the facts of essentially non-written transactions. Testimony descriptive of non-written transactions is not generally considered to be within the scope of the present rule and may be given without producing or explaining the absence of a writing recording the facts. [See Allen v. W. H. 0. Alfalfa Mill Co. 272 F.2d 98 (10th Cir. 1959) where costs of production were allowed without records]. As a result, choice (D) is also incorrect because the original document rule is inapplicable. Therefore, in accordance with FRE 602, “A witness may not testify to a matter unless evidence is introduced sufficient to support a finding that he has personal knowledge of the matter. .. ,“ choice (A) is the best answer. Since the construction supervisor personally oversaw the entire job, he would clearly have firsthand knowledge regarding the construction work. Choice (B) is inapplicable because the witness is testifying directly as to his firsthand knowledge and is not introducing information from the company’s records under FRE 803(6), the business records exception.
13
Q
  1. A plaintiff sued a defendant for leg injuries stemming from a collision between their cars at an intersection. Each party was driving his own car, and the plaintiff alleged that the defendant had sped through a red light and collided with the plaintiff’s car. The defendant denied this allegation.
    During the trial, the plaintiff testified that he and the defendant had conferred after the accident and that the defendant had said, “Don’t worry about anything; I’ll see to it that your doctor’s bills are paid.” The defendant’s attorney immediately objected.
    The trial judge should rule this testimony
    (A) admissible, because the defendant admitted his negligence.
    (B) admissible, because it is a declaration against interest.
    (C) inadmissible, because the prejudice in admitting the testimony outweighs its probative value.
    (D) inadmissible, because an offer to pay medical bills is not admissible to prove negligence.
A
  1. (D) FRE 409 provides that evidence of furnishing or offering or promising to pay medical, hospital, or similar expenses occasioned by an injury is not admissible to prove liability for the injury. This rule does not extend to conduct or statements not part of the act of furnishing or offering to pay medical and similar expenses. Thus, if the defendant had said, “Since I went through the red light, I’ll pay for any injuries you may have suffered,” the defendant’s admission that he went through the red light would be admissible, although the statement regarding medical expenses would be inadmissible in accord with FRE 409. Choice (A), therefore, is an incorrect answer. Choice (B) does not apply to this fact pattern because there is no indication that the defendant is unavailable at trial, a foundational element of FRE 804(b)(3), the statement against interest exception to the hearsay rule. Choice (C) is incorrect because it misstates the balancing test of FRE 403, which states that “evidence may be excluded if its probative value is substantially outweighed by the risk of unfair prejudice…
14
Q
  1. A pedestrian is suing a defendant for injuries she suffered to her leg when the defendant ran the pedestrian over with his motorcycle. On cross- examination of the pedestrian’s physician, the defendant’s attorney asked him whether or not the pedestrian had told him that the leg that she claimed was injured had been permanently stiff as a result of a former field hockey injury the pedestrian suffered in high school.
    Upon objection by the pedestrian’s attorney, the trial judge should
    (A) sustain the objection, as hearsay not within any recognized exception.
    (B) sustain the objection, as violative of the physician-patient privilege.
    (C) overrule the objection, as a statement made for the purpose of diagnosis or treatment.
    (D) overrule the objection, as a statement against interest.
A
  1. (C) Statements made for purposes of medicaL diagnosis or treatment and describing medical history, or past or present symptoms, pain, or sensations, or the inception or general character of the cause or external source thereof insofar as reasonably pertinent to diagnosis or treatment, are not excluded by the hearsay rule under FRE 803(4). Choice (A) is the wrong answer. The statement is hearsay, but it falls under a recognized hearsay exception. Choice (B) is incorrect. There is no standard physician-patient privilege under the FRE. Instead, under FRE 501, state law provides the basis for the privilege in federaL courts. In this fact pattern, there is no information gWer about the parameters of or exceptions to the state privilege. Although the pedestrian’s statement to her doctor might well be against her interest, the fact pattern gives no information about the physician’s unavailability, which would be required in order to trigger FRE 804(b) (3), the statement against interest exception. Thus, choice (D) is incorrect.
15
Q
  1. A newly elected mayor was charged with tax fraud. The prosecutor served a trial subpoena directing the mayor’s attorney to produce private bank records that the mayor delivered to the attorney. The attorney refused the request. The prosecutor in a hearing then sought to compel compliance with the subpoena.
    The court should
    (A) compel production, because it would not violate the attorney-client privilege.
    (B) compel production, because there is no element of confidentiality.
    (C) not compel production, because it would violate the mayor’s privilege against self- incrimination.
    (D) not compel production, because it would violate the attorney-client privilege.
A
  1. (A) This question deals with the issue of what type of communications are within either the attorney-client priviLege or the Fifth Amendment privilege against selfincrimination. Pre-existing written documents (contracts, leases, memos) do not become privileged by simply being handed over to the attorney. Written documents are comm unications protected by the attorney-client privilege if they were prepared specifically from the client to the attorney or vice versa. In light of this discussion, choice (A) is correct, and the court should compel production of the mayor’s private bank records. Choice (B) confuses the issue and is incorrect. Certainly, the mayor intended the bank records to be confidential; that’s why he gave them to his attorney. However, no privilege is available to him under these circumstances. Choice (C) is incorrect. Similarly, pre-existing records, especially those of a business nature, which were voluntarily prepared by or for the owner at an earlier time, may generally be obtained by the government without violating the privilege against self-incrimination. Lilly, Law of Evidence, pp. 339—340. Whereas ordinary business records may be seized pursuant to a valid search warrant because such a seizure does not involve any incriminating assertion by the owner of the records [Andresen v. Maryland, 427 U.S. 463 (1976)], even production of specified business records is generally allowed on the theory that the gathering and submitting procedure is not, standing alone, an incriminating assertion [Fisher v. United States, 425 U.S. 391 (1976)]. Private items, such as a diary, present a more difficult question. However, “the fact that a pre-existing, non-coerced communication is private in nature is probably not dispositive so long as it can be obtained and presented at trial without compelling the individual resisting disclosure to make incriminating statements or verify the truth of the contents.” Lilly, Law of Evidence, pp. 340—341. Choice (D) is the wrong answer, for the same reasons that choice (A) is the correct answer.
16
Q
  1. A guitarist and lead singer for a rock and roll band was performing a concert when an overhead strobe light fell on stage and struck him. The singer suffered a fractured skull and was hospitalized for an extended period of time. A lighting company was hired by the venue to perform the strobe lighting show at the concert.
    During his hospital stay, the singer sent a letter to the lighting company’s president threatening to sue and holding the lighting company responsible for the accident. After receiving the singer’s letter, the company’s attorney visited the singer at the hospital where he was being treated. The attorney entered the singer’s hospital room and told him, “The company will pay your medical expenses if you will give a release.” The singer remained silent, and the attorney then left the room.
    Thereafter, the singer filed a lawsuit against the lighting company to recover damages for his injury. At trial, the singer seeks to introduce into evidence the attorney’s statement at the hospital.
    Upon objection, the attorney’s statement should be
    a. admitted, as a vicarious admission.
    b. admitted, as a declaration against interest.
    c. excluded, as an offer to compromise.
    d. excluded, as a privileged attorney-client communication.
A

i6. (C) One of the most important Evidence distinctions tested on the bar exam is statements made in compromise or settlement discussions (FRE 408) and admissions made in connection with an offer to pay medical or hospital expenses (FRE 409). Statements made in compromise negotiations are not admissible. On the other hand, under FRE 409, we sever and admit the admission while the offer to pay medical expenses is not admissible. Based on the given answer choices, it is necessary to assume that the attorney’s statement was made in connection with settlement negotiations. Since the facts state that the singer has threatened to sue the lighting company, there is an actual dispute between the parties. Under the circumstances, the lighting company’s lawyer is attempting to engage in compromise negotiations. Thus, choice (C) is correct, and choice (A) is incorrect. Choice (B) is incorrect, fortwo reasons: (1) an offer to compromise is not necessarily a statement against interest (here, there is no admission of liability); and (2) the facts contain no evidence about the declarant’s unavailability, a critical foundational element for FRE 804(b) (3). Choice (D) does not apply here because the attorney was authorized by his client to make this communication to a third party.

17
Q
  1. An experienced equestrian rider sued a woman, claiming that her horse kicked and injured the rider. At trial, the rider testified that he was attending a horse show when, without provocation, he was kicked by a gray stallion with a black tail. The rider then called the woman as an adverse witness and asked her the following question: “Isn’t it true that you are the owner and trainer of a gray stallion with a black tail?” After answering affirmatively, the woman was not asked any further questions on direct examination. Whereupon in cross- examination, the woman was then asked, “Isn’t it true that your horse is very gentle and would not kick anyone unless provoked?”
    Upon objection by the rider’s attorney, the rider’s strongest argument would be that the woman’s proffered testimony is
    (A) inadmissible as an improper form of impeachment.
    (B) inadmissible as substantive evidence to show that the horse must have been provoked.
    (C) inadmissible, because the horse’s general propensity cannot be used to prove what occurred on a particular occasion.
    (D) inadmissible as beyond the scope of the woman’s direct testimony.
A
  1. (D) FRE 611(b) provides that “cross-examination should be limited to the subject matter of the direct examination and matters affecting the credibility of the witnesses. The court may, in the exercise of discretion, permit inquiry into additional matters as if on direct examination.” In the rider’s suit against the woman, by calling the defendant as an adverse witness, the plaintiff tactically limited the scope of direct examination only to the issue of ownership of the gray stallion. As a result, the ensuing cross-examination as to the temperament of the horse goes beyond the scope of direct examination. Subject to discretion of the court, such questioning will be inadmissible. Choice (D) is therefore correct. Note: FRE 611(b) will govern procedurally, despite the fact that the defense may use other proper means (e.g., calling other witnesses or calling the defendant at a later time) to impeach the rider’s testimony. Choice (A) is inapplicable here because the question focuses more on scope of direct than impeachment. Had the rider’s attorney not limited the scope of direct, the question on cross might very well have been permissible impeachment. Choice (B) is also inapplicable. The evidence of the horse’s temperament would be admissible to show it must have been provoked, but as pointed out earlier in this paragraph, the defense would have to introduce that evidence in some other way than cross-examination. Choice (C) is wrong, because the character rules for propensity (contained in FRE 404 and 405), apply to human beings, not animaLs.
18
Q
  1. A truck driver was assigned to drive a huge tractor- trailer loaded with logs. The logs were being sold by a lumber company, which had loaded the tractor- trailer. After four hours of steady driving, the truck driver stopped at a lounge and consumed five bottles of beer. He left the tavern in an intoxicated condition and then drove off to make his final delivery.
    As he came to the top of a steep incline in the highway and started his descent, an 11-year-old girl suddenly darted out into the road directly in front of him. The truck driver slammed on his powerful air brakes and swerved sharply to the left to avoid hitting her, but the girl was struck by the tractor- trailer and was seriously injured.
    The girl’s parents, on behalf of their daughter, assert a claim against the lumber company to recover damages for the girl’s injuries.
    At trial, which of the following is LEAST likely to be admissible?
    (A) Evidence that the truck driver’s reputation for driving is poor, if the lumber company and truck driver argue that the latter was not negligent.
    (B) Evidence that the truck driver pleaded guilty to a criminal charge of DUI arising from this incident.
    (C) Evidence that the lumber company carried liability insurance on the tractor-trailer, if an investigator for the lumber company’s insurance company testified for the lumber company that they deny ownership of the truck.
    (D) Evidence that the lumber company carried liability insurance on the tractor-trailer, if the lumber company argues that the truck driver was an independent contractor.
A
  1. (A) Where the owner of a motor vehicle is charged with liability for the acts of a person using it on the grounds of negligent entrustment, then the character of the custodian is “in issue” (to show negligence in entrusting the vehicle to an incompetent driver). This is a difficult question because the facts do not indicate that this is a negligent entrustment case. The facts simply state that the girl’s parents assert a claim against the lumber company. Consequently, choice (A) is correct because, if thisisanegligenceaction (which itwould be if the lumbercompanywere introducing evidence to show that the truck driver was not negligent on this occasion), then the defendant’s character would not be “in issue,” and character evidence would be inadmissible. Choice (B) is incorrect, because the truck driver’s DUI guilty plea would be admissible to help prove negligence. Choice (C) is incorrect. Although evidence of insurance coverage is generally inadmissible under FRE 411 to prove negligence, it is admissible to prove other things, such as proof of ownership. For similar reasons, choice (D) is incorrect. Proof of insurance coverage can be introduced to show “agency, ownership, or control.” Under these facts, the evidence could be admissible to prove agency or control.
19
Q
  1. Ajudge attended a local men’s club meeting. In introducing the judge to the club members, the club president said: “Gentlemen, I take great pleasure in presenting the judge, who we all know is the best judge money can buy in the city.”
    The judge sued the club president for slander. If the club president pleads truth as a defense, he may not properly introduce evidence to show
    (A) the judge has a reputation for dishonesty in the community.
    (B) the judge was convicted of bribery three years earlier.
    (C) the judge, though not convicted, embezzled money from a former employer before being elected to the bench.
    (D) the judge was convicted for a felony assault 12 years earlier.
A
  1. (0) Character evidence is generally not admissible in civil actions unless character is “in issue” (i.e., where it is a material element of a charge, claim, or defense). In the following tort actions, character is said to be “in issue” and, thus, character evidence is admissible: (1) defamation, (2) deceit, (3) negligent entrustment, and (4) assault/battery where the defendant claims he acted in self-defense, in which case the plaintiff’s reputation for violence or turbulence is in issue. Since this is a defamation action, the judge’s character for honesty is “in issue” especially since the club president is pleading truth as a defense. In this regard, FRE 405 provides “In all cases in which evidence of character or a trait of a person is admissible, proof may be made by testimony as to reputation or by testimony in the form of an opinion.” Thus, evidence that supports the statement in choice (A) may properly be offered, since the club president can prove that the judge has a reputation for dishonesty. FRE 405(2) states, “In cases in which character or a trait of character of a person is an essential element of a charge, claim, or defense, proof may also be made of specific instances of his conduct.” Therefore, evidence that supports the statement in choices (B) and (C) may properly be admitted because the club president can show specific instances of (mis)conduct on the judge’s part (namely that (a) he was convicted of bribery, and (b) he embezzled money) to prove his defamatory statements were truthful. The assertion in choice (D) is the only one that the club president may not introduce. The judge’s character for honesty is in issue, but the felony conviction for assault does not speak to his character for honesty. Witness impeachment by prior conviction is generally acceptable for certain crimes, but such evidence is generally inadmissible if 10 years or more have elapsed.
20
Q
  1. Two weeks before an election, a local newspaper published an article implying that one of the mayoral candidates was a thief. The candidate subsequently lost the election and sued the newspaper for defamation. The newspaper defended on the grounds of truth. At trial, the candidate took the stand and testified that he was not a thief. Thereafter, the newspaper called a witness to testify that three years ago the candidate committed a larceny while employed at his former job.
    Upon objection by the candidate’s attorney, the witness’s testimony should be
    (A) excluded, because bad acts may not be proved by extrinsic evidence.
    (B) excluded, because it is improper character evidence.
    (C) admitted as probative evidence of the candidate’s character for veracity.
    (D) admitted as relevant evidence of the candidate’s character as a thief.
A
  1. (D) FRE 405(b) provides, “In cases in which character or a trait of character of a person is an essential element of a charge, claim, or defense, proof may also be made of specific instances of his conduct.” Many students will incorrectly choose choice (B); however, choice (B) is wrong because although character evidence is generally not admissible in a civil action, there are certain exceptions. For example, character evidence is admissible in those cases in which a person’s character is “in issue” (e.g., defamation, negligent entrustment, and hiring). Since this is a defamation action, the candidate’s character is in issue and, therefore, all three forms of character evidence (opinion, reputation and specific acts) are admissible. On this very point, Lilly provides the following example: Assume that the defendant states that the plaintiff is “corrupt and dishonest”; in the resulting suit for defamation, the defendant bases his defense upon the truth of his statement. Plaintiff’s character for dishonesty and corruption is directly in issue, and the inferential chain stops with the establishment of these traits. Here, the distinguishing characteristic is that the candidate’s character trait for stealing, not veracity, is in issue. In other words, the newspaper wants to produce evidence to show that the candidate is a thief in order to sustain its defense of truth in this defamation action. That’s why choice (D) is a better answerthan choice (C). Choice (A) is incorrect, because FRE 405 permits proof of specific instances of a person’s conduct in cases in which character or a trait of character of a person is an essential element of a charge, claim, or defense, as it is in this case.
21
Q
  1. A man and a school entered into a written contract providing that the man would be employed for one year in various capacities (athletic director, teacher, football coach) in return for an annual salary of $37,750. Shortly thereafter and prior to the July commencement date of employment, the school elected to terminate its varsity football program. The school publicly attributed the decision to economic factors. Upon hearing this news, the man sought out the headmaster of the school to inquire as to the effect, if any, of this decision on his salary and responsibilities.
    In order to resolve the matter, the headmaster arranged a meeting among various school officials to discuss the man’s situation. Afterward, the school sent written notice to the man of its intent to terminate his employment contract. The man then commenced suit to recover the full balance of his salary calculated on the basis of the contract. At trial, the headmaster was called to testify. On direct examination, he testified that six people (including himself) attended the meeting where it was decided to terminate the man’s employment contract. When asked to identify the attendees, the headmaster gave the names of four but, despite trying, was unable to remember the name of the fifth person.
    The attorney who called the headmaster to testify now seeks to show him her handwritten notes that the attorney prepared during her pretrial interview with the headmaster, in which he provided all five names.
    The trial court is likely to consider the showing of the notes taken as
    (A) a proper attempt to refresh the headmaster’s recollection.
    (B) a proper attempt to introduce recorded recollection.
    (C) an improper attempt to support the headmaster’s credibility.
    (D) an improper attempt to lead the witness.
A
  1. (A) FRE 612 clearly states that “Except as otherwise provided . . . if a witness uses a writing to refresh his memory for the purpose of testifying, either (1) while testifying, or (2) before testifying, if the court in its discretion determines it is necessary in the interest of justice, an adverse party is entitled to have the writing produced at the hearing, to inspect it, to cross-examine the witness thereon, and to introduce in evidence those portions which relate to the testimony of the witness.” Refreshing may be accomplished by handing the writing to the witness, accompanied by a request to the witness to read the writing to himself. Thereupon counsel retrieves the writing and repeats the question. At this point, the witness testifies from his own jogged present recollection. In this question, the headmaster’s present recollection has been exhausted, since he is unable to remember the name of the fifth person who attended the meeting to terminate the man’s employment contract. Use of the handwritten notes prepared by examining counsel in her pretrial interview is a proper attempt to refresh the headmaster’s recollection. Choice (A) is correct. Choice (B) is incorrect in this case. As the Multistate Nuance Chart below demonstrates, a key foundational element for using someone’s recorded recollection under FRE 803(5) is a failed attempt to refresh the individual’s memory. There is nothing in the fact pattern to suggest that the attorney’s effort to refresh the headmaster’s recollection has failed. Choice (C) is wrong underthese facts, because there is no indication in the fact pattern of anyone attempting either to attack or buttress the headmaster’s credibility. Choice (D) is also wrong. Showing a witness a document to refresh his or her recoLlection is not “leading” within the meaning of FRE 611.
22
Q
  1. On Thursday, March 7, an owner’s home was destroyed by fire. The owner’s house was insured against such fire loss by an insurance company. According to the policy contract, the insured was required to notify the insurance company within 10 days following any loss due to fire or other catastrophe.
    The owner sued the insurance company to recover $750,000, which was the face amount that his home was insured for. The insurance company admitted that the owner’s house was insured for that amount, though the company denied liability because it claimed that it had not received notice of the fire within the 10-day period as required by the policy.
    At trial, the owner calls his bookkeeper to testify. The bookkeeper proposes to testify that on March 11, the owner handed her an envelope and told her it contained a notice form for the insurance company, and that she mailed the envelope on his behalf.
    If the bookkeeper intends to establish that the envelope that the owner handed her contained the notice of fire loss, her proposed testimony is
    (A) admissible, because the owner’s statement to her is non-hearsay.
    (B) admissible, because a letter properly mailed is presumed received in due course.
    (C) inadmissible, because the owner’s statement to the bookkeeper is not the best evidence.
    (D) inadmissible, because the owner’s statement to the bookkeeper is hearsay not within any exception.
A
  1. (D) Hearsay evidence is testimony in court, or written evidence, of a statement made out of court, the statement being offered as an assertion to show the truth of matters asserted therein, and thus resting for its value upon the credibility of the out-of-court asserter. Here, the owner’s out-of-court statement to the bookkeeper (that the envelope contained an insurance notice form) is being offered to prove the truth of the matter stated. As a consequence, it is inadmissible hearsay. Choice (A) is, accordingly, wrong, because it states that the evidence is not hearsay. Choice (B) is interesting, but wrong. Choice (B) implicates the “mailed letter presumption,” which is that one can presume a letter reached its destination if it was properly posted. The point here is subtle, but important:
    the bookkeeper is testifying about what the owner told her concerning the purpose of the letter, not the fact that she posted it on a particular date. Thus, choice (B) is incorrect. Choice (C) is also incorrect. The contents of the letter are not at issue, which is a basic requirement of FRE 1002, the best evidence rule.
23
Q
  1. A husband is charged with the murder of his wife. The prosecution alleges that on December 17, the husband murdered his wife by giving her a massive injection of succinyicholine while she was asleep. Succinylcholine is a drug used in small quantities by anesthesiologists as a muscle relaxant. The prosecution claims that the fatal dose given to the wife so totally relaxed her lung muscles that she suffocated. Further allegations claim that the husband’s principal motive was to collect his wife’s $500,000 life insurance policy under which he was the named beneficiary.
    The defense called the physician who signed the wife’s death certificate as its first witness. The defendant’s attorney asked her, “Is it not true that the cause of death was a heart attack?” The physician answered in the negative and stated that the cause of death was unknown. The husband’s attorney then assailed her testimony as a recent fabrication and asked her if she had not stated at the coroner’s inquest that the cause of death was a heart attack. The prosecution immediately objected to this question.
    The trial judge should rule that this question is
    (A) objectionable, because a party may not impeach its own witness.
    (B) objectionable, because the husband’s attorney did not lay a proper foundation.
    (C) unobjectionable, because a party may impeach its own witness by a prior inconsistent statement.
    (D) unobjectionable, because the physician’s testimony at a proceeding that occurred shortly after the death would be more reliable.
A
  1. (C) In accordance with FRE 607, the credibility of a witness may be attacked by any party calling him (to testify). The common law rule against impeaching one’s own witness has been abandoned. Where the impeachment is by a prior statement, it is free from hearsay dangers. For this reason, choice (C) is correct, and choice (A) is incorrect. Choice (B) is wrong under these facts, because there is no indication that the defense failed to lay a proper foundation for the physician’s testimony. The physician’s expert testimony would be admissible under FRE 702 whereby “a witness qualified as an expert by knowledge, skill, expertise, training or education may testify thereto in the form of an opinion or otherwise.” Although choice (D) is closely related to choice (C), after all, a benefit of impeaching the physician with her prior statement is the inference that a statement made closer in time to an event is more accurate than a statement made later. Choice (D) is not the best choice because it fails to specifically address the doctrine that any party may impeach a witness.
24
Q
  1. A toxicologist employed at the state crime laboratory testifies at a murder trial about the autopsy he performed on the victim. His testimony is that he detected traces of succinic acid and choline, the components of succinylcholine, in the victim’s brain tissues. Over defense objections, he testified further that, in his opinion, the victim’s cause of death was succinylcholine poisoning. On cross-examination, the toxicologist admitted that up until his findings, it had been universally accepted in the medical community that succinylcholine would be undetectable in the brain within a few minutes of its injection. He also admitted that his methods of detecting succinylcholine were experimental and that his results were not presently publishable because they were not complete enough. The defense attorney then makes a motion to strike the toxicologist’s testimony regarding the cause of death.
    The court will most likely
    (A) grant the motion, because the toxicologist’s opinion could not be substantiated.
    (B) grant the motion, because the toxicologist was not properly qualified as an expert.
    (C) deny the motion, because the toxicologist was qualified as an expert in medical matters.
    (D) deny the motion, because he was qualified to give an opinion on the ultimate issue of the case.
A
  1. (A) With regard to facts ordata upon which expert opinions are based, FRE 703 requires that the facts or data “be of a type reasonably relied upon by experts in the particular field.” In the present case, the toxicologist admitted that his methods of detecting succinylcholine were “experimental” and that his results were incomplete and not publishable. Therefore, since the toxicologist could not substantiate his theory of death by succinylcholine poisoning, it would be struck as falling outside FRE 703. Choice (A) is also correct for another reason. FRE 702 states that an expert may give an opinion in a case if”(1) the testimony is based on sufficient facts or data, (2) the testimony is the product of reliable principles and methods, and (3) the witness has applied the principles and methods reliably to the facts of the case.” Under these facts, the toxicologist’s testimony likely fails prongs (1) and (2) of the FRE 702 test. Experimental methods that have not been published and subjected to peer review are suspect under FRE 702, as well as the seminal Supreme Court case on expert testimony, Daubert v. Merrell Dow Pharmaceuticals, 509 U.S. 579 (1993). Choice (B) is incorrect, because the toxicologist meets the requirements to be qualified as an expert under FRE 702 by virtue of his knowledge, training, and experience. Choice (C) is misleading: even if an individual is qualified as an expert under FRE 702, his testimony must still satisfy the FRE 702 three-prong reliability test and FRE
    703’s requirement that the underlying facts or data be of a type reasonably relied on by experts in the field. Choice (D) is aLso a misleading answer. Even though FRE 704 permits experts to testify on the ultimate issue in a case, their testimony must still satisfy FRE 702’s reliability test.
25
Q
  1. A collector told a local coin dealer that he wished to purchase an 1804 silver dollar. The coin dealer told the collector that he would locate one of these rare coins and the cost would be approximately $40,000 plus a commission. The coin dealer successfully located and arranged for the purchase of the 1804 silver dollar. After the collector purchased the coin, a disagreement arose as to the amount of commission. The coin dealer brought suit, claiming that the agreed-upon amount was 15%. The collector contends that they agreed upon 12%.
    At trial, the coin dealer introduces photocopies of company records showing that in all his previous dealings with the collector, and all other customers as well, he charged a 15% commission when asked to locate rare coins.
    The court should find that the offer of this evidence is
    (A) admissible, under the business records exception.
    (B) admissible as past recollection recorded.
    (C) inadmissible as a violation of the best evidence rule.
    (D) inadmissible as hearsay not within any recognized exception.
A
  1. (A) Since the coin dealer’s records are made in the course of regularly conducted business activities, they can be admitted under the business records exception. Choice (B) is incorrect because this is not an example of past recollection recorded, under which an authenticated memorandum, made by a witness, may be read into evidence where the witness is unable to recall the facts recorded in the document. Choice (C) is wrong because under the FRE, photocopies are as admissible as original documents and, therefore, do not violate the best evidence rule. Choice (D) is incorrect, since the company records are within a recognized exception.
26
Q
  1. A grain broker specializes in exotic grains for the growing organic-food market. Using his network of contacts across the globe, he is able to find sources for almost any kind of grain, grown anywhere in the world, to be shipped anywhere in the world. Last year, an organic food co-op asked the grain broker to find quinoa for them. The grain broker said, “I’ll do it. It’ll cost you market rate plus the commission.” The grain broker found the quinoa and had it shipped to the co-op at a cost of $15 per bushel plus a 20% commission. The co-op paid $15 per bushel plus an 8% commission. The grain broker brought suit, claiming that the appropriate commission was 20%. The grain broker introduced business records and other documentary evidence establishing that he normally charges a commission of between 10% and 20% for grains, with a higher commission for more exotic grains. The owner of the co-op called another grain broker, a competitor of the plaintiff, to testify. The witness testified that he never charges more than an 8% commission when asked to locate exotic grains for his customers.
    The trial judge should rule this testimony
    (A) admissible as evidence of customary business practices in the community.
    (B) admissible as a proper lay opinion.
    (C) inadmissible as hearsay.
    (D) inadmissible as irrelevant.
A
  1. (A) This is a classic example of a Multistate cross-over testing pattern involving an interrelationship of both Evidence and Contracts principles. When answering a Contracts question that involves buyers and sellers, apply UCC rules when presented with a hypothetical involving a merchant. In this regard, course of dealing, usage of trade, or course of performance evidence may explain or supplement language in a writing, for it shows how the parties themselves interpreted their own deal. Thus, since the grain broker was a merchant, UCC Section 1-205(4) would permit evidence of customary business practices to aid in interpreting the agreement between the parties. Choice (B) is incorrect because the witness is giving fact testimony about business practices in the area, not rendering an opinion. Choice (C) is wrong because there are no out-of-court statements being offered for their truth. Choice (D) is incorrect. FRE 401 defines relevant evidence as having “any tendency to make the existence of any fact that is of consequence to the action more probable or less probable than it would be without the evidence.” The witness’s testimony about prevailing trade practices is highly relevant on the issue of the agreed-upon commission.
27
Q
  1. A buyer filed a lawsuit against a seller based on a written contract allegedly executed at the time of the sale of the seller’s hot dog stand to the buyer. Under the terms of the contract, the seller agreed not to open a hot dog stand for a period of two years within a distance of two miles of the old hot dog stand, now owned by the buyer. The buyer alleged in his complaint that the seller violated the contract by opening a hot dog stand one-half mile from the old location. The original contract was lost prior to trial. At trial, the buyer wants to introduce a duplicate carbon copy of the written contract. The seller objects as to the authenticity of the original written contract.
    The copy is
    (A) inadmissible, because the seller contests the authenticity of the written contract.
    (B) inadmissible, because the best evidence rule requires that the original contract always be used as evidence.
    (C) admissible, because the duplicate is a carbon copy.
    (D) admissible, because the original contract was lost.
A
  1. (A) Under FRE 1003, a duplicate is admissible to the same extent as an original unless (1) a genuine question is raised as to the authenticity of the original, or (2) in the circumstances it would be unfair to admit the duplicate in lieu of the original. Therefore, if the seller contests the authenticity of the original written contract, the duplicate introduced by the buyer would not be admitted into evidence in accordance with subsection (1) of FRE 1003. Choice (B) is an incorrect rule of law. The best evidence rule does notalways require that the original document be used as evidence. If the original document is (1) lost or destroyed, (2) not obtainable by judicial process, or (3) in the possession of an opponent who refuses to produce the original at trial, other “secondary” evidence of the writing, recording, or photograph is admissible. See FRE 1004. Choice (C) is not the best answer. Although duplicate copies are generally admissible to the same extent as originals under FRE 1003, there are exceptions to this rule. Choice (A) correctly states an exception that applies to this fact pattern, whereas choice (C) merely states the general rule. Choice (D) is misleading. The key issue is that the seller contests the authenticity of the original, an issue that brings this question squarely within FRE 1003’s rule on the use of duplicates.
28
Q
  1. One afternoon a motorist was driving her car when she struck a pedestrian who entered the intersection. The pedestrian was seriously injured in the accident and sued the motorist to recover damages for personal injuries. There were no witnesses to the accident. The motorist claims that the traffic light was green at the time she entered the intersection. The pedestrian now has amnesia and cannot remember how the accident occurred.
    At trial, the pedestrian’s attorney calls a witness to testify. The witness proposes to testify that she was in the emergency room at the hospital when the pedestrian was brought in following the accident. The witness further testifies that while the pedestrian was being treated for his injuries, the nurse asked him, “How did you get hurt?” The pedestrian replied, “The car ran the red light and struck me as I was walking through the intersection.”
    Upon objection, this evidence is
    (A) admissible as a statement made for purposes of medical treatment and diagnosis.
    (B) admissible as a present sense impression.
    (C) inadmissible, because the witness was eavesdropping on a confidential communication between the pedestrian and a hospital representative.
    (D) inadmissible, because it is hearsay not within any recognized exception.
A
  1. (D) Hearsay is defined under FRE 801(c) as a “statement, other than one made by the declarant while testifying at the trial or hearing, offered in evidence to prove the truth of the matter asserted.” The witness’s testimony as to the pedestrian’s out-ofcourt reply to the nurse’s question is hearsay, since it is being offered for its truth; namely, to prove that the motorist’s car ran a red light. By process of elimination, choice (D) is correct. The pedestrian’s reply is inadmissible as hearsay, since no exceptions apply. Choice (A) is incorrect because, under FRE 803(4), a statement made for purposes of medical diagnosis or treatment must describe “medical history, or past or present symptoms, pain, or sensations, or the inception or general character of the cause or external source thereof insofar as reasonably pertinent to diagnosis or treatment.” However, statements pointing to persons responsible for the condition are considered irrelevant to medical diagnosis or treatment and do not fall within the exception. Federal Rules of Evidence Manual p.830. The pedestrian’s statement implicated the motorist and will, therefore, be inadmissible as a statement relating to the cause of the accident. Choice (B) is incorrect because, under FRE 803(1), a present sense impression must describe or explain an event or condition white the declarant was perceiving the event or condition or immediately thereafter. Here, the pedestrian’s statement was made in the emergency room fottowingthe accident. Choice (C) is incorrect because the presence of a non-essential third person, such as the witness, would destroy whatever confidentiality existed between the pedestrian and the nurse.
29
Q
  1. A driver has been charged with leaving the scene of an accident. The driver allegedly hit another car in a parking lot and drove away without leaving proper insurance and contact information. A bystander wrote down her license plate number and called the police. At trial, the driver proposes to testify that the day after the accident, she met with the police officer investigating the accident, and told him that she did not leave until after she had inspected both cars and determined there was no damage. The bystander is not available to testify at trial.
    The driver’s testimony is
    (A) admissible, because it is the statement of the witness herself, who is subject to cross-examination.
    (B) admissible, because it is a statement based on firsthand knowledge.
    (C) inadmissible, because the bystander is unavailable and, therefore, cannot contradict the driver.
    (D) inadmissible, because it is hearsay not within any recognized exception.
A
  1. (D) In this case, the driver is the out-of-court declarant proposing to testify as to the truth of the matter asserted that she only left the scene after determining there was no damage to either vehicle. As in the previous example, the statement itself is hearsay and will be inadmissible, since no exceptions apply. Choice (D) is, therefore, correct. Choice (A) is incorrect because the driver is proposing to testify in court as to what she said at another prior time out of court. The driver’s prior statement is, therefore, hearsay, despite the fact that she is both the witness, as well as the out-of-court declarant. By the same reasoning, choice (B) is incorrect because the driver is not actually testifying as to firsthand knowledge; rather, she is quoting herself. Choice (C) is incorrect because the rules of evidence do not predicate the admissibility of evidence on the availability of witnesses to contradict it.
30
Q
  1. While walking across the street one afternoon, a pedestrian was hit by a car and thrown down in the middle of the intersection. The driver of the vehicle did not stop. Although he never lost consciousness, the pedestrian was in excruciating pain and has been taking strong painkillers since the accident that have affected his memory of the accident. At trial, the pedestrian calls a renowned hypnotist to testify. The hypnotist proposes to testify that after he placed the pedestrian under a hypnotic spell, the pedestrian gave a detailed account of the accident and told the hypnotist that the driver ran a red light and that the driver’s license plate number was GIJO-7 14. The driver concedes that his license plate number is
    GIJO-7 14.
    The hypnotist’s proposed testimony will most likely be held to be
    (A) admissible, because the license plate identification substantiates that the other information is accurate and trustworthy.
    (B) admissible, because it is a statement of past recollection refreshed.
    (C) inadmissible, because it is hearsay not within any recognized exception.
    (D) inadmissible, because statements procured through hypnosis cannot be authenticated as truthful.
A
  1. (C) The hypnotist’s testimony as to what the pedestrian told him while under the hypnotic spell, is an out-of-court assertion being offered for its truth. The statement will be inadmissible as hearsay, since no exceptions apply. Because the hypnotist’s testimony is inadmissible hearsay, the corroborating evidence of the driver’s license plate number does not matter. Thus, choice (A) is incorrect. Choice (B) is wrong because the foundational elements of past recollection refreshed have not been met. Past recollection refreshed under FRE 612 applies only when an attempt is made to refresh the recollection of a testifying witness. Here, the pedestrian is not testifying. Choice (D) asserts that hypnotically refreshed testimony that cannot be authenticated may be true, but authentication (orthe lackthereoO does not matter if a statement is inadmissible hearsay under the rules. Choice (0) is incorrect.
31
Q
  1. Under the Federal Rules of Evidence, which of the following items of evidence is NOT self- authenticating?
    (A) A pamphlet on air safety regulations that a plaintiff claims is an official publication of the Federal Aviation Administration.
    (B) A copy of a magazine that a plaintiff claims contains a libelous picture.
    (C) A holographic will without attesting signatures that a plaintiff claims was written by her mother in her own handwriting.
    (D) A certified copy of his car registration form that, according to a plaintiff, verifies his ownership of the vehicle.
A
  1. (C) Choice (C) is correct, since an unattested holographic will of the plaintiffs mother will not be self-authenticating. Under FRE 902, extrinsic evidence of authenticity as a condition precedent to admissibility is not required with respect to the following: (1) domestic public documents under seal; (2) certified genuine foreign public documents; (3) certified copies of public records—such as the certified copy of the car registration form in choice CD); (4) official publications—including books and pamphlets issued by public authority—such as the Federal Aviation Administration pamphlet on air safety regulations in choice (A); and (5) newspapers and periodicals—such as the copy of a magazine in choice (B).
32
Q
  1. An owner of a storage facility stores flammable gases under high pressure in large spherical tanks. The facility was constructed for the owner by a company that specializes in the construction of such facilities. After the facility had been in use for three months, one of the tanks ruptured, causing a spillage and loss of the chemicals. The owner sued the company for strict liability, claiming that the storage tank had been defectively manufactured.
    At trial, the owner called a photographer who testified she was hired by the owner to photograph the ruptured tank after the accident. She further testified that the roll of film was sent to a commercial film lab where the photographs were developed. Thereupon, the owner’s attorney showed the photographer the photographs, which she identified as the ones she had taken and given to the Owner.
    The owner’s attorney offers 10 photographs into evidence. Upon objection by the company’s counsel, which of the following, if true, is the best reason to exclude the photographs from evidence?
    (A) The photographer was not shown to be an expert or professional photographer.
    (B) There was an unaccountable break in the chain of custody from the time when the photographs were taken to the period when the negatives were processed.
    (C) There was no testimony that the photographs accurately portrayed the ruptured tank.
    (D) The photographs are not the best evidence to prove how the tank ruptured.
A
  1. (C) To authenticate a photograph, it is always required that the proponent establish that the pertinent parts of the picture are a reasonable and accurate representation of the subject pictured. This requirement may be satisfied if a witness (not necessarily the photographer) has observed the scene and testifies that the picture is an accurate representation. Lilly, Law of Evidence, p.423. Choice (C) is the best answer to exclude the 10 photographs. Lack of proper authentication would preclude admissibility if no testimony were offered that the photographs accurately portrayed the ruptured tank. Choice (A) is incorrect because there is no requirement under the rules that a photographer be qualified as an expert or professional photographer. Choice (B) is wrong for two reasons: (1) it is predicated on facts that are not contained in the fact pattern; and (2) even if there were a break in the chain of custody, it would still be possible to authenticate the photographs as reasonably and accurately representing the ruptured tank. Choice (D) is wrong. The best evidence rule only applies when the contents of a writing, recording, or photograph are at issue. See FRE 1002.
33
Q
  1. A woman was sailing in a bay when she was struck by a motorboat. The woman, who was seriously injured in the accident, sued the motorboat operator in a personal injury action. During the course of the lawsuit, an important question of consequence was the wind speed on the afternoon the woman was sailing.
    At trial, the woman calls an orthopedic surgeon to testi1’. The surgeon, who is an amateur boating enthusiast, testified that he was navigating his sailboat into the bay when the motorboat hit the woman. Furthermore, the surgeon testified that in the cockpit of his boat he maintains a sophisticated electronic weathering device that measures wind speed, temperature, and barometric pressure at periodic intervals. The woman then offers into evidence the computer printout from the surgeon’s weathering device measuring the wind speed at the time of the accident.
    Upon objection by the motorboat operator, the printout is
    (A) admissible as past recollection recorded.
    (B) admissible as the record of regularly conducted activity.
    (C) inadmissible as hearsay not within any recognized exception.
    (D) inadmissible, unless there is foundation testimony as to the accuracy and good working condition of the surgeon’s electronic weathering device on the afternoon in question.
A
  1. (D) In the woman’s personal injury action against the motorboat operator, her witness, the surgeon, is offering into evidence a computer printout from his sophisticated electronic weathering device measuring wind speed. Regarding this computer printout, students should begin by understanding that under FRE 1001(3), “If data are stored in a computer or similar device, any printout or other output readable by sight, is an ‘original’.” There is no best evidence problem as to the admissibiLity of the computer printout. Next, a foundation for the authenticity of the scientific wind device must be laid. Under the original Frye v. United States standard, scientific evidence is admissible where the device or principle from which the deduction is made is “sufficiently established to have gained general acceptance in the particular fieLd.” McCormick, pp. 610—614. Choice (D) is correct. Choice (A) is incorrect because the “past recollection recorded” hearsay exception requires a human witness. Choice (B) is wrong; it describes the business records exception of FRE 803(6), which is not at issue here. Choice (C) is wrong because hearsay under the rules requires either a human declarant or, in the case of computer records, input generated by a human being. According to Mueller & Kirkpatrick’s treatise on evidence, “when information provided by machines is mostly a product of mechanical measurement or manipulation of data by common scientific or mathematic techniques, hearsay concerns are usually addressed by requiring the proponent to show the machine and its functions are reliable, that it was correctly adjusted or calibrated, and that basic data put into the machine are accurate.” Christopher Mueller& Laird Kirkpatrick, Evidence § 8.11 (3d ed. 2003). Mueller& Kirkpatrick go on to explain that when the output of a machine depends on human input, regular hearsay concerns apply.
34
Q
  1. A shopper suffered injuries to her back and legs when she slipped on a loose piece of carpeting in a hallway connecting a department store and a cafeteria.
    The shopper brought suit against the owner of the entire building. The owner claimed that since he leased the cafeteria, he no longer retained any control over the connecting hallway.
    If the shopper offers into evidence the fact that two days after the accident, the owner had the hallway carpet re-laid, the court should rule that the evidence is
    (A) admissible, because it is relevant to the issue of the owner’s knowledge of the unsafe condition.
    (B) admissible, because it shows that the owner retained ownership and control.
    (C) inadmissible, because it shows subsequent repair, which is against public policy.
    (D) inadmissible, because the evidence is irrelevant to the accident.
A
  1. (B) ALthough evidence of subsequent repair is usually excluded because its basic relevancy is outweighed by public policy considerations, evidence of subsequent remedial measures is admissible for the Limited purpose of showing ownership or control. Choice (A) is incorrect. One of the policies underlying Rule 407 is to encourage owners to repair unsafe conditions. Permitting a jury to infer that repair of an unsafe condition equates to knowledge it was unsafe at the time of the accident would discourage owners from making repairs. Although choice (C) is a true statement, it is not the best answer in this case because the exception to show ownership or control applies more specifically to the fact pattern. Choice (D) is incorrect. The evidence is relevant under FRE 401.
35
Q
  1. A man and a woman are tennis partners who play regularly. Last January, the man slipped and fell while restocking grocery shelves at work. The man is suing the grocery store for damages, claiming that he suffers from pain in his back and legs as a result of the accident. At trial, the woman testified that five days prior to the accident, the man told her, “I can’t complete the tennis match because of pains in my back and legs.”
    The trial judge should rule this testimony
    (A) inadmissible as hearsay.
    (B) inadmissible as opinion testimony.
    (C) admissible as a proper lay opinion.
    (D) admissible as a statement of bodily condition.
A
  1. (D) As a general rule, a declaration of a physical condition (e.g., “I have pains in my back and legs”) is admissible as an exception to the hearsay rule under FRE 803(3) (then-existing mental, emotional, or physical condition). Choice (D) is, therefore, correct. Choice (A) is wrong. The statement is hearsay, but it is admissible under a recognized hearsay exception. Choices (B) and (C) are incorrect for similar reasons. The man’s statement is a description of his physical condition, not a statement of opinion.
36
Q
  1. A man works at a tire warehouse, where his job duties consist of loading tires onto trucks. The job is exhausting, and the warehouse is always short of personnel. One day, while loading a heavy tractor tire onto a truck by himself, he experienced a sharp and excruciating pain in his lower back. He did not report the incident to his employer, but continued to work. A week later, after work, he went to the doctor for treatment. The man then sues his employer, alleging that the employer failed to provide him with adequate assistance, safety equipment, and training to load heavy tractor tires, thereby contributing to his injury. The employer alleges that the man is merely trying to obtain compensation for an old back injury, unrelated to his employment.
    At trial, the man’s doctor testified that the man told her that his back began hurting while he was loading the tire on the truck. The doctor’s testimony is
    (A) admissible as expert testimony.
    (B) admissible as a statement for the purposes of medical diagnosis.
    (C) inadmissible as a violation of the physicianpatient privilege.
    (D) inadmissible as hearsay not within any recognized exception.
A
  1. (B) Statements made to assist a physician in diagnosing or treating a physical condition are hearsay and require the exception listed in choice (B) in order for them to be admissible. Choice (A) is wrong because the doctor is repeating the man’s statement and not giving his expert opinion. Choice (C) is incorrect because the doctor’s testimony, in such a situation, does not violate the doctor-patient privilege. Choice CD) is incorrect; although the statement is hearsay, it does meet an exception and is admissible.
37
Q
  1. A defendant was being prosecuted on a charge of manslaughter in the first degree for causing the death of a 15-year-old girl on whom he had allegedly performed an abortion in his private clinic.
    A witness for the defendant testified to the defendant’s non-culpability at the grand jury proceeding. The witness was later called as a defense witness at the trial. During his direct examination, defendant’s attorney asked him, “Is it not true that the defendant could not have performed the abortion on the decedent since he was assisting you in another abortion at the time?” The witness answers in the negative and states that the defendant did, in fact, perform the abortion on the girl.
    The defendant’s attorney then assails the witness’s testimony as a recent fabrication, and asks the witness if he had not stated to a grand jury that the defendant was not guilty of performing the abortion, thus causing the death of the girl. The prosecutor objects to this question.
    The trial judge should rule that this question is
    (A) objectionable, because the defendant’s attorney did not lay a proper foundation for impeaching his own witness.
    (B) objectionable, because a party may not impeach his own witness.
    (C) unobjectionable, because a witness’s testimony before a grand jury is more reliable because it occurred shortly after the abortion was allegedly performed.
    (D) unobjectionable, because a party can impeach his own witness by a prior inconsistent statement.
A
  1. (D) In accordance with FRE 607, the credibility of a witness may be attacked by any party, including the party calling him. The common law rule against impeaching one’s own witness has been abandoned. Where the impeaching is by a prior statement in a grand jury proceeding, it is free from hearsay dangers and is excluded from the category of hearsay under FRE 801 (d) (1)(A). For similar reasons, choice (B), which states that a party may not impeach its own witness, and choice (A), which claims that the attorney did not lay a proper foundation for impeachment, are the wrong choices. Choice (C) is incorrect. The issue in impeachment by prior inconsistent statements is not the reliability of the prior statement, but the fact that the witness has made inconsistent statements on the same matter.
38
Q
  1. While undergoing a routine plastic surgery procedure, a patient died on the operating table. The patient’s husband is suing the plastic surgeon alleging malpractice. Two years later, at trial, the defense called another doctor as an expert witness. The expert witness testified that the patient’s death was a freak accident and that the surgeon’s performance met the highest standards for medical care. On cross-examination, the plaintiff’s attorney brings out the fact that, one week before trial, the expert witness and the surgeon signed an agreement to become partners in an outpatient plastic surgery clinic. The plaintiff’s attorney suggests that the expert’s testimony was awfully convenient and likely motivated by his desire to maintain a good relationship with the defendant. The defendant then calls another surgeon at the hospital as a witness. He is asked to tell the jury about a conversation he had with the expert inunediately following the patient’s death. The witness stated that “the expert told me the patient’s death was a textbook example of a freak accident and there was nothing anyone could have done to prevent it.”
    On objection to this testimony, defendant’s attorney seeks a ruling on the admissibility of this question and answer. The trial judge should
    (A) sustain the objection, because the testimony is hearsay.
    (B) overrule the objection, because a witness’s veracity may be rehabilitated by a prior consistent statement.
    (C) sustain the objection, because a prior consistent statement cannot be used to rehabilitate a witness’s testimony.
    (D) overrule the objection, because the expert’s statement is a statement against interest.
A
  1. (B) Generally, the prior consistent statements of a witness are not admissible to support the witness’s testimony at trial. However, where the testimony of a witness is assailed as a “recent fabrication,” it may be confirmed by proof of declarations of the same tenor before the motive to falsify existed. Prior consistent statements used for this purpose are not hearsay under FRE 801(d) (1). Because prior consistent statements are not hearsay under FRE 801(d)(1), choice (A) is incorrect. Choice (C) is wrong: as discussed above, prior consistent statements can be used to rehabilitate a witness’s testimony. Choice (D) is incorrect, for two reasons: (1) the statement was not against the expert’s interest; and (2) in orderto use the statements-againstinterest hearsay exception of FRE 804(b)(3), the decLarant must be unavailable. The expert, having just testified in the trial, is available.
39
Q
  1. A defendant was charged with vehicular battery after driving through a red light and crashing into a woman’s small convertible. The woman suffered massive internal injuries and lapsed into a coma for several hours after she reached the hospital. The woman’s best friend, who was a passenger in the car, miraculously received only a few minor bruises. After the collision, the passenger stayed with the woman trying to comfort her until the ambulance arrived.
    At trial, the passenger is called to testify. ‘While on the witness stand, she gazes with sorrow at the woman, who is still wearing a cast on her leg. The passenger testifies that the first thing she remembered seeing after the accident was the woman’s bloodied body trapped in the mangled wreckage. The state’s attorney then asked the passenger if the woman said anything to her before being taken to the hospital. The passenger answered, “Yes, she told me: ‘I know I’m dying. Why did he have to run that red light?”
    Upon objection by the defendant’s attorney, the court should rule the passenger’s testimony
    (A) admissible, because it is irrelevant that this is not a homicide case.
    (B) admissible, because the woman believed her death was imminent when she made the declaration.
    (C) inadmissible, because of the close relationship between the woman and the passenger.
    (D) inadmissible, because it is hearsay not within any recognized exception.
A
  1. (A) In order for a dying declaration to be admitted, the dec(arant must be unavailabLe to testify. In this example, the dying declaration exception is inapplicable because the woman is not unavailable. In accordance with FRE 804, the dying declaration exception is not excluded by the hearsay rule if the declarant is unavailable as a witness. Since the woman is in court and available as a witness, choice (B) is wrong. Consequently, choice (A) is correct because her statement, “I know I’m dying,” is admissible under FRE 803 as a statement of then-existing physical condition. In addition, her statement, “Why did he have to run the red light?” is not excluded by the hearsay rule because, under FRE 803, it qualifies as a present sense impres. sion. Note that in regard to the hearsay exceptions classified under FRE 803, the availability of the declarant is immaterial. Choice (C) is incorrect. The relationship between a hearsay declarant and a witness is immaterial in determining the admissibility of the hearsay statement. Choice (D) is also incorrect because, although the statement is hearsay, it fits within recognized hearsay exceptions.
40
Q
  1. A victim was standing on a street corner waiting to make a drug buy. The drug dealer was always late, so the victim continued to wait outside in the freezing cold weather because he needed a quick fix. Suddenly a car rounded the street corner and a shot rang out. The victim slumped over and fell to the sidewalk. Immediately after being shot and before he died, the victim exclaimed, “The defendant did it!” The defendant was indicted and is now on trial for the victim’s murder.
    Based on these facts alone, the admissibility of the victim’s statement should be determined by
    (A) the jury, after a preliminary determination by the judge.
    (B) the jury, without a preliminary determination by the judge.
    (C) the judge, with instruction to the jury that it may consider the dying declaration if it determines that the victim knew he was dying.
    (D) the judge, without assistance from the jury.
A
  1. (D) FRE 804 (B) (2), also referred to as the “dying decLaration exception” to the hearsay rule, provides that “in a prosecution for homicide or in a civil action or proceeding, a statement made by a declarant while believing that his death was imminent, concerning the cause or circumstances of what he believed to be his impending death is not excluded by the hearsay rule if the declarant is unavailable as a witness.” Note two important points in regard to the dying declaration exception: (1) the FRE has broadened the traditional use of dying declarations in prosecutions for homicide by now allowing such statements to be utilized in civil cases; and (2) in accordance with the FRE, the declarant does not need to die in order for his statement “made under belief of impending death” to be deemed admissible. Rather, the requirement that must be met is that the declarant be unavailable to testify. Choices (A), (B), and (C) all misstate the proper roles of the judge and the jury pertaining to the admissibility of evidence. According to FRE 104(a), questions concerning the admissibility of evidence “shall be determined by the court [judge].” Choice (D) is the only answer that correctly states the role of a judge in determining the admissibility of evidence.
41
Q
  1. A defendant, on trial for robbery, took the stand and testified in his own behalf. On cross-
    examination, the prosecuting attorney asked the defendant whether he had committed false pretenses by making long-distance telephone calls by the unauthorized use of a credit card number. The defendant replied, “That’s a lie, I never did.” In rebuttal, the prosecuting attorney called the court officer to testif’ that the defendant admitted committing false pretenses. The court officer further testified, however, that the defendant was granted immunity from prosecution in exchange for his testimony against others.
    The court officer’s testimony is
    (A) admissible, as bearing on the defendant’s credibility.
    (B) admissible, as bearing on the defendant’s guilt.
    (C) inadmissible, because the testimony is extrinsic evidence.
    (D) inadmissible, because the court order is the
    best evidence.
A
  1. (C) In accordance with FRE 608 (b), specific instances of the conduct of a witness, for the purpose of attacking or supporting his credibility, otherthan conviction of crime as provided in FRE 609, may not be proved by extrinsic evidence. They may, however, in the discretion of the court, if probative of truthfulness or untruthfulness, be inquired into on cross-examination of the witness (1) concerning his character for truthfulness or untruthfulness, or (2) concerning the character for truthfulness or untruthfulness of another witness as to which character the witness being cross- examined has testified. Choice (A) is incorrect, because of the prohibition against using extrinsic evidence. Choice (B) is also incorrect. Although the evidence does have a bearing on the defendant’s guilt, the rules prohibit using it. Finally, choice (D) is wrong because the contents of the court order are not in dispute; therefore, the best evidence rule is inapplicable.
42
Q
  1. In a suit by an investor against a speculator, a relevant fact is the price of gold as quoted on a specific date 20 years ago. The investor calls a librarian to authenticate the microfilm copy of a newspaper from that date. This copy, kept in the archives of the public library, is the only record of that particular issue of the newspaper.
    The microfilm is
    (A) admissible, to prove the price of gold on that day.
    (B) admissible as an ancient document.
    (C) admissible as past recollection recorded.
    (D) inadmissible as not the best evidence.
A
  1. (A) Under FRE 1003, duplicates of a document are generally admissible to the same extent as the original. According to FRE 1001(4), a duplicate includes a photographic miniature. Thus, a microfilm of the newspaper would be considered a duplicate and would be admissible as an original under FRE 1003. Choice (B) is not a bad answer—the ancient documents exception of FRE 803(16) permits the admissibility of statements in a document more than 20 years old, the authenticity of which is established, but choice (A) is a better answer because the use of a microfilm copy of the newspaper raises issues regarding the admissibility of duplicates. Choice (C) is incorrect. The past recollection recorded exception applies when a witness on the stand is unable to remember an event but made a record of it close to the time of its occurrence. Choice (D) is wrong because it misstates the best evidence rule.
43
Q
  1. For which of the following preliminary questions of fact will the trial court judge, not the jury, determine admissibility?
    (A) Whether the voice on a sound recording is that of the defendant.
    (B) Whether defendant’s copy of a document accurately reflects the contents of the original writing.
    (C) Whether a conspiracy existed and defendant and declarant were members of the conspiracy before admitting an alleged declaration by a co-conspirator.
    (D) A plaintiff sues on a lost writing, and the defendant contends that it was not lost because it never existed. The question to be decided is whether the original writing ever existed.
A
  1. (C) Lawyers sometimes use the expression that the jury decides questions of fact, and the judge resolves questions of law. But there are numerous occasions when the judge makes factual determinations. In the pre-trial process, for example, he resolves factual disputes that may arise in connection with discovery proceedings. The judge also decides preliminary factual questions that accompany the application of the exclusionary rules of evidence. Here, choice (D) is wrong because FRE 1008 states that “when an issue is raised (a) whether the asserted writing ever existed, or (b) whether another writing, recording, or photograph produced at trial is the original, the issue is for the trier of fact to determine as in the case of other issues of fact.” Choices (A) and (B) are wrong because they also involve factual issues to be decided by the jury. Choice (C) is correct because, according to McCormick, “the judge should make the preliminary determination whether a conspiracy existed and defendant and declarant were members of it, before admitting an alleged declaration by a co-conspirator.” Evidence, pg. 139.
44
Q
  1. A defendant is on trial for robbery. The defendant’s attorney plans to call a witness. Nine years earlier, the witness had been convicted of misdemeanor battery. The defendant’s attorney has filed a motion disallowing the prosecution from questioning the witness regarding this prior conviction.
    The motion will likely be
    (A) granted, as a matter of right.
    (B) granted, because the prejudice to the defendant substantially outweighs the probative value of admitting the witness’s conviction.
    (C) denied, because the credibility of a witness may be attacked by any party.
    (D) denied, because a period of less than 10 years has elapsed since the date of conviction.
A
  1. (A) Under FRE 609 (a)(2), “For purposes of attacking the credibility of a witness, evidence that he has been convicted of a crime shall be admitted if elicited from him or established by public record during cross-examination but only if the crime (2) involved dishonesty or false statement, regardless of the punishment.” The only way a conviction of a misdemeanor can be used to impeach a witness under this rule is if the misdemeanor involves dishonesty or false statement. Since battery does not, the prosecution may not question the witness regarding this prior conviction. Choice (A) is thus correct. Choice (B) is wrong because the “balancing test” under FRE 609 applies only to convictions more than 10 years old. Choice (C) is wrong. Although any party may attack the credibility of a witness, FRE 609 specifically controls the use of convictions evidence. Choice (D) is misleading. The question is designed to attract students because everyone remembers the 10-year time limit of FRE 609. It is important to remember that misdemeanor offenses not involving dishonesty or false statements aren’t admissible under FRE 609 to impeach a witness.
45
Q
  1. A plaintiff is bringing an action against a defendant to quiet title to determine ownership of lakefront riparian property. At issue is the mean water level of the lake. For the past several years, a commercial fisherman has kept daily logs of the lake’s water level. In preparation for trial, the plaintiff hired a graphic artist to prepare charts from the fisherman’s logs to establish the property’s boundary line. Based upon the information contained in the fisherman’s logs, the graphic artist compiled a number of charts, which the plaintiff now seeks to introduce into evidence. At trial, it was determined that the fisherman is an expert on water level computations.
    The defendant objects to the charts being offered into evidence. The court will most likely rule the charts
    (A) admissible, because the fisherman is an expert on water level computations.
    (B) admissible, because they are summaries of voluminous records.
    (C) inadmissible, because the charts are hearsay not within any recognized exception.
    (D) inadmissible, because the fisherman’s daily logs are the best evidence of the lake’s water level.
A
  1. (B) According to FRE 1006, the contents of voluminous writings, recordings, or photographs that cannot conveniently by examined in court may be presented in the form of a chart, summary, or calculation.The originals, or duplicates, shall be made available for examination or copying, or both, by other parties at a reasonable time and place. In this question, the fisherman has kept daiLy Logs for severaL years on the water level of the lake. Clearly, these logs are voluminous records that may not be conveniently reviewed in court by the jury. Consequently, the charts drafted by the graphic artist may be admitted as summaries of voluminous records. Choice (A) is incorrect. There is no requirement that the compiler of records be qualified as an expert. Choice (C) is incorrect. The fisherman’s logs themselves are probably hearsay, but would be admissible under the business records exception of FRE 803(6). The reason for using a summary under FRE 1006 is that the fisherman’s records themselves could not be conveniently reviewed in court; a summary would be much better. Choice (D) misstates the best evidence rule and is wrong.
46
Q
  1. A man is on trial for securities fraud. He takes the stand to testify in his own behalf. On direct examination, the man’s attorney proffers evidence of his good character.
    Upon objection by the prosecution, which of the following statements is correct?
    (A) The defendant may introduce evidence to show truthfulness.
    (B) The defendant may introduce evidence to show honesty.
    (C) The defendant may not introduce evidence to show honesty because character evidence is inadmissible.
    (D) The defendant may not introduce evidence of truthfulness unless it is inquired into on cross- examination.
A
  1. (B) A trial for fraud is a criminally related civil proceeding. As a general rule, character evidence is not admissible in a civil case unless character is “in issue.” According to FRE 405, character is in issue if it is an essential element of a charge, claim, or defense. For the exam purposes, remember that character is generally in issue and, thus, admissible in the following civil actions: (1) defamation, (2) child custody, (3) negligent entrustment, and (4) negligent hiring. Another civil action where character is in issue is an action for fraud. In regard to a fraud or deceit action, the defendant’s character for honesty is in issue. Therefore, the defendant may introduce evidence to show his character trait for honesty. Choice (B) is, therefore, correct. Choice (A) is wrong because, in a fraud action, it is the defendant’s character trait for honesty, not truthfulness, that is in issue. Choice (C) is wrong for the same reasons that choice (B) is correct. Choice (0) is wrong because it suggests that the defendant cannot respond unless his character has been attacked on cross-examination. If the defendant testifies, he becomes a witness. His character for truthfulness may be attacked “by opinion or reputation evidence or otherwise.” FRE 608(a). Once his character for truthfulness is attacked, he can respond with reputation or opinion evidence of his positive character for truthfulness.
47
Q
  1. A gardener sued a homeowner for breach of contract. The gardener claims that the homeowner hired him to perform landscape work at his home. After performing the gardening services, the homeowner paid him $100. The gardener, who speaks only English, contends that the homeowner, who speaks only Japanese, originally agreed to pay him $200 for the work. When the parties entered into their agreement, the gardener was accompanied by his friend, who is now deceased, who spoke both English and Japanese. This jurisdiction has a typical Dead Man’s Statute in effect.
    At trial, the gardener now seeks to testify to the terms of his agreement with the homeowner as communicated to him by his friend. Upon objection by the homeowner’s attorney, the gardener’s proposed testimony is
    (A) admissible as an admission by a party-opponent.
    (B) admissible as non-hearsay to corroborate evidence of a verbal act.
    (C) inadmissible, because of the Dead Man’s Statute.
    (D) inadmissible, unless it is established that the gardener accepted the friend as the homeowner’s agent in communicating the terms of the contract.
A
  1. (D) Under FRE 801(d)(2)(D), a statement offered against a party by his agent or servant concerning a matter within the scope of his agency or employment, made during the existence of the relationship is admissible as a vicarious admission. If it is established that the friend was the homeowner’s agent in communicating the terms of the contract, then the friend’s statement may be offered as a vicarious admission against the party, the homeowner, by his opponent, the gardener. Choice (D) is correct. Choice (C) is incorrect. A Dead Man’s Statute typically provides that in any action commenced or defended on behalf of a decedent, any other party surviving shall be disabled from testifying as to any transaction involving the decedent. The practical consequence of this type of statute is that if a survivor has rendered services or furnished goods to a person he trusted, without an outside witness or admissible written evidence, he is helpless if the other dies and the representative of his estate declines to pay. McCormick, Evidence, p. 159. In these facts, the contract action was not commenced or defended on behalf of the friend, the deceased, so the Dead Man’s Statute will not apply. Choice (A) is incorrect because the gardener is seeking to testify as to the friend’s statements. The fact that the friend is not a party to the lawsuit precludes the use of his statements as an admission. Choice (B) is incorrect because the friend’s statements concerning the terms of the homeowner’s agreement would be inadmissible as hearsay, unless a basis for a vicarious admission is established.
48
Q
  1. A victim and a defendant both worked as longshoremen at a shipyard. After the victim was shot to death, the defendant was tried for murder and acquitted. Following the acquittal, the victim’s estate sued the defendant in a wrongful death action. During the civil trial, the victim’s estate called a witness to testify. The witness, who worked with both men at the shipyard, testified that two weeks before the shooting, the victim came to work with a broken nose and said that the defendant had caused it. The attorney for the victim’s estate then asked the witness the following question, “Was the defendant present during your conversation with the victim, and if so, did he say anything about the victim’s broken nose?” The witness replied, “Yes, the defendant was present, and after the victim told me that the defendant broke his nose, the defendant said, ‘And that’s only the beginning.”
    Upon objection by the defendant’s attorney, the witness’s testimony is
    (A) admissible, because it reports a declaration against interest.
    (B) admissible, because it reports the defendant’s adoptive admission of the victim’s assertion.
    (C) inadmissible, because of the principle of collateral estoppel.
    (D) inadmissible, because it is hearsay not within any recognized exception.
A
  1. (B) FRE 801(d) (2) (B), which defines an adoptive admission, provides that a statement is an admission if it is offered against a party and is “a statement of which (the party) has manifested his adoption or belief in its truth,” under circumstances where a reasonable person would deny the statement. The defendant’s silence when the victim told the witness that the defendant broke his nose, coupled by his assertion, “And that’s only the beginning,” clearly evidence the defendant’s admission that he did, in fact, break the victim’s nose. Such a “tacit admission” of a party, namely, the defendant, will be admissibLe against him. Choice (B) is correct. Choice (A) is incorrect, since a declaration against interest is a hearsay exception. But the statement being offered is not hearsay, since it is an admission. Choice (C) is incorrect, since the question of whether or not the defendant broke the victim’s nose would not have to be a necessarily determined (or even actuaLly litigated) issue concerning the murder of the victim. Choice (D) is incorrect, since admissions are defined as non-hearsay.
49
Q
  1. A man was prosecuted for assault and battery after he admitted striking a victim with a pool cue during a barroom argument. The man claimed that he acted in self-defense after he was attacked by the victim, who was drunk and belligerent.
    As his first defense witness, the man calls his neighbor to testify that the man is a good neighbor. The neighbor’s testimony is
    (A) admissible, because it is relevant to show the improbability of the man’s having committed an unprovoked attack.
    (B) admissible, because it is relevant to support the man’s credibility.
    (C) inadmissible, because it is merely the opinion of a character witness.
    (D) inadmissible, because it is not directed toward a pertinent trait of the man’s character.
A
  1. (D) FRE 404 provides that evidence of a person’s character or a trait of his character is not admissibLe for the purpose of proving that he acted in conformity therewith on a particular occasion, except: (1) when evidence of a pertinent trait of his character is offered by the accused, or (2) by the prosecution to rebut the same. The neighbor’s testimony that the man is a “good neighbor,” however, is not admissible because it is not directed to a pertinent trait of the man’s character (e.g., peacefulness, honesty, and the like). On the contrary, the neighbor can testify that the man is “non-violent” or that he has a reputation for “peacefulness” because such traits are pertinent to show circumstantially the improbability that the man was the aggressor in support of his claim of self-defense. Choice (A) is incorrect because evidence of being a good neighbor is not relevant to whether a defendant would commit an unprovoked attack. Choice (B) is wrong, because being a good neighbor is not relevant to the issue of credibility. Finally, choice (C) is inapplicable to this fact pattern. In addition, under FRE 405, character witnesses are required to testify in the form of reputation or opinion testimony. If otherwise proper under the ruLes, there is no problem with a character witness testifying as to his or her opinion.
50
Q
  1. A defendant was prosecuted for aggravated assault after he stabbed a man in the chest during a dispute over a gambling debt. The defendant claimed he acted in self-defense after the man drew a knife on him and made menacing gestures with it. The defendant called a witness to the stand to testify that the defendant had a reputation in the community for being a peaceful person.
    On cross-examination of the witness, the prosecuting attorney asked the witness if he had heard that the defendant had often engaged in fights and brawls.
    Upon objection by the defendant’s attorney, the trial judge should rule the question
    (A) proper, because it tends to show the witness’s knowledge of the defendant’s reputation.
    (B) proper, because it is relevant to prove the defendant’s violent propensities.
    (C) not proper, because it puts into evidence separate and unrelated offenses.
    (D) not proper, because character cannot be proved by generalities.
A
  1. (A) FRE 405 provides that “In all cases in which evidence of character or a trait of character of a person is admissible, proof may be made by testimony as to reputation or by testimony in the form of an opinion. On cross-examination, inquiry is allowable into relevant specific instances of conduct.” Accordingly, the witness may be asked whether he had heard or whether he knew “that the defendant had often engaged in fights and brawls.” This is allowable as to whether the witness (who is a character witness) had knowledge of the defendant’s reputation for fighting. Choice (B) is incorrect because the purpose of the cross-examination is not to contest the defendant’s character, but rather to test the witness’s knowledge. Choice (C) is wrong because the defendant’s prior fights and brawls are related to the character trait of peacefulness upon which the witness testified. Choice (D) is also wrong. Character can be provided through opinion and reputation testimony on pertinent character traits. To an extent, every character trait is a generality, a shorthand expression of a series of actions and interactions overtime.
51
Q
  1. A defendant was charged with illegally selling arms to a foreign country without a government license. During the trial, the defendant admitted selling the weapons, which included missiles, rocket launchers, bazookas, and ammunition, to agents of the foreign country. In his defense, however, the defendant testified that he did not sell the arms as a private citizen, but rather for his employer, a federal agency. In rebutting the defendant’s testimony, the government called a high-ranking official from the federal agency to the stand. He testified that if the defendant were, in fact, a member of the agency, that information would ordinarily be contained in the agency’s employment records. Asked whether the defendant’s employment records were on file with the agency, the official then testified, “I have searched the agency employment records diligently, but I haven’t found any documentation that the defendant was employed by the agency.”
    Upon objection by the defendant’s attorney, the official’s testimony is
    (A) admissible, because it shows the absence of an entry in the record of a public agency.
    (B) admissible, because the official had firsthand knowledge of the information contained in the agency’s employment records.
    (C) inadmissible, because it is hearsay not within any recognized exception.
    (D) inadmissible, because the agency employment records are the best evidence to substantiate whether the defendant was actually in the employ of the governmental intelligence agency.
A
  1. (A) In United States v. De Georgia, 420 F.2d 889 (1969), proof that a car rental agency’s records showed no lease or rental activity regarding a certain vehicle was admissible as tending to show that the defendant in possession of the vehicle had stolen it. FRE 803(10) provides that “to prove the absence of a record, report, statement, or data compilation, in any form, or the nonoccurrence or nonexistence of a matter of which a record, report, statement, or data compilation, in any form, was regularly made and preserved by a public office or agency, evidence in the form of a certification in accordance with FRE 902, or testimony, that diligent search failed to disclose the record, report, statement, or data compilation, or entry” is admissible as a hearsay exception. As a result, the official’s testimony is admissible under this hearsay exception, so choice (C) is incorrect. Choice (D) is likewise incorrect because the mere non-existence of the employment records, not their contents, is at issue; therefore, the best evidence rule does not apply. Choice (B) is wrong because a witness has firsthand knowledge when he actually observes an event (e.g., a car accident). If a witness testifies to someone else’s statement or to a writing prepared by someone else, then a hearsay issue arises. Since the official is testifying to the contents of the employment records (which obviously were compiled by someone else), this presents a hearsay problem.
52
Q
  1. An incumbent was running for re-election as the mayor of a city. During one of his opponent’s campaign speeches, he stated that the mayor had once propositioned a prostitute. A television correspondent who was covering the election was present when the opponent made the statement. She recorded the opponent’s remark about the mayor in her notebook. In a stunning upset, the opponent defeated the mayor in the election. Many of the pollsters attributed the mayor’s loss to the question of his moral character following the opponent’s smear campaign depicting the mayor as a client of prostitutes.
    Following the election, the mayor sued the opponent for defamation, basing his suit on the opponent’s statement that the mayor had “once propositioned a prostitute.” At trial, the mayor calls the television correspondent to testify concerning her recollection of what the opponent said during the campaign speech attended by the television correspondent.
    The television correspondent’s testimony is
    (A) admissible, because the television correspondent recorded the opponent’s statement in her business capacity.
    (B) admissible, because the television correspondent had firsthand knowledge of the opponent’s statement.
    (C) inadmissible, because the notebook entry is the best evidence.
    (D) inadmissible, because it is hearsay not within any recognized exception.
A
  1. (B) Where personal knowledge is offered as testimony to prove an event or statement, the best evidence rule does not apply. In other words, the television correspondent’s testimony is not “reliant” on the notes she recorded in her notebook; she’s testifying as to her own recollection. Therefore, the notebook entry is not the best evidence of the opponent’s remark. Choice (C) is incorrect. Generally, testimony descriptive of non-written transactions is not considered to be within the best evidence ruLe and may be given without producing or explaining the absence of a writing recording the fact. McCormick, p. 707. Likewise, in a defamation case, the actual “tortious words” of slander are oral acts which, by definition, are non-hearsay. Therefore, choice (D) is incorrect. Choice (A) is incorrect because, inasmuch as the opponent’s defamatory statement is non-hearsay, it cannot be admissible under the business record exception. Choice (B) is correct.
53
Q
  1. A witness lived next door to a victim. Late one night, the witness overheard the victim scream, “The defendant, please don’t do it!” The next day the victim was found dead in her bedroom. The defendant was arrested and charged with murdering the victim.
    At trial, the witness proposes to testify to the victim’s statement. Upon objection by defendant’s counsel, the court should rule the witness’s testimony regarding the victim’s statement
    (A) admissible as a dying declaration if the jury determines that the victim believed that her death was imminent.
    (B) admissible as a dying declaration if the judge, by preponderance of the evidence, determines that the victim believed that her death was imminent.
    (C) inadmissible, because the probative value is substantially outweighed by the danger of unfair prejudice.
    (D) inadmissible as hearsay not within any recognized exception.
A
  1. (B) Under FRE 104 (a), “Preliminary questions concerning the qualification of a person to be a witness, the existence of a privilege, or the admissibility of evidence shall be determined by the court.” The judge is not bound by the rules of evidence at this point, except those with respect to privileges. To the extent that these inquiries are factual, the judge acts as a trier of fact. The judge’s decision is final and is not subject to a contrary determination by the jury. On the other hand, the jury is to determine how much, if any, probative value or “weight” to accord to the admitted evidence, as well as to decide issues of credibility (i.e., whether to believe, wholly or in part, the witness’s testimony). The judge not only decides factual issues, he also determines the appLicability of any technical evidentiary rules. He decides, for example, if a dying declarant had a sense of impending death; if an entry was made promptly in the regular course of business; if there was the necessary state of excitement to qualify a declaration as an excited utterance; if a witness is unavailable; or if an original document is unavailable so as to justify the admission of a copy under the best evidence rule. Lilly, Law of Evidence, 2d Ed., p. 459. Finally, note that preliminary questions of fact are ordinarily resolved in both criminal and civil trials by using a preponderance of the evidence standard. The witness’s testimony as to the victim’s statement will be admissible as a dying declaration if the judge, by a preponderance of the evidence, determines the victim believed her death was imminent. Choice (B) is correct. Choice (A) is incorrect because it misstates the role of the judge and jury in determining preliminary questions of admissibility under FRE 104. Choice (C) is also incorrect. There is not enough information in the fact pattern to balance the probative value and prejudicial impact of this evidence. Finally, choice (D) is wrong because the dying declaration is a recognized hearsay exception.
54
Q
  1. A defendant is on trial for attempted murder. The alleged victim is called by the prosecution to testify. During her testimony, the victim recounted the incident and described how the defendant savagely beat her with a baseball bat. She was not asked by the prosecution whether she made any statements during the attack.
    After the victim was excused and left the witness stand, the prosecution called another witness to testify. The witness proposes to testify that when the beating stopped, the victim screamed, “I’m dying. Don’t let the defendant get away with this.”
    Upon objection by the defendant’s attorney, the witness’s proffered testimony is
    a. admissible as an excited utterance.
    b. admissible as a dying declaration.
    c. inadmissible as hearsay not within any recognized exception.
    d. inadmissible, because the victim was not questioned about the statement before being excused.
A
  1. (A) Under FRE 803 (2), an excited utterance is defined as a statement relating to a startling event or condition made while the declarant was under the stress of excitement caused by the event or condition. The witness’s testimony of the victim’s statement, “I’m dying. Don’t let the defendant get away with this,” related to the savage beating (startling event) and was made when the beating stopped (while the victim was still under the stress of excitement). The testimony will be admissible substantively as an excited utterance. Choice (A) is correct. Clearly, the distractor answer was Choice (B). Under FRE 804 (b) (2), a dying declaration is a statement in a criminal homicide case or any civil proceeding made by the declarant while believing his death was imminent and concerning the cause or circumstances of what the declarant believed to be his impending death, provided the declarant is unavailable. First, the victim, the declarant, was not unavailable, since she testified at the trial. Second, the defendant is being charged with attempted murder, which is a non-homicide criminal case, so the federal rules do not allow for a dying declaration to be admissible in this situation. Choice (C) is wrong because the excited utterance is a recognized hearsay exception. Choice (D) is incorrect. There is no requirement that a hearsay declarant, even one who has already testified or will testify at trial, be questioned about their out-of-court statements.
55
Q
  1. A man was on trial for murder. Following a recess, the man was in the hallway outside the courtroom speaking with his attorney. During their conversation, the man said, “So what if I killed him, big deal.” The man’s statement was overheard by the presiding judge as he was walking down the corridor to the rest room. The judge then informed the prosecuting attorney of the man’s comment. After trial is reconvened, the prosecutor calls the judge as a witness to testify to what he heard the defendant tell his attorney.
    Upon objection by defendant’s attorney, may the judge be called as a witness?
    (A) No, because the man’s statement was a confidential communication and protected under the attorney-client privilege.
    (B) No, because a judge may not testify in a matter over which he or she is presiding.
    (C) Yes, because a judge has a duty to disclose incriminating evidence to the prosecution.
    (D) Yes, because the man’s statement was not a confidential communication.
A
  1. (B) According to FRE 605, a judge presiding at the trial may not testify in that trial as a witness. No objection need be made in order to preserve the point. Choices (A) and (D) are incorrect because the man made the statement in a public hallway area. If another individual (other than the judge) had overheard the statement, she would be permitted to testify. The statement would no longer be considered confidential. Choice (C) is wrong in this context because it has nothing to do with the evidentiary issue of whether a judge presiding over a trial can testify at that trial.
56
Q
  1. A shopper purchased a can of sardines, which had been produced by a fish company. A short while later at home, the shopper was preparing a sardine sandwich when she opened the can and, to her astonishment, she saw a decomposed mouse. The shopper became nauseated and vomited upon seeing the mouse.
    After consulting her attorney, the shopper initiated a strict liability action against the company. At trial, the shopper presented into evidence the sardine can with the company’s label, along with other corroborating evidence. The attorney representing the company objected to the introduction of the label on the can.
    The trial court should
    (A) overrule the objection, because the label is self-authenticating.
    (B) overrule the objection, because the label is the best evidence showing that the can was produced by the company.
    (C) sustain the objection, because there is no corroborating evidence that the company placed the label on the can.
    (D) sustain the objection, because the shopper’s proper cause of action is for infliction of emotional distress, not strict liability.
A
  1. (A) According to FRE 902(7), “inscriptions, signs, tags, or labels purporting to have been affixed in the course of business and indicating ownership, control or design” are self-authenticating. In this regard, McCormick states that a manufacturer’s certificate affords prima fade authenticity to the contents of the product. Evidence, pg. 700. Choice (B) is wrong because the best evidence rule applies only when the proponent is attempting to prove the contents or terms of a writing. Here, the plaintiff is not trying to prove the contents of a writing (such as the terms of a contract) but rather is introducing the label as prima fade evidence that the company produced the can of sardines. Choice (C) is incorrect; there is no requirement to corroborate self-authenticating evidence. Choice CD) is wrong because the fact pattern and question pertain to an evidentiary issue and not the appropriate cause of action for the case.
57
Q
  1. A bank was robbed by a person wearing a yellow ski mask, goggles, and a fluorescent yellow jumpsuit. Three weeks after the robbery, a defendant was arrested and charged with the crime. At trial, the prosecuting attorney seeks to introduce evidence that the defendant had previously committed a robbery wearing a yellow ski mask, goggles, and a fluorescent yellow jumpsuit.
    Upon objection by the defendant’s attorney, the proffered evidence should be
    (A) admissible as circumstantial evidence that the defendant committed the crime as charged.
    (B) admissible, to show the identity of the defendant.
    (C) inadmissible, because the probative value of the evidence is outweighed by the danger of unfair prejudice.
    (D) inadmissible, because it constitutes improper character evidence.
A
  1. (B) Under highly tested FRE 404(b), evidence of other crimes, wrongs, or acts is not admissible to prove the character of a person in order to show that he acted in conformity therewith. It may, however, be admissible for other purposes, such as proof of motive, opportunity, intent, preparation, plan, knowledge, identity, or absence of mistake or accident. This so-called MIMIC rule allows admission of other criminal acts as circumstantial character evidence to show conduct in conformity therewith, provided that the purpose for admission of such evidence is not solely to show criminal disposition or propensity to commit the crime being tried. Choice (A) is a good answer, but it is not as complete as choice (B). Without a direct identification of the defendant, the prosecution will necessarily have to present a circumstantial case. The indirect evidence of identity is circumstantial evidence. Choice (C) is incorrect. The fact pattern does not provide sufficient information to weigh the probative value and prejudicial effect of the evidence. Finally, choice (0) is wrong because the evidence is being offered for a non-character purpose.
58
Q
  1. After being passed over for a promotion, an aeronautic engineer became a whistleblower. He contacted a government agency to state that the company for which he worked was submitting false safety reports on newly built aircraft. When the company learned that the engineer was leaking this information to the government agency, he was fired from his job.
    Afterward, the engineer sued the company for wrongful termination of employment. During the discovery stage of litigation, the engineer was deposed by the company’s attorney. In his deposition, the engineer stated that the company submitted false safety reports to the government agency to cover up structural defects in its aircraft.
    A pilot was injured when one of the company’s
    airplanes he was piloting crashed. The pilot asserted
    a strict products liability tort against the company.
    At thal, the pilot seeks to introduce into evidence portions of the engineer’s deposition from his wrongful termination lawsuit against the company. Assume that the engineer is unavailable to testify at trial.
    Upon objection by the company, the trial court judge should rule the engineer’s deposition testimony
    (A) admissible as former testimony.
    (B) admissible as a vicarious admission.
    (C) inadmissible as hearsay not within any recognized exception.
    (D) inadmissible, because the company did not have the opportunity to cross-examine the engineer on the liability issue for which the statement is now being offered.
A
  1. (A) Usually, formertestimony questions will involve the same parties where each had the opportunity to interrogate the witness at the earlier trial. In this question, we have a plaintiff (who was not party to the first suit) seeking to introduce former testimony against the same defendant. According to Lilly, “using the testimony against the same defendant, which was a party to both suits and had an opportunity at the first trial to interrogate the witness, appears to fit comfortably within the exception.” Evidence, p. 286. On the other hand, Lilly points out that if a plaintiff seeks to use all or part of the former testimony against a new defendant (who did not have an opportunity to interrogate the witness), then the testimony should be excluded. For these reasons, choice (A) is the correct answer, and choice (D) is incorrect. Choice (B) is wrong. Under FRE 801(d)(2)(D), a vicarious admission by an employee can occur only if made duringthe employment relationship. At the time of this deposition, the engineer no longer worked for the company. Choice (C) is incorrect. While the evidence is hearsay, it is admissible under a recognized hearsay exception.
59
Q
  1. A plaintiff was driving her car when she stopped at a red light at an intersection. A defendant, who was behind her, did not see the red light and failed to stop. He crashed into the rear of the woman’s car and pushed her onto the curb and into a tree.
    When the plaintiff got out of her car, she felt fine physically, but was emotionally upset when she saw that the hood and trunk of her car were severely damaged. The next day she received an estimate from the mechanic for $1,950 to repair her car. She consulted an attorney the following afternoon and, that evening, suddenly developed shooting back pains.
    The plaintiff filed a lawsuit against the defendant based on a theory of negligence. One day prior to trial, two years after the accident, the plaintiff went to take a photograph of the accident scene.
    At trial, the photograph is
    (A) inadmissible, if the defendant objects, because the photograph is non-verbal hearsay.
    (B) inadmissible, if the defendant objects, because the photograph was taken two years after the accident.
    (C) admissible, if the plaintiff identifies the photograph at trial.
    (D) admissible, if the photograph correctly and accurately portrayed the accident scene.
A
  1. (D) As a general rule, a photograph is viewed merely as a graphic portrayal of oral testimony and becomes admissible only when a witness has testified that it is a correct and accurate representation of relevant facts personally observed by the witness. Moreover, the witness who lays the foundation need not be the photographer, but he needs to know about the facts represented or the scene photographed. Once this knowledge is revealed, the witness can testify as to whether the photograph correctly and accurately portrays these facts. Choice (A) is wrong because a photograph is not an “out-of-court statement.” Choice (B) is incorrect. The photograph would be admissible if it met the foundational elements, even over the defendant’s objection. Choice (C) is wrong because it suggests that admissibility depends only on the plaintiff’s identification of the photograph. In fact, she (or, for that matter, any other witness familiar with the location), must establish the foundational elements of “correct and accurate representation” of the location.
60
Q
  1. A man and a woman were involved in a car accident that occurred when the man abruptly switched lanes in front of the woman without signaling. Unable to stop, the woman hit the man’s car into a median. Immediately after the accident, as the two drivers emerged from their vehicles, a pedestrian, obviously emotional and upset from having just observed an accident, ran up to the man and shouted, “You careless driver, you. Didn’t you ever learn that you’re supposed to use a turn signal when you switch lanes?” The pedestrian is never identified and is not present at trial.
    The pedestrian’s statement is
    (A) admissible as non-hearsay.
    (B) admissible, even though it is hearsay.
    (C) admissible under the excited utterance exception.
    (D) inadmissible, because the bystander cannot be identified and is not present at trial.
A

6o. (C) Under FRE 803(2), a statement relating to a startling event or condition made while the declarant was under the stress of excitement caused by the event or condition is not excluded by the hearsay rule. The theory behind the “excited utterance” exception is simply that circumstances may produce a condition of excitement that temporarily stills the capacity of reflection and, thus, produces utterances free of conscious fabrication. Spontaneity is the key factor in determining whether the statement was, indeed, made without any conscious reflection. It is important to note here that choice (B) is not incorrect. Choice (A) is wrong. The statement would not qualify as an admission under FRE 801. Choice (D) is incorrect. If the proper foundational elements are met (as they are here), there is no requirement either to identify the declarant or produce him at trial.

61
Q
  1. An investor sued a corporation for stock fraud. In presenting his case-in-chief, the investor sought to introduce an issue of a newspaper to show the corporation’s stock price on that given day.
    Upon objection by the corporation’s attorney, this evidence should be
    (A) admitted, under the business records exception.
    (B) admitted, under the market reports exception.
    (C) excluded, because the newspaper copy does not fit within any established exception to the hearsay rule.
    (D) excluded, because it violates the best evidence rule.
A

6i. (B) This exception encompasses “newspaper market reports, telephone directories, city directories, and mortality and annuity tables used by life insurance companies.”
Federal Rules of Evidence 803 (17). The basis of trustworthiness is general reliance by the public, or by a particular segment of it, and the motivation of the compiler to foster reliance by being accurate. Choice (A) is not the best answer because business records contemplate the existence of “both an informer who supplies the information from personal knowledge and an entrant-recorder who makes the written entry in the regular course of the business activity.” This, obviously, is not the case with the publication of newspaper market prices. Choice (C) is wrong for the same reasons that choice (B) is the correct answer. Choice CD) is incorrect. The best evidence rule applies when the contents of a document (in other words, what the document actually said) are at issue, which does not appear to be the case here.

62
Q
  1. A teenager shot and killed a man she said tried to attack her as she was walking home from an all night market. The teenager, 18 years old and 6 feet 3 inches tall, said a very short man placed a knife against her throat and unzipped her dress as she was walking down the street. She pulled a gun from her purse and shot him. The man died of a chest wound. Police said the man, who was 5 feet 5 inches tall and weighed 130 pounds, was an ex-felon, twice convicted of attempted rape. The teenager is charged with the murder of the man, as a result of the above described incident.
    At trial, the teenager wishes to offer evidence of the fact that the man was previously convicted of attempted rape. The evidence would be
    (A) admissible, because it proves the man’s disposition to commit rape.
    (B) admissible, because the victim’s prior convictions would be relevant on the issue of the defendant’s self-defense.
    (C) inadmissible, because the evidence is being used to prove propensity.
    (D) inadmissible, because it violates the Dead Man’s Statute.
A
  1. (B) Past crimes may be introduced to show anything but disposition to commit a present crime or wrong; because no other legitimate reason, beyond disposition, appears to be raised by these facts, choice (C) is the correct answer. While there are special rules concerning sexual assault cases, this question does not involve a defendant being charged with a sexual assault crime (the defendant here is the teenager, not the man). Moreover, in a criminal case, where self-defense is at issue, a criminal defendant may offer evidence of the victim’s violent character to prove that the victim was the first aggressor; however, this may only be offered in the form of reputation or opinion evidence, not specific act evidence. Finally, the fact that the prior acts in question are convictions does not appear to impact this question, since while convictions may be specialized in terms of how they may be used to attack the credibility of a witness, this issue is not raised by the facts (the man is not a witness). For these reasons, choices (A) and (B) are incorrect. Choice (D) is incorrect, since the Dead Man’s Statute is not a rule of evidence and is inapplicable here.
63
Q
  1. A man was arrested and charged with sexually assaulting a child. At trial, in its case-in-chief, the prosecution seeks to introduce evidence that the man had twice been previously involved in similar cases with other children. The prosecution argues that these past crimes demonstrate the likelihood that the man molested this child.
    Defense counsel objects to the evidence regarding other children. The judge should find the evidence
    (A) admissible, because the man’s past crimes may be introduced to show that he is more likely to have committed the crime with which he is charged.
    (B) admissible, because a common plan or scheme may be proven by other similar criminal acts.
    (C) inadmissible, because past crimes may not be introduced to prove disposition to commit certain acts.
    (D) inadmissible, because the prosecution cannot initiate evidence of bad character.
A
  1. (A) According to FRE 413, in a criminal case in which the defendant is accused of an offense of sexual assault, evidence of the defendant’s commission of another offense or offenses of sexual assault is admissible and may be considered for its bearing on any matter to which it is relevant. For this same reason, choice (C) is the wrong answer. Choice (B) is a good answer, but not as strong as choice (A) in this case because FRE 413 permits the introduction of this evidence for any purpose. Choice (D) is incorrect. Under FRE 413, the prosecution can introduce this evidence on its case-in-chief.
64
Q
  1. A defendant has been charged with committing the armed robbery of a convenience store. During the defendant’s case-in-chief, the defendant called several character witnesses to testify as to his good character. As part of the prosecution’s rebuttal, the defendant’s ex-wife was called to testify that during their marriage the defendant had a violent temper, beat her frequently, and once threatened her with a knife.
    The ex-wife’s testimony should be ruled
    (A) admissible, because it is relevant to the crime charged.
    (B) admissible, because the defendant opened the door by introducing evidence of his good character.
    (C) inadmissible, because the defendant will be able to prevent the testimony using the spousal testimony privilege.
    (D) inadmissible, because character evidence may not be introduced in this manner.
A
  1. (D) Under the FRE, once the defendant in a criminal trial opens the door by introducing evidence as to their good character, the prosecution may so rebut. However, the prosecution is constrained in the same manner as the defendant in that the evidence of character must be introduced through reputation or opinion evidence only, no specific acts. Therefore, choice (A) is incorrect. While the ex-wife’s testimony may be relevant to the crime charged, it is in the form of specific acts and, therefore, inadmissible underthe FRE. Choice (B) is incorrect for the same reasons stated above. While it is true that the defendant properly opened the door, the prosecution is attempting to rebut with specific acts, which are inadmissible. Choice (C) is incorrect because (1) the spousal testimony privilege ends at divorce, and (2) where applicable, the spousal testimony privilege allows the witness-spouse to refuse to testify, but may not be used by the defendant-spouse to prevent the testimony. Choice CD) is, therefore, the correct answer and the evidence is inadmissible.
65
Q
  1. A famous comedian was acting as guest host of a popular late-night television talk show. Also appearing on the show was a professional football player. During the course of the television show, the comedian continually referred to the player as a “bench warmer.” Angered by the comedian’s insults, the player told him on the telecast, “Listen, I am a starter, not a bench warmer.”
    The football player sued the comedian for slander. At trial, the football player calls a witness who saw the show in question to describe what he had heard on the broadcast. The comedian’s attorney objects, claiming that the witness’s testimony would be hearsay.
    The trial judge should
    (A) sustain the objection, because the witness’s testimony is not the best evidence.
    (B) sustain the objection, because the witness does not have firsthand knowledge whether the comedian was reading from a script.
    (C) overrule the objection, because defamatory statements aren’t hearsay.
    (D) overrule the objection, because the witness is qualified to render a lay opinion as to what he heard.
A
  1. (C) The hearsay rule forbids evidence of out-of-court assertions to prove the facts asserted in them. Clearly, proof of utterances and writings may be made with an almost infinite variety of other purposes, not resting for their value upon the veracity of the out-of-court declarant and, hence, falling outside the hearsay classification. A few of the more common types of non-hearsay utterances include (1) verbal acts, (2) utterances and writings offered to show effect on hearer or reader, (3) declarations offered to show circumstantially the feelings or state of mind of the decLarant, and (4) knowLedge. Choice (C) is correct because (a) defamation, (b) slander, and (c) deceit are admissible as non-hearsay verbal acts. Choice (A) is incorrect because it misstates the best evidence rule, which applies only when the contents of documents, photographs, or recordings are in dispute. Choice (B) is irrelevant to the issue and, therefore, incorrect. Choice (D) is also wrong. The witness is not rendering an opinion, but rather testifying about something he personally heard and observed.
66
Q
  1. A college student and a man got into a fight at a bar while playing pooi. The college student allegedly started the fight when he struck the man in the head with a pool cue after the man sank a game-winning shot. At the college student’s trial for assault, the prosecution called one of the college student’s classmates to testify. The classmate was not at the bar during the fight. The classmate offered to testify that the next day at class, he approached the college student and said, “It was wrong of you to start a fight with the man over a game of pool,” at which point, the college student bowed his head and walked away.
    The classmate’s testimony is
    (A) admissible as part of the res gestae.
    (B) admissible as an admission.
    (C) inadmissible as hearsay not within any recognized exception.
    (D) inadmissible as self-serving.
A
  1. (B) If a statement is made by a person in the presence of a party to the action, containing assertions of facts which, if untrue, the party would, under all circumstances, naturally be expected to deny, his failure to speak has traditionally been receivable against him as an admission. See Wigmore, Evidence, Section 1071. Choice (A) is wrong because a conversation the day after an incident is not part of the incident’s res gestae. Choice (C) is also incorrect. By definition, an admission is not hearsay under FRE 801(d)(2). Finally, choice (D) is incorrect. The critical aspect of this fact pattern is the college student’s admission by failure to respond to the classmate, not the classmate’s motives for asking the question or testifying at trial.
67
Q
  1. While a ski lift was ascending the mountain, the overhead cable broke, dropping a skier 15 feet to the ground. The skier suffered a broken pelvis on account of the fall. Thereafter, the skier sued the ski resort alleging negligent maintenance of the ski lift machinery.
    The ski resort’s skier’s attorney took a discovery deposition of a witness who was seated behind the skier at the time of the accident. The witness died shortly after the deposition. At trial, the skier offers the witness’s deposition testimony into evidence.
    Upon objection by the ski resort’s attorney, the deposition is
    (A) admissible as former testimony.
    (B) admissible, provided that the witness’s statements are more probative on the point than any other evidence the skier can procure through reasonable efforts and that the skier has given the ski resort reasonable notice that she would offer the deposition.
    (C) inadmissible, because it violates the ski resort’s right to confront the witnesses against itself.
    (D) inadmissible, because it is hearsay not within any recognized exception.
A
  1. (A) Former testimony is defined under FRE 804(b) as “Testimony given as a witness at another hearing of the same or a different proceeding, or in a deposition taken in compliance with law. In the course of the same or another proceeding, if the party against whom the testimony is now offered, or in a civil action or proceeding, a predecessor in interest, had an opportunity and similar motive to develop the testimony by direct, cross, or redirect examination” . . . if the declarant is unavailable as a witness. Choice (A) is correct because the ski resort, the party against whom the deposition is being offered, deposed the deceased (i.e., unavailable) declarant, the witness. At that time, the ski resort had the opportunity to examine the witness, thereby satisfying the requirements for admissibility of former testimony. Choice (B) is incorrect because the former testimony exception does apply. Choice (C) is incorrect because a defendant’s right to confrontation allows an accused in a criminal case the right to effectively confront and cross-examine those who testify against him at trial. Choice (D) is incorrect because the statement is hearsay within a recognized exception.
68
Q
  1. A plaintiff files suit in federal district court against a lawnmower manufacturer, alleging strict liability, negligent manufacture, and breach of warranty. The suit is to recover damages for injuries. suffered by the plaintiff when his gasoline-powered lawnmower exploded, while the plaintiff was mowing his front lawn. At trial, the plaintiff calls a man who witnessed the explosion to testify as an eyewitness. The witness suffers from a mental illness that affects his perception of reality.
    Which of the following is true regarding the witness’s mental condition?
    (A) It disqualifies him from testifying
    (B) It maybe proved to impeach him as a witness
    (C) It has no bearing on either his competence or his credibility.
    (D) It must not be referred to because it is unfairly prejudicial.
A
  1. (B) The FRE specify no qualifications for mental competency. FRE 601 effectively eliminates as grounds for competency the following: age, religious belief, mental incapacity, conviction of a crime, marital relationship, and interest in the litigation as a party or attorney. Such matters, generally regarded at common law as grounds for disqualification, survive under the FRE as avenues for impeachment. In this question, the witness’s mental state does not disqualify him. Choice (A) is incorrect, but it does bear upon his competence and credibility. Choice (C) is incorrect, since the defendant can use the witness’s mental illness for purposes of impeachment. Choice (B) is correct. There are two basic competency requirements under the FRE:
    under FRE 602, a witness must possess personal knowledge; under FRE 603, every witness must declare to testify truthfully (i.e., be capable of understanding the obligation to tell the truth). Such a two-fold approach to competency is often referred to as a test of minimum credibility.
69
Q
  1. A testator died and through his will gave his entire estate to charity, but the gift failed because the will was executed 10 days prior to the testator’s death. A man then filed a claim against the estate, alleging that he was the son and heir of the testator. At the trial on the man’s claim, the man was unable to produce any proof of a ceremonial marriage, but established the following by undisputed evidence: (a) that his mother and the testator had cohabited for a number of years; and (b) that his mother had predeceased the testator by one day. Assume that this jurisdiction has a relevant Dead Man’s Statute in effect.
    At trial, the man’s attorney called a clerk at a winter resort hotel. The clerk testified that the testator and the man’s mother had come to the hotel for many years and he often saw the testator sign the hotel register as “Mr. and Mrs. Testator.”
    The trial court should rule the clerk’s testimony
    (A) admissible, because the clerk had personal knowledge of the matter.
    (B) admissible as a pedigree exception to the hearsay rule.
    (C) inadmissible, because the clerk’s testimony is not the best evidence.
    (D) inadmissible, under the Dead Man’s Statute.
A
  1. (A) The clerk’s testimony would be deemed admissible, since he had personal knowledge of the fact that the testator signed the hotel register as “Mr. and Mrs. Testator.” The clerk actually observed the testator signing the hotel register. Choice (B) is wrong because the pedigree exception covers community reputation of a person’s pedigree. In this case, the clerk is offering direct evidence, not reputation evidence, that the testator signed the hotel register in a particular way. Choice (C) is also wrong. The best evidence rule is applicable only where the terms or contents of a writing/document are in issue. Here, the terms of the hoteL registry are not in issue; the man is trying to prove that he is the testator’s son and heir to his estate. Moreover, in accordance with McCormick, Law of Evidence, Second Edition, Section 233, testimony descriptive of non-written transactions is not generaLly considered to be within the scope of the present rule and may be given without producing or explaining the absence of a writing recording the facts, such as evidence of a marriage, without production of the marriage certificate. Choice (D) is incorrect and somewhat misleading. Dead Man’s Statutes prohibit claimants to an estate from testifying about conversations they had with the decedent. In this case, the clerk is not a claimant to the estate.
70
Q
  1. A year ago, a very wealthy man died in a car accident. A woman, whose mother worked as a secretary to the wealthy man for years, filed a claim against the estate, alleging that she was the wealthy man’s daughter. At trial, the woman’s attorney called the wealthy man’s butler to testify. The butler testified that it was Common knowledge in the household that the wealthy man and the woman’s mother were having an affair. The butler also testified that the wealthy man had often confided in him and told him shortly after the woman’s birth that the woman was the wealthy man’s daughter.
    Upon objection by the attorney for the executor of the wealthy man’s estate, the court will most likely
    a. sustain the objection bc butler’s testimony would be violative of hearsay rule.
    b. sustain, bc butler’s testimony would be violative of Dead man’s statute.
    (C) overrule the objection because the butler’s testimony would be relevant to the ultimate issue in the case.
    (D) overrule objection because the butler’s testimony would qualify under the pedigree exception to the hearsay rule.
A
  1. (D) Under FRE 803 (19), the butler’s testimony would be admitted under the pedigree exception to the hearsay rule as “reputation among members of his family by blood, adoption, or marriage, or among his associates, or in the community, concerning a person’s birth, adoption, marriage, divorce, death, legitimacy, relationship by blood, adoption, or marriage, ancestry, or other similar fact of his personal or family history.” Choice (A) is wrong. Even though the evidence is hearsay, it satisfies a recognized hearsay exception. Choice (B) is incorrect because the butler is not a claimant to the estate and, therefore, is not prohibited by a Dead Man’s Statute from testifying about conversations with the decedent. Choice (C) is a good answer, but the wrong choice. The butler’s testimony is, indeed, relevant to the ultimate issue in the case. Choice (D) is a better choice than choice (C) because it specifically addresses the hearsay issues raised by the use of this evidence.
71
Q
  1. A nephew brings an action against the administrator of his uncle’s estate based upon a contract for services rendered in the management of his uncle’s property. In order to rebut the presumption that the services rendered were gratuitous, since the nephew was a relative of his uncle, the nephew called his sister-in-law as a witness. Assume that this jurisdiction has a relevant Dead Man’s Statute in effect.
    The sister-in-law testified that she had lived in the nephew’s house, knew his uncle, and that she was familiar with the uncle’s handwriting. The nephew’s attorney asked her to look at a letter and to tell the court whether it was written by the uncle.
    Upon objection by the attorney for the administrator of the estate, the trial judge would most likely
    (A) sustain the objection, because the sister-in-law is not a handwriting expert.
    (B) sustain the objection, because of the Dead Man’s Statute.
    (C) overrule the objection, because the letter qualifies as a past recollection recorded, an exception to the hearsay rule.
    (D) overrule the objection, because an authenticating witness need not be an expert if familiar with the handwriting of the person in question.
A
  1. (D) It is generally heLd that anyone familiar with the handwriting (or signature) of a given person may supply authenticating testimony in the form of his opinion that a writing or signature is in the handwriting of that person. In this regard, adequate familiarity may be present if the witness has seen the person write, or if he has seen writings purporting to be those of the person in question. FRE 901 (b) (2). Choice (A) is incorrect because a Lay witness can authenticate handwriting. Choice (B) is wrong because the sister-in-law is not a claimant to the estate. Choice (C) is incorrect. The past recollection recorded hearsay exception applies when (1) an in-court witness has forgotten something; (2) the attorney has unsuccessfully attempted to refresh the witness’s recollection; and (3) a writing is available containing matters “shown to have been made or adopted by the witness when the matter was fresh in the witness’s memory and to reflect that knowledge correctly.”
72
Q
  1. A tenant lived in a small apartment building that was owned by a landlord. In July, the landlord was killed in an accident. In August, the tenant brought an action against the administrator of the landlord’s state. The tenant alleged that he and the landlord had a contract under the terms of which the landlord was supposed to pay him $50 a week to mow the lawn and perform small maintenance repairs for the other tenants. He claimed that the landlord had not paid him for the past six months. He testified that there was a written contract and that the landlord kept the only copy of it in his home. At trial, he called the landlord’s accountant to testify about the contract. The accountant testified that she had seen the contract. She further testified about the terms of the contract, consistent with the tenant’s testimony. She testified that the contract, along with many other documents pertaining to the landlord’s business, had most likely been destroyed when the landlord’s business office was robbed a few months earlier.
    The trial judge should rule that the accountant’s. testimony is
    (A) admissible as secondary evidence under the circumstances.
    (B) admissible as a written admission by the party opponent.
    (C) inadmissible under the Dead Man’s Statute.
    (D) inadmissible, because the accountant’s testimony is not the best evidence.
A
  1. (A) Under FRE 1004, secondary evidence of the contents of a writing, recording, or photograph is admissible if the originals are lost or destroyed. Most courts hold that the exclusion of secondary evidence should be confined to cases where the writings are purposely destroyed to avoid producing them. Thus, choice (A) is correct and choice (D) is incorrect. Choice (B) is wrong because there is no written admission to enter into evidence. Choice (C) is incorrect because the accountant is not a claimant to the estate and, in any event, is not testifying about any conversations with the decedent.
73
Q
  1. A minor-plaintiff suffered permanent head injuries as a result of being hit by a car driven by a defendant. The plaintiff’s father commences this action as a parent in his own right and on behalf of his son.
    At trial, the plaintiff’s attorney called the only witness to the accident, the plaintiff’s friend, aged seven. The friend was four years of age at the time of the accident. It was determined that the friend lacked the capacity to perceive and relate the accident at the time it actually occurred.
    The court should rule that the friend would be
    (A) competent to testify, because he had personal knowledge of the accident.
    (B) competent to testify, because a seven-year-old is presumed to be chronologically mature.
    (C) incompetent, because he lacked the capacity to perceive and relate the accident at the time of its occurrence.
    (D) incompetent, because all children under the age of 10 are deemed incompetent to testify.
A
  1. (C) It is necessary to measure the competency of an infant witness at the time of the occurrence or event (about which he is called to testify), and not at the time of trial. There is no rule that excludes an infant of any specified age from testifying, but in each case, the traditional test is whether the witness has intelligence enough to make it worthwhile to hear him at all and whether he feels a duty to tell the truth. Choices (B) and (D) are wrong because they both incorrectly state presumptions that do not exist under the law. Choice (A) is incorrect. Although witnesses are presumed competent under FRE 601, personal knowledge alone does not make a witness competent to testify. Competency factors under the law include whether the witness can make an adequate oath or affirmation and communicate with a jury.
74
Q
  1. One day, while riding his motorcycle through a residential area, a man hit and injured a child who darted into his path. The child’s mother files suit against the man. At trial, the mother is then called to testify as to her conversation with the man at the hospital where her son was rushed immediately following the accident. According to the mother, the man told her, “Don’t worry, my insurance company will pay for all of your son’s medical expenses.”
    If offered into evidence, the court will most likely rule that the mother’s statement is
    (A) admissible, on the issue of the defendant’s negligence.
    (B) admissible, to show that the defendant is able to pay for the medical expenses.
    (C) inadmissible, because the testimony was highly prejudicial.
    (D) admissible, only to prove ownership or control of the motorcycle.
A
  1. (D) Under FRE 411, evidence that a person was or was not insured against liability is not admissible upon the issue of whether he acted negligently or otherwise wrongfully. The rule does not require the exclusion of evidence of insurance when offered for another purpose, such as proof of agency, ownership, or control of the vehicle. Therefore, choice (D) is correct, and choices (A) and (B) are incorrect. Choice (C) is also incorrect because it incorrectly states the standard for excluding evidence under FRE 403. Under Rule 403, evidence may be excluded if its probative value is substantially outweighed by the danger of unfair prejudice. Evidence is not excluded merely because it is “highly prejudicial.”
75
Q

skip

A

skip

  1. (D) Generally, a spouse (whether or not a party) has a privilege during the marital relationship and afterward to refuse to disclose and to prevent another from disclosing a communication if he claims the privilege, and that the communication was made in confidence between him and his spouse while they were husband and wife. However, the privilege is lost if the communication was made, in whole or in part, to enable or aid anyone to commit, or plan to commit, a crime or fraud. Thus, choice (D) is correct, and choice (A) is incorrect. Choice (B) is also wrong, since the marital privilege does not apply to fraud actions. Finally, choice (C) is incorrect because either spouse can claim the privilege.
76
Q
  1. A plantiff sued a defendant for injuries suffered m a fall on the sidewalk of the defendant’s home. The plaintiff’s complaint alleged that the walk was covered by a thick sheet of ice, which had been negligently left there for several days by the defendant. In his answer, the defendant set forth that the ice formed overnight and that the plaintiff fell before the defendant had a chance to remove it. During the trial, a physician, whose office was next door to the defendant’s home, testified that he saw the plaintiff fall and that the ice, which had been there for several days was at least two inches thick. On cross-examination, counsel for the defendant asked the physician the following question: “During your treatment of the plaintiff on the day in question, is it not true that he told you his fall resulted from a loss of equilibrium after he suffered from dizziness?”
    Upon objection by the plaintiff’s attorney, the physician’s testimony will be ruled
    (A) admissible, because it is reasonably pertinent to diagnosis or treatment.
    (B) admissible, because the physician had personal knowledge of the plaintiff’s injuries.
    (C) inadmissible, because the defendant’s counsel failed to lay a proper foundation.
    (D) inadmissible, because the physician’s testimony is not relevant to prove that the plaintiff’s alleged injuries are false or exaggerated.
A
  1. (A) The FRE allow not only statements of past symptoms and medical history as exceptions under the hearsay rule, but also the cause or source of a patient’s past physical condition, insofar as reasonably pertinent to diagnosis or treatment. In this regard, the plaintiff’s statements to the physician would be admissible as reasonably pertinent to diagnosis and/or treatment regarding the cause or source of his injuries. Choice (B) is not the best answer here. While the physician might have personal knowledge of the injuries the plaintiff suffered from the fall, he is also privy to knowledge about past symptoms and medical history. Choice (C) is incorrect. The question occurred on cross-examination and required no foundation. Choice (D) is wrong because the physician’s evidence is highly relevant in this case.
77
Q

skip

A

skip

  1. (C) In accordance with FRE 503 (b), the attorney-client privilege provides that a client has a privilege to refuse to disclose and to prevent any other person from disclosing confidential communications made for the purpose of facilitating the rendition of professional legal services to the client, between himself or his representative and his attorney or his attorney’s representative. Choice (A) is incorrect. Whatever the man said to his attorney might have been an admission, but it was covered by the privilege. Choice (B) is also wrong. The attorney-client privilege survives the attorney-client relationship. Finally, choice (D) is an incorrect statement. Like most evidence precluded by privilege, this statement is highly relevant.
78
Q
  1. A shopper slipped and fell on a wet spot at the local mall, suffering injuries. The shopper has sued the mall owner for negligence. At trial, the shopper’s housekeeper testified that the shopper was bedridden and unable to work for a month because of the sprained back she suffered when she fell at the mall. The housekeeper also testified that about one week after the shopper returned home from the hospital, the shopper told her, “My back is really killing me. The pain is excruciating. It’ll be a miracle if I can ever stand up again.” The mall’s counsel then moved to strike the housekeeper’s testimony.
    The motion should be
    (A) granted, because the housekeeper’s testimony is hearsay not within any recognized exception.
    (B) granted, because the housekeeper’s testimony would be self-serving.
    (C) denied, because the housekeeper’s testimony would be admissible as an adoptive admission.
    (D) denied, because the shopper’s statement to her housekeeper would be admissible as a declaration of present bodily condition.
A
  1. (D) Choice (D) is correct, since the shopper’s statement, “My back is really killing me,” is admissible as a declaration of present bodily condition. Under FRE 803(3), such statements need not have been made to a physician. Thus statements to hospital attendants, ambulance drivers, or even members of the family might be included. Choice (A) is wrong. Although the statement is hearsay, it falls within a recognized exception. Choice (B) is incorrect. The self-serving nature of hearsay testimony does not affect its admissibility. Choice (C) is also incorrect. A statement of present bodily condition is not an admission.
79
Q
  1. A defendant was arrested and charged with forgery for unlawfully signing her ex-husband’s signature on his pay check. Before trial, the defendant entered into plea bargain negotiations with the prosecution. During the course of these negotiations, the defendant sent a letter to the prosecuting attorney in which she stated: “I don’t want to go to jail, so if you recommend a suspended sentence, I’ll plead guilty and admit that I forged my ex-husband’s signature on the check.” Subsequently, the plea bargain negotiations fell through.
    Shortly thereafter, the defendant was arrested and charged with forging her ex-husband’s signature on his next pay check. She has denied culpability regarding the second forgery charge. At trial for the alleged second forgery, the prosecution seeks to introduce the letter the defendant sent to the prosecution during her earlier plea bargain negotiations.
    The defendant’s attorney objects to the admissibility of the letter. The trial court judge should rule the letter is
    (A) admissible as evidence of the defendant’s propensity for committing the crime.
    (B) admissible as an admission.
    (C) inadmissible, because it was written during the course of plea bargain negotiations.
    (D) inadmissible, because specific instances of misconduct are not admissible in a criminal case.
A
  1. (C) FRE 410 provides that evidence of the following is not, in any civil or criminal proceeding, admissible against the defendant who made the plea: “…(4) any statement made in the course of plea discussions with an attorney for the prosecuting authority which does not result in a plea of guilty or which results in a plea of guilty later withdrawn.” The thrust of the rule is to promote the plea bargaining process by freeing the accused from the apprehension that his declarations may be used to his detriment in the pending criminal trial or in subsequent litigation. In this question, the defendant’s letter to the prosecution admitting to the forgery of the first check was written during the course of plea bargaining negotiations and, therefore, may not be used by the prosecution as an admission in the subsequent trial for alleged forgery of the next pay check. Choice (C) is the correct answer, and choice (B) is incorrect for the same reasons; plea agreements and negotiations often contain admissions. Choice (D) is incorrect because character evidence in the form of specific acts in a criminal case may be offered by the prosecution under the so-called MIMIC rule, but two similar instances of conduct do not establish a common plan or scheme. Choice (A) is incorrect because FRE 404 prohibits the use of propensity evidence in criminal cases.
80
Q
  1. A plaintiff sued an insurance company to recover life insurance proceeds. At trial, the issue before the court is the insured’s correct date of birth.
    Which of the following is inadmissible if offered to prove the insured’s correct date of birth?
    (A) A family portrait engraved with the ages of the family members.
    (B) A family bible with the insured’s birth date inscribed on the cover.
    (C) A photocopy of the insured’s birth certificate.
    (D) A sworn affidavit from the insured’s brother verifying the insured’s birth date.
A
  1. (D) One of the oldest exceptions to the hearsay rule encompasses statements concerning family history, such as the date and place of births and deaths of members of the family and facts about marriage, descent, and relationship. Under FRE 803 (13), “statements of fact concerning personal or family history contained in family Bibles, genealogies, charts, engravings on rings, inscriptions on family portraits, engravings on urns, crypts, or tombstones” are admitted. Therefore, choices (A) and (B) are wrong. Choice (C) is incorrect because the photocopy of the insured’s birth certificate would be admissible under FRE 1003, since a duplicate is admissible to the same extent as the original. Choice (D) is the best choice because the affidavit is inadmissible hearsay, since it is an out-of-court statement that is being offered for its truth.
81
Q
  1. After two men robbed a local bank, they left the bank and went in different directions with a plan to meet three days later at the train station. Two days after the robbeiy, one of the men was involved in an auto accident. The man was rushed to the hospital and taken to the x-ray room. A doctor positioned the man so that the x-ray technician would be able to take x-rays of his lower back area. Based on the man’s complaints about the area of pain, the doctor stated, “It seems as if you might have a fractured pelvis. If you do, you’ll have to stay in bed for several weeks.” The man responded that he couldn’t do that because he had to meet the other man tomorrow. A technician who was in the room at the time overheard the man’s response to the doctor. Subsequently, both men are charged with robbery and conspiracy.
    At the man’s trial, as evidence of a conspiracy, the prosecution attempts to introduce the doctor’s testimony that the man said, “I have to meet the other man tomorrow.” The defense objects.
    This evidence is
    (A) inadmissible, because it violates the physician- patient privilege.
    (B) admissible, because the technician overheard the conversation.
    (C) admissible, because of the nature of the man’s statements.
    (D) admissible, because the statement was not confidential, under the circumstances.
A
  1. (C) The physician-patient privilege is not available to the man under the facts presented. In order fora statement to be deemed admissible under the physician-patient privilege, the information obtained from or the communication from the patient must be necessary to enable the physician to prescribe or act for the patient. Since the man’s statement to the doctor does not relate to treatment, it is not protected by the privilege. Thus, choice (A) is incorrect, and choice (C) provides the best answer. Choice (B) is an incorrect rule of law, since the privilege is not destroyed if it is overheard by an x-ray technician. Choice (D) is also wrong because the communication here remains confidential, even though it is not privileged.
82
Q
  1. A man and a woman worked for an armored car company, picking up and delivering cash from banks and businesses. They figured out a plan to fake a holdup and robbery of the man’s armored car. After faking the holdup, they planned to purchase a boat with the proceeds and travel to a remote island to deposit the money in a bank account there. The police got wind of the plan before it could be carried out, and the man and the woman were charged with conspiracy. At trial, the prosecution wants to introduce a letter from the man to the woman before the robbery in which the man wrote:
    “Dear woman,
    I hope our plan for the heist works. If it does, we’ll have a giant pile of bills to divide between us. (signed) the man”
    Before the contents of the letter can be admitted into evidence, the prosecution must
    (A) have an expert witness identify the man’s signature as genuine.
    (B) bring the original letter into court.
    (C) demonstrate the relevance of the letter.
    (D) present evidence of the authenticity of the letter.
A
  1. (D) In accordance with FRE 901, choice (D) is the best answer, since the letter must be properly authenticated by the proponent before it will be admitted into evidence. FRE 901 (a) states that “the requirement of authentication or identification as a condition precedent to admissibility is satisfied by evidence sufficient to support a finding that the matter in question is what its proponent claims.” A letter must be authenticated, but not necessarily by an expert under the FRE. Thus, choice (A) is incorrect. Choice (B) is incorrect because the contents of the letter are not in issue. Choice (C) is a good answer; all evidence must satisfy the relevance requirements of FRE 401 to be admitted at trial. Choice (D) is a better answer, however, because even if the letter is relevant, it won’t come into evidence unless it is authenticated.
83
Q
  1. A witness testified under a state grant of immunity about statewide gambling activities being investigated by a state grand jury. Five months later, the witness was subpoenaed by a federal grand jury investigating gambling related activities.
    Which of the following statements is correct with respect to the witness’s rights before the federal grand jury?
    (A) The witness’s grant of immunity by the state would extend to all subsequent investigations.
    (B) Federal authorities have the burden of showing that they have an independent source for their evidence against the witness.
    (C) The witness’s Fifth Amendment protection against double jeopardy would extend to federal prosecution.
    (D) In order for the witness’s grant of immunity to apply in the federal investigation, it must be broader than the protection afforded by his Fifth Amendment privilege against selfincrimination.
A
  1. (B) In Murphy v. Waterfront Commission, 378 U.S. 52 (1964), once a defendant demonstrates that he has testified, under a state grant of immunity, to matters related to the federal prosecution, the federal authorities have the burden of showing that their evidence is not “tainted,” by establishing that they had an independent, legitimate source for the disputed evidence. Choice (A) is incorrect because the state does not have the authority to bind the federal government with a grant of use immunity, and choice (0) is incorrect for similar reasons. Choice (C) is incorrect because jeopardy is not triggered until a jury is impaneled and sworn or, in the case of a bench trial, the first witness is sworn.
84
Q
  1. During a federal grand jury investigation into money laundering, a man’s business and tax records were subpoenaed by the government. The man invokes his Fifth Amendment privilege against self- incrimination to prevent production of the records in his possession.
    The court would most likely
    (A) compel production of the records, because the Fifth Amendment is inapplicable.
    (B) order a preliminary hearing to determine if the man has grounds to invoke the Fifth Amendment.
    (C) permit the man to exercise his Fifth Amendment privilege, since such records may have the same incriminating effect as testimony.
    (D) admit the records under the business records exception to the hearsay rule.
A
  1. (A) In Couch v. United States, 409 U.S. 322 (1973), the U.S. Supreme Court ruled that a taxpayer may not invoke his Fifth Amendment privilege against compulsory self- incrimination to prevent the production of his business and tax records. The Court noted that “it is important to reiterate that the Fifth Amendment privilege is a personal privilege: it adheres basically to the person, not to information which may incriminate him.” Choice (B) is incorrect because the rule stated in Couch is clearly applicable to the man’s case. Choice (C) is incorrect. Business and tax records are not considered testimonial for the purposes of Fifth Amendment protection. Choice (0) is also incorrect. The FRE do not apply to grand jury proceedings.
85
Q
  1. A driver had a new car and, as he was carefully driving within the posted speed limit, a girl, aged 11, suddenly darted into the street in front of his car. Although\he driver immediately applied his brakes and swerved to avoid the girl, the cat hit the girl, fracturing her legs. As the girl was lying in the street awaiting an ambulance, the driver rushed over to her and said: “Im terribly sorry. This is the first time I ever drove this car. I don’t know what happened, but it must have been my fault. Send me all your hospital bills. I’fl pay for everything.”
    When the girl was later released from the hospital, her parents contacted the driver who refused to pay anything. The driver told the girl’s mother, “Since your daughter ran into the street, it was her fault. I have witnesses who saw what she did. If I weren’t such a nice guy, I’d sue you for the damage to my new car.”
    If the girl’s parents, on her behalf, sue the driver in tort, which of the following is the most accurate statement regarding the driver’s post-accident statements?
    (A) The driver’s statement regarding his operation of the car, as well as his statement concerning payment of the hospital bills, are both admissible on the issue of negligence.
    (B) The driver’s statement regarding payment of the hospital bills is admissible, but his statement concerning his operation of the car is not admissible.
    (C) The driver’s statement regarding his operation of the car is admissible on the issue of negligence, but his statement concerning payment of the hospital bills is not admissible in accordance with public policy considerations.
    (D) Neither the driver’s statement regarding his operation of the car nor his statement concerning payment of the hospital bills is admissible on the issue of negligence.
A
  1. (C) The driver’s statement regarding his operation of the car is admissible, but his statement concerning payment of the hospital bills is not admissible. FRE 409 excludes evidence of offers or promises to pay medical, hospital, or similar expenses occasioned by injury to prove liability for the injury. However, if the reference to the medical expenses is severable from the defendant’s admission of liability, the courts will permit such severance. Thus, choice (C) is correct, and choices (A) and (D) are incorrect. Choice (B) is wrong because it states the rule on severance exactly wrong.
86
Q
  1. Late one evening a man was driving home from his job as a security guard at a jewefry store. As the man approached a railroad crossing he noticed that the barriers to the crossing were open. The man cautiously looked both ways for any train. Assuming that no train was coming, he proceeded across the railroad tracks. Suddenly, an oncoming train struck the man’s car, completely demolishing it. The man was dead on arrival at the local hospital. Unknown to the man, the railroad crossing guard, in a drunken stupor, had fallen asleep at the barrier controls. He failed to see or hear the signals from the oncoming train that struck and killed the man. The railroad company was well aware of the crossing guard’s alcoholism, but continued to employ him. However, two weeks after the accident occurred, the railroad company fired the crossing guard.
    The executor of the man’s estate institutes a wrongful death action against the railroad company. During the trial, the attorney for the man’s estate offered into proof the fact that the defendant railroad company dismissed the crossing guard shortly after the accident.
    Upon objection by the railroad company’s attorney, the court should
    (A) overrule the objection, as the employee’s dismissal was an admission that the railroad company was aware of the crossing guard’s chronic alcoholism.
    (B) overrule the objection, as an adoptive admission by conduct.
    (C) sustain the objection, because the probative value of the evidence would be outweighed by its prejudicial effect.
    (D) sustain the objection, as evidence of subsequent remedial repairs is inadmissible.
A
  1. (D) FRE 407 states that “When, after an event, measures are taken which, if taken previously, would have made the event less likely to occur, evidence of the subsequent measures is not admissible to prove negligence or culpable conduct in connection with the event.” The evidence that the defendant railroad company dismissed the crossing guard would thus be inadmissible to prove that they were admitting negligence in connection with the accident that resulted in the man’s death. The exclusion of evidence of subsequent remedial repairs under the FRE rests specifically today on a social policy of encouraging people to take, or at least not discouraging them from taking, steps in furtherance of added safety and repairs. For these reasons, choice (D) is correct, and choices (A) and (B) are incorrect. Choice (C) is wrong for two reasons: (1) it misstates the FRE 403 standard for excluding evidence; and (2) there is no information in the fact pattern that would permit weighing the probative value and prejudicial effect of the evidence.
87
Q
  1. A plaintiff was seriously injured in a collision involving three trucks. A sideswipe occurred between the plaintiff’s pickup truck and a trailer truck as they passed in opposite directions. The sideswipe caused the plaintiff’s truck to careen down the road and into the path of a second trailer truck. The driver of the second trailer truck was immediately killed. Both of the trailer trucks were owned by the same transport company.
    At trial, the plaintiff called a witness to the collision to testify that he heard an unidentified female witness scream, “Oh my God, that trailer truck sideswiped that little pickup truck.”
    The trial judge should rule the witness’s testimony
    (A) admissible as a declaration of existing state of mind.
    (B) admissible as an excited utterance.
    (C) inadmissible, because it contains inadmissible opinion evidence.
    (D) inadmissible as hearsay not within any recognized exception.
A
  1. (B) Under FRE 803 (2), the statement of the unidentified bystander, made in the presence of the witness, should qualify as an “excited utterance” or “spontaneous exclamation” within the meaning of that exception to the hearsay rule. The theory of this exception is simply that there has been some occurrence startling enough that temporarily stills the capacity of reflection and produces utterances free of conscious fabrication. Choice (A) is wrong because the statement describes an observation made by the declarant and not the declarant’s then-existing state of mind. Choice (D) is incorrect. The statement is hearsay but, as discussed above, meets a recognized exception. Choice (C) is incorrect. The statement expresses a factual observation and not opinion testimony.
88
Q
  1. While driving her company vehicle near a pedestrian mall, a woman came upon the scene of a three-car accident. She was so busy gawking at the damaged vehicles that she failed to see one of the victims lying on the road in front of her car. She hit and ran over the victim, who survived and sued the woman’s company. The victim offers the testimony of a witness to the incident. Referring to the woman, the witness stated, “The driver of that car ran over the victim as he was lying on the ground awaiting an ambulance, and said ‘It is all my fault, I should have been paying more attention to my driving.” Assume for this question that the woman is available to testify.
    The trial judge should rule that the testimony is
    (A) admissible as a declaration against interest.
    (B) admissible as a present sense impression.
    (C) admissible as an admission.
    (D) inadmissible as hearsay not within any recognized exception.
A
  1. (C) A statement will not be barred from admission by the general rule against hearsay where the statement is offered against an opposing party and was made by the party’s agent or employee on a matter within the scope of that relationship and while it existed. Indeed, there is a substantiaL trend favoring the admission of statements relating to matters within the scope of the agency or employment. Therefore, choice (C) is correct, because the statement will be admitted as a vicarious admission. Choice (A) is wrong because a declaration against interest can only be admitted if the declarant is unavailable at trial. Choice (B) is incorrect. The woman is admitting liability and negligence, not “describing an event or explaining an event or condition” within the meaning of FRE 803(1). Finally, choice (D) is wrong because the statement is hearsay that falls within a recognized exception.
89
Q
  1. A husband and wife had been married for 10 years. The wife obtained a divorce from the husband after learnitg that he was engaging in an extramarital love affair with one of his co-workers. Following the divorce, the husband was arrested and charged with embezzling funds from his employer during a two- year period. In preparing for trial, the prosecuting attorney interviewed the wife, who stated that during their marriage, the husband admitted committing the embezzlement. This jurisdiction has adopted both the common-law marital and spousal privileges.
    At the husband’s trial, the wife is called to testify regarding statements the husband made to her during the marriage regarding commission of the crime.
    Upon objection by the husband’s attorney, the wife’s testimony will be
    (A) admissible, under both the marital and spousal privileges.
    (B) admissible, under the spousal privilege but not admitted under the marital privilege.
    (C) admissible, under the marital privilege but not admitted under the spousal privilege.
    (D) inadmissible, under either the marital or spousal privileges.
A
  1. (B) On the MBE, students are frequently tested on the distinction between the spousal and marital privileges. At common law, the spousal privilege (or incapacity) prevented one spouse from testifying against the other in a criminaL case. According to Lilly, “the trend has been to abolish the privilege entirely, or to make the witness the holder, leaving the accused without the power to object to his spouse’s voluntary adverse testimony.” Lilly further notes that “a majority of states now follow the ruLing in Trammel.” Evidence, p. 391. Therefore, underthe marital privilege, the wife may be prevented from testifying but not under the spousal privilege. ft is important to note that the marital privilege survives the termination of marriage either by death or divorce. For these reasons, choices (A) and (C) are incorrect. Choice (D) is incorrect because the evidence would be admissible under the spousal privilege.
90
Q
  1. A defendant was arrested and charged with arson, which in this jurisdiction is defined as the “malicious or intentional burning of any structure or dwelling.” When the defendant was arrested, he was inside a warehouse that was engulfed in flames. At the time he was apprehended, the police found in his possession a book of matches with four matches missing.
    At trial, the prosecution introduced the book of matches into evidence. Following closing arguments, and before the jury retired for deliberations, the judge gave instructions concerning the law of the case. The judge instructed the jury that it could infer the defendant’s intent to set the fire based on the fact that four matches were missing from the matchbook. This instruction was specifically objected to by the defense counsel.
    The judge’s jury instruction concerning the defendant’s intent was
    (A) proper, because it constituted a rebuttable presumption of fact.
    (B) proper, because it constituted a permissible inference.
    (C) improper, because the prosecution must prove each element of the crime beyond a reasonable doubt.
    (D) improper, because the instruction shifts the burden of persuasion from the prosecution to the defendant defense.
A
  1. (B) Here is another example of why the Evidence questions are among the trickiest on the exam. By now, you can probably see that the Multistate exam is not testing rote outline memorization but, rather, it homes in on subtle nuances and fine-line distinctions. In this regard, you need to differentiate between inferences and presumptions. According to LiLly, “when a genuine presumption is accepted as true by the trier of fact, it gives rise to a mandatory inference, which is properly called a presumed fact.” Evidence, pg. 56. Once the basic facts are believed, the resuLting presumed fact must be accepted by the trier of fact unless it is rebutted by contravening evidence. Consequently, in criminal cases, presumptions are disfavored and subject to close constitutional scrutiny because they tend to shift the burden of producing evidence to the accused. On the other hand, an inference never has such a compulsory effect. The trier of fact always is at liberty either to accept or reject an inferred fact. In this regard, Lilly further states that “a presumption creates a compulsory finding that remains obligatory untiL the presumed fact is rebutted.” However, this is not true of an inference, which results only in creating a jury question as to whether the inferred fact exists. As a result, choice (B) trumps choice (A) because an inference is favored over a presumption in criminal cases. Choices (C) and (D) are wrong because an inference does not shift the burden of proof away from the prosecution.
91
Q
  1. A plaintiff was suffering from a respiratory infection and his doctor prescribed for him a drug that was manufactured by a pharmaceutical company. With every prescription of this drug distributed, the company inserted a written warning, which stated:
    “Use of this drug may cause dizziness.”
    In accordance with the prescription, the plaintiff received a four-day supply of the drug from his neighborhood pharmacy, which passed along the warning to the plaintiff. After two days, the plaintiff felt much better and went back to work at a home improvement store, where he was the manager. The plaintiff continued to take the drug until his supply was exhausted. On the fourth day, the plaintiff took his final capsule at 2:00 p.m. While he was driving his car home at 5:45 that afternoon, he became dizzy and crashed into a tree. As a result, he received multiple injuries.
    The plaintiff decided to consult an attorney, who filed a lawsuit on the plaintiff’s behalf against the pharmaceutical company, alleging that the company was responsible for the accident and resulting injuries on the theories of negligence and strict liability.
    At trial, the plaintiff’s attorney attempts to introduce evidence that subsequent to this accident, the company expanded its warning to read: “Use of this drug may cause dizziness. Do not drive for five hours after use.”
    Such evidence is
    (A) admissible as an indication that the company did not use due care in wording its original warning.
    (B) admissible as a declaration against interest, to prove the possibility of alternate warnings.
    (C) inadmissible to show that the company was negligent in wording its original warning.
    (D) inadmissible, because the subsequent warning is not relevant to the feasibility of a modified warning.
A
  1. (C) The evidence of the company’s expanded warning for use of the drug is inadmissible to show that the company was negligent in wording the original warning. Evidence of the company’s revised warning creates the obvious inference that the manufacturer was negligent in failing to use the subsequent warning earlier. This type of subsequent measures (revising original warning), which is being introduced to prove negligence, is inadmissible under the FRE. FRE 407 states, “when, after an event, measures are taken which, if taken previously, wouLd have made the event less likely to occur, evidence of the subsequent measures is notadmissible to prove negligence or culpable conduct in connection with the event.” This rule does not require the exclusion of evidence of subsequent measures when offered for another purpose, such as (1) proving ownership; (2) control; or (3) feasibiLity of precautionary measures, if controverted by the defendant. Choice (A) is the wrong answer because of the policy concerns implicated by FRE 407 to encourage remedia’ measures. For similar reasons, choice (B) is also incorrect. Choice (D) is a correct outcome, but for the wrong reason. A subsequent warning would be relevant to show the feasibility of modifying the original warning. However, the policy concerns of FRE 407 trump the relevance of this evidence.
92
Q
  1. In an automobile accident case, the defendant claimed that the plaintiff was contributorily negligent by driving at an excessive speed. The defendant’s attorney calls its first witness, who was standing along the side of the road when the plaintiff’s and defendant’s cars collided. The defendant’s attorney asks the witness: “Is it a fact that in your estimation, plaintiff was traveling at a speed of 45 m.p.h. in a 35 m.p.h. speed zone at the time of the accident?” The plaintiff objects.
    The trial judge should
    (A) overrule the objection, because the judge may take judicial notice of the speed limit within the jurisdiction.
    (B) overrule the objection if the witness has already been shown to be an experienced driver who was able to estimate the approximate speed of moving vehicles.
    (C) sustain the objection if the witness has not been qualified as an expert witness.
    (D) sustain the objection, because the witness is not an adverse witness.
A
  1. (D) Choice (D) is the best answer, since leading questions in accordance with FRE 611 are generally inadmissible on the direct examination of a witness. This question requires a two-step analysis. First, you must determine if the objection should be sustained or overruled. Since the question posed by defendant’s attorney on the direct examination of the witness is leading, the trial judge should, thus, sustain plaintiff’s objection. Second, you must determine the correct reasoning for the judge’s sustaining of the objection. Choice (D) is correct, since one of the exceptions in which leading questions may be asked on direct examination is where the witness is adverse or hostile. Choice (C) is incorrect because one need not be an expert witness to testify to speed and, furthermore, the question is still objectionable as leading. Therefore, choice (D) states the correct ruling by the trial judge, as well as the best legal reasoning of the two “sustain-the-objection” alternatives. There are several exceptions to the general rule that leading questions are not permissible on the direct examination of a witness. The exceptions include the use of leading questions (a) to bring out preliminary matters, such as the name and occupation of a witness; (b) to elicit matters not substantially in dispute; (c) to question a hostile witness or an adverse party; and (d) to aid a witness in testifying because of the witness’s loss of memory, immaturity, or mental weakness. Choice (B) is incorrect; lay witnesses can testify about such things as speed estimates without having to be qualified as experienced drivers. Finally, choice (A) is wrong because the issue isn’t the speed limit itself, but how fast the plaintiff was driving.
93
Q
  1. George was charged with the murder of a well- known criminal. The victim was found shot to death in his car. The only witness to the shooting was the victim’s pet parrot, which was found hovering over the victim’s body repeating over and over, “George don’t. . . George don’t.”
    To prove the identity of the victim’s killer, the prosecution has a witness testify that about the time of the murder, he was in the area and heard the victim’s parrot calling George’s name.
    The best objection to the testimony is that it is
    (A) hearsay.
    (B) irrelevant.
    (C) opinion testimony.
    (D) self-serving.
A
  1. (B) Choice (A) is incorrect because the FRE require that the hearsay rule apply to the out-of-court statements or declarations of individuals. Since the statement in the present question was made by a parrot, the best objection would be that the testimony is irrelevant, not hearsay. Choice (C) is incorrect because the rules on opinion testimony refer to human witnesses, not animals. Choice (D) is also wrong; it is difficult to conceive of an animal testifying at trial, let alone giving self-serving
    testimony.
94
Q
  1. As part of his defense to a murder charge, a defendant offered testimony that he was committing a bank robbery in another state on the day that the victim was killed.
    The testimony is
    (A) admissible as not hearsay.
    (B) admissible as an admission.
    (C) admissible as a declaration against interest.
    (D) admissible as then-existing state of mind.
A
  1. (A) Under FRE 801(d)(2)(A), an admission is a statement offered against a party by a party opponent. Here, the statement by the defendant is not an admission because the defendant was simply testifying to his actions on the day the victim was killed. The question is tricky because the defendant is, of course, testifying that he was committing another offense. This fact could cause a test-taker to forget that under the rules, admissions must be offered by a party opponent. Because the defendant is testifying to his location and what he was doing, the statement is not hearsay. Since the statement is not hearsay, choices (B), (C), and (D) are not correct.
95
Q
  1. A man is charged with murder. During the trial, defense counsel offers the testimony of a man on death row admitting he hired another man to shoot the victim.
    The testimony is
    (A) not hearsay.
    (B) hearsay, but admissible as an admission.
    (C) hearsay, but admissible as a dying declaration.
    (D) hearsay not within any recognized exception.
A
  1. (A) Since the man is testifying, the confession would not be hearsay in that hearsay is an out-of-court statement. As a consequence, choices (B), (C), and (D) are incorrect. Furthermore, this is not an admission of the defendant or a dying declaration, in that the statement does not deal with the cause or circumstances of the declarant’s death (and there is no fear of death being impending).
96
Q
  1. A defendant was charged with murder. The defendant was a butcher by trade, and it was crucial to the prosecution’s case against the defendant to prove that the victim was stabbed to death with a butcher’s knife. The prosecution called a police officer to the stand. The assistant district attomey marked a colored photograph of the dead murder victim for identification and was ready to question the police officer. Before the prosecutor proceeded further, the defense counsel objected to any use of the victim’s photo, which pictured 14 stab wounds to his body.
    The photo is
    (A) admissible, for the limited purpose of showing that the stab wounds resulted from a butcher’s knife.
    (B) admissible, because it is relevant to the prosecution’s case, and all relevant evidence is, by its nature, admissible.
    (C) admissible, because the photo would neither mislead the jury nor waste time.
    (D) inadmissible.
A
  1. (D) Choice (D) is the best answer when compared to the alternatives. In accordance with FRE 403, the danger of unfair prejudice to the defendant would outweigh the relevance of the photo depicting the victim’s blood-spattered body. FRE 403 provides that “although relevant, evidence may be excluded if its probative value is substantially outweighed by the danger of unfair prejudice, confusion of the issues, or misleading the jury, or by considerations of undue delay, waste of time or needless presentation of cumulative evidence.” The prejudice to the defendant results from the facts that the photo was in color and the body was blood-spattered from the butcher’s knife wounds. Moreover, the prosecution can use other methods to attempt to prove that the wounds are those from a butcher’s knife (medical testimony from an autopsy report regarding the victim’s wounds). Choice (A) is wrong since, if the photo is admissible, there is no reason to believe it must be limited to that single purpose. It could also be relevant to indicate the angle of wounds, a possible struggle, etc. Choice (B) is incorrect, since all relevant evidence is not necessarily admissible. Choice (C) is incorrect because it reaches a conclusion about the effect of the evidence on the jury that is not necessarily correct under this fact pattern.
97
Q
  1. A woman has been charged with felony murder for holding up a convenience store and shooting the clerk during the course of the robbery. The prosecutor now wishes to introduce the testimony of a bystander who was in the parking lot and saw the shooting through a nearby window. The bystander proposes to testify that he heard a customer in the store scream out, “Knock that gun away! Don’t let her do it again!”
    This evidence is admissible
    (A) as non-hearsay.
    (B) if the bystander can be shown to have been a co-conspirator.
    (C) as an excited utterance.
    (D) as a statement made under belief of impending death.
A
  1. (C) Under the excited utterance exception to the hearsay rule, the bystander’s testimony about the customer’s statements would be admissible. FRE 803 (2) provides that an “excited utterance” is not excluded by the hearsay rule, even though the declarant is available as a witness. An excited utterance is a statement relating to a startling event or condition made while the declarant was under the stress of excitement caused by the event or condition. The customer’s statements clearly relate to the startling event of the shooting that he was observing. Choice (A) is wrong, since the testimony is hearsay. Choice (B) is wrong, since the co-conspirator’s admission rule, FRE 801 (d)(2)(E), is incorrectly interpreted here. Choice (D) is not correct, since the unavailability requirement has not been met.
98
Q
  1. A recent law school graduate applied for a legal position with a law firm. A senior partner in the law firm contacted a professor at the law school seeking information about the graduate’s qualifications. The professor replied, “Don’t be misled by that ‘A’ I gave the graduate in entertainment law. I later found out that he had obtained the grade by cheating.”
    The graduate soon received a letter from the law firm notifying him that he was not suitable to become a member in the prestigious tradition of the firm. After the graduate received the rejection letter, he phoned the senior partner to inquire why he was not suitable for employment. The senior partner told him what the professor had said and then stated, “We do not hire cheaters who are dumb enough to get caught.”
    The graduate brings suit against the professor for defamation for falsely charging the graduate with being a cheater.
    If the professor pleads truth as a defense, he should not be permitted to introduce evidence that shows that
    (A) the graduate did, in fact, cheat on an examination in the entertainment law class.
    (B) another professor told him that the graduate had cheated on an admiralty law final examination.
    (C) the findings of the law school indicated that the graduate had submitted a plagiarized paper in a moot court brief.
    (D) the reputation among the graduate’s fellow classmates was that he frequently cheated on law school examinations.
A
  1. (B) Character evidence is generally not admissible in civil actions unless character is “in issue” (i.e., where it is a material element of a charge, claim, or defense). In the following tort actions, character is said to be “in issue” and, thus, character evidence is admissible: (1) defamation, (2) deceit, (3) negligent entrustment, and (4) assault/battery where the defendant claims that he acted in self-defense, in which case the plaintiff’s reputation for violence or turbulence is in issue. Since this is a defamation action, answer choices (A), (C), and (D) are admissible character evidence. Choice (B) is inadmissible as hearsay because it is an extra-judicial (or out- of-court) statement, which is being offered to prove the truth of the matter asserted.
99
Q
  1. A defendant was arrested and charged with burglarizing a home. At trial, the main issue of contention centered on the identity of the burglar. The state called the homeowner to testify, but his identification testimony was inconclusive. The homeowner testified that he was asleep during the burglary and awoke as the burglar was fleeing the premises. The homeowner admitted that he didn’t see the face of the burglar and couldn’t even tell whether the person was male or female. Thereafter, the prosecution offers evidence that at the time of the burglary, the defendant needed money to defend himself against other burglary charges.
    This proffered evidence should be
    (A) admitted without restriction as probative evidence of motive.
    (B) admitted, provided the court instructs the jury to limit use of the evidence to the defendant’s motive to burglarize.
    (C) excluded as lacking any probative value. (D) excluded as substantially more unfairly prejudicial than probative.
A
  1. (D) FRE 403 states that “Although relevant, evidence may be excluded if its probative value is substantially outweighed by the danger of unfair prejudice, confusion of the issues, or misleading the jury, or by consideration of undue delay, waste of time, or needless presentation of cumulative evidence.” The policy behind the often utilized balancing standard is best viewed by pitting the maximum probative value of evidence against the likely prejudicial effect (i.e., in close cases, the decision should be to let in the evidence). This question is clearly not a close case. The prosecutor’s proffered evidence as to the defendant’s motive for the burgLary (that he needed the money to defend against other burglary charges) is more prejudicial than probative because the defendant’s identity is in issue, not his motive. Therefore, choice (D) is correct, and choice (A) is incorrect. FRE 404(b) was also being tested, but it did not apply. Specifically, Rule 404(b) excludes admissibility of other crimes, wrongs, or acts as to character evidence to prove conduct in conformity therewith; however, such crimes may “be admissible for other purposes, such as proof of motive, opportunity, intent, preparation, plan, knowledge, identity, or absence of mistake or accident.” Again, evidence of the defendant’s motive to commit burglary should be excluded, since its prejudicial effect outweighs its probative value. Thus, choice (B) is incorrect. Finally, choice (C) is incorrect because the evidence does have probative value; it’s just that the danger of unfair prejudice substantially outweighs its probative value.
100
Q
  1. A man was charged with tax fraud. He wished to offer evidence, on his behalf, of his honesty while running for mayor. During his political campaign, he refused to lie about a psychiatric problem in his past, and his honesty cost him the election. The man was quoted as saying, “If I lied, I’d be mayor today, but that’s no way to win an election.”
    The BEST argument of the objector to such evidence in the tax fraud case would be which of the following?
    (A) Character evidence is not admissible in such cases.
    (B) One incident doesn’t prove character.
    (C) This is an impermissible way to prove character.
    (D) The evidence is irrelevant.
A
  1. (C) Since character evidence is admissible in a criminal case (when the defendant “opens the door” and places her character in issue), choice (A) is incorrect. Choice (B) is not the best argument, since it applies to evidence of habit rather than character. Choice (C) is the best choice because specific acts are not admissible in a criminal trial for the purpose of proving character. Choice (D) is incorrect because the standard for relevance under FRE 401 is relativeLy low: evidence having any tendency to make the existence of a determinative fact more or less probable than it would be without the evidence.
101
Q
  1. A salesman, who had worked 20 years for the same company, was suddenly terminated for no apparent reason. Thereafter, the salesman sued the company, alleging age discrimination. At trial, he wants to call an employee of the company as an adverse witness. The salesman seeks to have the employee testify that she was present at a company board meeting when the company’s president allegedly said, “Now, I’m sure that everyone agrees that the salesman is too old, and he really doesn’t typify the image we want our employees to project.” It is the common practice of the company to tape record all such board meetings. Moreover, it is customary for the company’s secretary to transcribe the tapes following the board meetings.
    Upon objection by the company’s attomey, the employee’s proposed testimony will be held
    (A) admissible, because the employee was present during the board meeting.
    (B) admissible, because the president’s statement was an admission by a company representative.
    (C) inadmissible, because the tape of the meeting is the best evidence.
    (D) inadmissible, because the secretary’s transcribed notes are the best evidence.
A
  1. (B) FRE 801 (d)(2)(D) allows a statement by a party’s agent or servant concerning a matter within the scope of the agency or employment, made during the existence of the relationship, to be admissible against the party as a vicarious admission. In O’Donnell v. Georgia Osteopathic Hospital, 748 F.2d 1543 (1984), the plaintiffs claim based upon age discrimination was upheld where the Court admitted (as a vicarious admission) the testimony of the person who repLaced the plaintiff describing discussions he had had with the hospital’s executive director concerning the plaintiffs treatment. The statements made by the company’s president concerning the salesman’s age not typifying the company’s image were made within the scope of his empLoyment and during the existence of the reLationship. As such, his testimony will be admissible as a vicarious admission. Choice (B) is correct. Choice (A) is incorrect because the mere fact that the employee was present at the board meeting and had personal knowledge is insufficient in itself to admit an out- of-court statement which that otherwise be hearsay. Allowing the statement as an admission however, under choice (B), addresses this issue, since admissions, by definition, are non-hearsay. Choices (C) and (D) are incorrect. The best evidence rule is not relevant under these facts. The employee’s testimony of what the president said is based on personal knowledge and is not reliant on any tape recording or transcribed notes.
102
Q
  1. A customer sued a printing company for negligence, claiming that the holiday cards he specially ordered were of poor quality and had his name misspelled. At trial, the customer shows the court a printed statement from a printer’s association defining the minimum standard of skill for the printing industry in the community. The customer asks the court to take judicial notice that the printed statement constitutes the applicable standard of care for printing companies in the area.
    The court should
    (A) grant the request, because it promotes expeditious and economical disposition of the proceedings.
    (B) grant the request, because the standard of care is amply supported by information supplied to the court.
    (C) deny the request, because judicial notice cannot be taken of matters that are ultimate issues in the case.
    (D) deny the request if the judge determines that the statement is subject to reasonable dispute.
A
  1. (D) A judicially noticed fact must be one not subject to reasonable dispute in that it is either (1) generally known within the territorial jurisdiction of the trial court or (2) capable of accurate and ready determination by resort to sources whose accuracy cannot reasonably be questioned. FRE 201. Choice (D) is the best choice because, according to the rule, a judge will not take judicial notice of a fact over which there is a reasonable dispute. Choice (C) is incorrect because it is too conclusory. While courts are hesitant to take judicial notice over matters that are the ultimate issues in a case, that is not to say that courts will always refrain from doing so. Choice (A) is wrong. If the statement in (A) were correct, a judge could take judicial notice of nearly every disputed issue at trial in order to save time and promote efficiency. Choice (B) is also incorrect. There simply isn’t enough information about this document to determine whether it amply supports or defines the standard of care for printers.
103
Q
  1. A grocery store is sued by a customer for damages for injuries allegedly suffered to the face and forearms, which resulted from his collision with the store’s automatic doors. The customer contended that at the time of the mishap, the doors were programmed to swing outward, taking would-be customers entering the store, by surprise. At trial, the customer testified to both the nature of his injuries and the events leading up to their occurrence. The grocery store denied that the doors to its store were in any way unsafe or that the customer’s injuries were caused by his encounter with the doors. The customer’s sister testified at trial that she accompanied her brother to the store the day of the accident and witnessed the outward swinging doors, which struck him. She further alleged that when she returned to the store the following week, the doors had been re-programmed to swing inward.
    The trial judge should rule the sister’s testimony
    (A) admissible as a commonsense impression for which a lay opinion is entirely proper.
    (B) admissible as a tacit admission of a party opponent.
    (C) inadmissible, on the grounds of irrelevancy.
    (D) inadmissible, on the grounds of public policy considerations.
A
  1. (D) Evidence of subsequent remedial measures is generally excluded on grounds that the possible relevancy of the evidence is outweighed by public policy considerations (e.g., encouraging repairs after accidents). Therefore, choices (A), (B), and (C) are incorrect.
104
Q
  1. A man is suing a store for damages for injuries allegedly suffered when a can of soup fell on his toe while he was shopping. At trial, the store calls a member of the local community to testif’ that the man’s reputation for honesty is poor, and that he is known far and wide as the biggest storyteller in
    town.
    This testimony is
    (A) admissible, but only to show the likely exaggeration of the man’s alleged injuries.
    (B) admissible, but only to discredit the man’s testimony that the accident happened in the manner in which he claims it did.
    (C) inadmissible character evidence.
    (D) inadmissible, because the man’s testimony has not yet been rebutted by the store, and his credibility is, therefore, not yet susceptible to attack.
A
  1. (B) Note that the witness’s testimony (regarding the man’s reputation) is inadmissible character evidence. As a general rule, character evidence is not admissible in a civiL case unless defendant’s character is “in issue” (e.g., in defamation or negligent entrustment cases). Thus, choice (C) does state a correct rule of law. However, choice (B) is a better answer because even though the witness’s testimony is not admissible as character evidence, it is admissible for purposes of impeachment. According to FRE 608, for impeachment purposes opinion and reputation evidence of character (subject to the limitation that such evidence may refer only to character for truthfulness or untruthfulness) are admissible. In this case, testimony about the man’s character for truthfulness is properly admissible to impeach him. Choice (A) is wrong because it lists an incorrect purpose for the admission of the evidence. Choice (D) is also wrong. Once a witness in a case testifies, his or her credibility is at issue. There is no requirement to first rebut the witness’s testimony in order to attack his credibility.
105
Q
  1. A plaintiff alleges in a lawsuit against a store that he purchased a hot dog with chili that gave him food poisoning and caused him excruciating pain. The store offers into evidence certified copies of court proceedings revealing that the plaintiff has filed three similar lawsuits in the last 18 months against other convenience stores, claiming food poisoning resulting from the purchase of food from those stores.
    These copies should be ruled
    (A) inadmissible, because of the best evidence rule.
    (B) inadmissible, because they are irrelevant to the present claim.
    (C) inadmissible, because there is the danger of undue prejudice to the plaintiff, which outweighs the probative value of the evidence.
    (D) admissible, because they establish a pattern of similar actions.
A
  1. (C) Since evidence of prior tort claims is generally inadmissible because of the chance for prejudice and conclusion, choice (C) is correct. Choice (A) is incorrect because certified copies may be admitted in Lieu of original documents. Choice (B) is wrong because the evidence may have some probative value. Choice (0) is incorrect because the probative value is outweighed by the prejudicial effect of the evidence.
106
Q
  1. While shopping at a mall, a woman suffered injuries when an escalator malfunctioned, crushing her foot. The woman brings suit against the mall. At trial, she calls another shopper to testif’ that he rode the same escalator just moments before the woman’s accident and had to jump in the air at the last minute to avoid getting his foot crushed in the escalator.
    This testimony is
    (A) inadmissible, because it is irrelevant.
    (B) inadmissible, because it is not known whether the other shopper the woman was exercising reasonable care in riding the escalator.
    (C) admissible, because it tends to prove that a dangerous condition was present at the time the woman’s foot was crushed in the escalator.
    (D) admissible, because of its probative value in establishing that the woman’s injury was caused by the malfunctioning escalator.
A

io6. (C) The testimony is admissible to prove the existence of the condition that the woman claimed caused her injuries. When there appears to be sufficient proof of similarity in conditions and proximity in time to the accident, such evidence is admissible. Choice (A) is wrong, since the information is, indeed, relevant. Choice (D) is an incorrect statement, since the evidence is only admissible to prove the existence of the condition, but not that the accident happened. Choice (B) is incorrect because it establishes an unnecessary condition precedent for the use of this evidence.

107
Q
  1. A witness is scheduled to testify in a criminal case. To prove that he was a juvenile on a given date, evidence is offered that on that date he was confined in a juvenile detention facility.
    If a party wished to argue that this evidence is hearsay, whom would he point to as the hearsay declarant?
    (A) The witness on the stand.
    (B) The party offering the evidence.
    (C) The juvenile authorities.
    (D) The witness.
A
  1. (C) In this question, the facts tell you that someone will argue that this evidence is hearsay. In every hearsay statement, the out-of-court statement is made by the declarant. Here, the statement wouLd be that the witness was confined in a juvenile detention facility. That statement would have been made originaLLy by the juvenile authorities. So choice (C) is the best choice. Choice (A) is not as good because it is not necessarily true. The witness on the stand might have had nothing to do with the witness’s juvenile records. Choice (B) is wrong. The party offering hearsay evidence is rarely the declarant. Choice (D) is incorrect because the witness would not have been responsible for the creation and maintenance of his juvenile records.
108
Q
  1. During a custody dispute, a court granted the request of the defendant and appointed a clinical pathologist to conduct the blood grouping tests of the child, the complainant, and the defendant. After first stating all of his qualifications at the non-jury trial, the pathologist testified that he and his associates made five separate blood grouping tests and that all proper safeguards were taken to protect the integrity and accuracy of the tests. The pathologist also offered more detailed testimony about the testing procedures and stated that no discrepancies were found in the testing methods. Finally, the pathologist made the statement that the blood grouping tests were conclusive in proving that the defendant could not be the father of the child.
    If the complainant’s attorney makes an objection to the introduction of the blood tests into evidence, the court will most likely
    (A) sustain the objection, because the blood tests are not conclusive evidence of paternity.
    (B) sustain the objection, because blood tests have not been generally recognized by the scientific community.
    (C) overrule the objection, because the court will take judicial notice of the accuracy of such tests.
    (D) overrule the objection, because the blood tests are an admission.
A
  1. (C) The court is warranted in taking judicial notice of the correctness of such blood tests excluding paternity where it is shown that the person making the tests is qualified and that no discrepancies were found in the testing method. See FRE 201. Choice (A) states an incorrect standard. Evidence does not have to provide conclusive proof of the matter at issue to be admissible. Choice (B) is wrong because blood tests are, in fact, generally recognized as accurate by the scientific community. Choice (D) is incorrect because a blood test is not an admission within the meaning of FRE 801(d).
109
Q
  1. An accounting professor has been called as a defense expert witness in a white-collar criminal case. He testified that accounting records conclusively demonstrated that the defendant had not committed any type of fraud. On cross-examination of the professor, the prosecuting attorney asked him if he was familiar with a particular treatise. The professor responded affirmatively. The attorney then asked him if the treatise was accepted as authoritative and if he was aware of the conclusions regarding the ability of accountants to detect fraud. The professor responded in the affirmative. The prosecutor now attempts to read the following section of the treatise into evidence: “It is virtually impossible to state with conclusive certainty the existence, or lack thereof, of fraud in the majority of white-collar crime cases, given the sophistication of defendants and the variety of accounting methods available.”
    On objection by the defendant’s attorney, the trial judge will
    (A) overrule the objection and permit the entire treatise to be introduced into evidence.
    (B) overrule the objection and permit only the contradictory statements to be read into evidence.
    (C) sustain the objection, because the statements in the treatise are hearsay not within any recognized exception.
    (D) sustain the objection, because the professor did not rely on the treatise in making his conclusions.
A
  1. (B) Under FRE 803 (18), the following are not excluded by the hearsay rule “… to the extent called to the attention of an expert witness upon cross-examination or relied upon by him in direct examination statements contained in published treatises, periodicals, or pamphlets on a subject of history, medicine, or other science or art, established as a reliable authority by the testimony or admission of the witness or by other expert testimony or by judicial notice. If admitted, the statements may be read into evidence but may not be received as exhibits.” Choice (A) is wrong because only the entire treatise itself cannot be placed into evidence. Choice (C) is incorrect. The statements in the treatise are hearsay, but fall within a recognized hearsay exception. Choice (D) is also wrong. One purpose of this hearsay exception is to test the knowledge of expert witnesses on cross-examination and provide contradictory testimony to the fact-finder.
110
Q
  1. During a paternity trial, the mother’s attorney asks to have the baby girl, now 36 months old, exhibited for the purpose of calling attention to certain facial resemblances between the child and the reputed father. The defendant’s attorney objects to the offering of the baby into evidence for comparison.
    The trial judge should
    (A) permit the child to be exhibited as a non-objectionable form of demonstrative evidence.
    (B) not permit the child to be exhibited, because such an exhibition would be highly prejudicial to the defendant.
    (C) permit the child to be exhibited as within the hearsay exception of pedigree.
    (D) not permit the child to be exhibited, because such an exhibition would be self-serving to the complainant.
A
  1. (A) The majority of states hold that a trial court may, in its discretion, permit a child to be exhibited (e.g. demonstrative evidence) in a proceeding for the purpose of showing a resemblance to the putative father. Judway v. Kovacs, 239 A.2d 556 (1967). Accordingly, choice (A) is correct, and choice (B) is incorrect. Choice (C) is incorrect. The hearsay exception for pedigree requires an out-of-court statement. The live exhibition of a child at trial cannot constitute an out-of-court statement by any stretch of the imagination. Choice (D) is wrong. Much of the evidence admitted in an adversarial trial is self-serving to the offering party.
111
Q
  1. A man was driving his car when he suddenly hit a pedestrian as she was crossing the street. Following the accident, an ambulance was summoned and the pedestrian was taken to a nearby hospital. The man, who also drove to the hospital, approached the pedestrian in the emergency room while she was being treated for her injuries. When the pedestrian saw the man, she told him, “You went through that red light. You could have killed me.” She then asked the man for $5,000 to cover her anticipated medical expenses. The man replied that he would have to think about it. Shortly thereafter, the man sent a letter to the pedestrian in which he wrote, “I realize I drove through the red light. Naturally, I would like to settle this without any litigation. I’m willing to pay all your medical expenses and give you $2,000 if you don’t sue me.” The pedestrian did not accept the man’s offer and brought suit against him.
    At trial, the pedestrian’s attorney offers the first sentence of the letter into evidence, and the man objects. The trial judge should rule the evidence
    (A) admissible as an admission.
    (B) inadmissible as an admission made in connection with an offer to pay medical expenses.
    (C) inadmissible as an admission made in connection with settlement negotiations.
    (D) inadmissible, because documents must be offered in their entirety.
A

iii. (C) In accordance with FRE 408, evidence of (1) furnishing or offering or promising to
furnish, or (2) accepting or offering or promising to accept, a valuable consider
ation in compromising or attempting to compromise a claim that was disputed as
to either validity or amount, it is not admissible to prove liability for the claim or
the amount. Two grounds for the rule of inadmissibility may be advanced: (1) lack
of relevancy, and (2) the public policy aspect is to promote the settling of disputes,
which would be discouraged if offers of compromise were admitted in evidence.
Choices (A) and (B) are incorrect because of the public policy considerations of FRE 408. Choice (D) is incorrect. There is no requirement for a proponent to offer documents in their entirety. The Rule of Completeness in FRE 106 does, however, permit an adverse party to demand the contemporaneous admission of any part of a recording or writing that ought to, in fairness, be considered, along with the portion offered by the proponent of the evidence.

112
Q
  1. In a suit between a plaintiff and a defendant arising out of an automobile accident, a bystander was
    called to the stand to testify that the defendant had driven through a red traffic light immediately before the collision. On cross-examination by the defense, the bystander admitted to having made a prior statement to a friend that he was not sure if the traffic signal was yellow or red when the defendant drove through the intersection.
    The trial judge should
    (A) permit the jury to consider the prior statement as substantive evidence.
    (B) permit the defendant to call other witnesses to confirm the fact that the prior statement had been made.
    (C) on request by the defendant, instruct the jury that the bystander’s testimony that the defendant had driven through the red light be
    disregarded.
    (D) on request by the plaintiff, instruct the jury that the prior statement may be used only to
    impeach the bystander’s credibility, and not as substantive evidence.
A
  1. (D) This is an example of another key Multistate testing area, namely, whether a prior
    inconsistent statement should be admissible as substantive evidence or for the
    limited purpose of impeaching the witness. Under the Multistate viewpoint, these
    prior statements will often be inadmissible as evidence of what they state because
    they constitute hearsay and are not within any exceptions to the hearsay rule. Even
    though inadmissible hearsay as evidence of the facts stated, they are nevertheless
    admissible for the limited purpose of impeaching the witness. See McCormick on
    Evidence, pp. 66—67, 601—602. Consequently, choice (D) is correct. Choice (A) is
    incorrect. The evidence can be considered for its impeaching value, but not sub
    stantively. Choice (B) is incorrect. If a witness denies making an inconsistent state
    ment, the opposing party can offer extrinsic evidence of it under FRE 613. Since
    this witness admitted the inconsistency, there is no need for extrinsic evidence of
    it. Choice (C) is wrong because it is inconsistent with the purposes of impeachment
    by prior inconsistent statements. Impeaching statements are not admitted to prove
    their truth, but rather to demonstrate that the witness is inconsistent.
113
Q
  1. A defendant is charged with murder and relies upon a claim of self-defense. At trial, the defendant attempts to show that prior to the killing, he was told by a drinking buddy that “The victim has killed five men and would as soon kill a man as look at him.”
    If the statement by the drinking buddy is offered into evidence to prove that the victim, in fact, killed the five men, the statement should be ruled
    (A) admissible, because the statement is offered to demonstrate its effect on the defendant.
    (B) admissible, because it qualifies under the state of mind exception to the hearsay rule.
    (C) inadmissible, because it is hearsay not within any recognized exception.
    (D) inadmissible, because it is self-serving.
A
  1. (C) Choice (C) is correct since, under FRE 801, hearsay is any extra-judicial statement
    offered in evidence to prove the truth of the matter stated. Choice (A) is incorrect
    because the fact pattern tells us that the statement is being offered to prove that
    the victim killed five men. It is not being offered to prove its effect on the defendant.
    Choice (B) is wrong because the statement does not satisfy the foundational elements of FRE 803(3): it is not a statement of the declarant’s then-existing physical or mental condition. Finally, choice (D) is incorrect. “Self-serving” is not a valid evidentiary objection.
114
Q
  1. A fan attended a minor league hockey game in his hometown. Unfortunately, he was only able to obtain tickets in the visitor’s section. While at the game, he became involved in an altercation with a fan of the visiting team. When the fan cheered for a home team goal, the visiting fan turned around and threatened to kill the home fan if he didn’t shut up. The home fan pulled a knife out of his pocket and stabbed the visiting fan in the arm. At his trial for aggravated assault, the home fan wants to introduce a statement from a witness who was standing next to the visiting fan at the game. The statement, which the witness had made earlier in the game when the home fan cheered for the home team, was, “You’d better watch out. At a hockey game last week, the visiting fan put two guys in the hospital when they wouldn’t shut up. One of them had 33 stitches after the visiting fan bashed his head against the steps.”
    Assume that the witness’s statement is offered as proof of the effect it produced in the home fan’s mind. In this regard, the statement would most likely be
    (A) admissible as non-hearsay.
    (B) admissible as a present sense impression.
    (C) inadmissible as hearsay not within any recognized exception.
    (D) inadmissible, because the statement is selfserving.
A
  1. (A) Choice (A) is correct, since the witness’s statement that the visiting fan had put
    two men in the hospital at a hockey game the week before is being offered to show
    its effect on the defendant’s state of mind and not offered to prove the truth of
    the assertion. Thus, the witness’s statement bears upon the reasonableness of the
    home fan’s fear to justify his self-defense plea. Choice (B) is wrong. The statement
    does not satisfy the foundational elements of FRE 803(3): it is not a statement of
    the declarant’s then-existing physical or mental condition. Choice (C) is incorrect
    because the statement is not being offered as hearsay. FinaLly, choice (D) is incor
    rect. “Self-serving” is not a valid evidentiary objection.
115
Q
  1. A customer purchased a mattress from a furniture store. The mattress had a defective spring, which one of the manufacturer’s workers had not properly tied down. When the customer slept on the mattress, she was wounded in the back by the sharp point of the spring. The customer sued the furniture store for negligence and breach of warranty.
    At trial, the customer testified that the salesman assured her that the furniture store inspected each mattress before delivery. Furthermore, the customer testified that she remembered the salesman telling her that “The furniture store stands behind all of its mattresses with a lifetime guarantee.” She is then asked if the salesman made any other representations about the mattress. The customer stated, “Yes, in fact the day before the mattress was delivered, I received a letter from him thanking me for my patronage at the furniture store. As I recall, he also made some assurances about the manufacturer’s mattresses.” The customer’s attorney then asked, “What assurances were made in the letter?”
    The customer’s answer is
    (A) admissible as an admission.
    (B) admissible as a declaration against interest.
    (C) inadmissible, under the best evidence rule.
    (D) inadmissible as hearsay not within any recognized exception.
A
  1. (C) This is a classic Multistate example where the “best evidence” ruLe is the correct
    answer. The best evidence (or, commonly, the original document) rule provides that
    in proving the terms of a writing, where the terms are material, the original writing
    must be produced unless it is shown to be unavailable for some reason other than
    the serious fault of the proponent. Here, since the customer is attempting to testify
    as to the assurances (or representations) made in the salesman’s letter to support
    her breach of warranty action, the writing itself would be the “best evidence” for
    proving the terms contained therein. Choice (A) is incorrect because there is no admission involved in the contents of the letter. Choice (B) is aLso wrong. Even if the salesman is unavailable to testify, this is not a statement against interest, but rather a thank-you letter and, possibly, a warranty declaration. Choice (D) is also incorrect. The answer to the question would not be offered for the truth of the matter asserted, but rather to establish what was said.
116
Q
  1. A man suffered frostbite when he spilled a soft drink on his lap that he had purchased from a restaurant. The man sued the restaurant for negligence and damages. At trial, the man’s attorney calls a witness to testify that the witness also suffered frostbite when she spilled a soft drink in her lap that she had purchased at the restaurant two weeks earlier. The witness also states that she told the restaurant about her injury immediately after suffering it.
    Upon objection by the restaurant’s attorney, the trial judge should rule the witness’s testimony
    (A) admissible to prove the restaurant’s negligence, provided that the court gives a cautionary instruction that the testimony should not be considered as bearing on the issue of damages.
    (B) admissible to prove that the restaurant should have been aware of the danger of frostbite posed by its soft drinks.
    (C) inadmissible, because it is not probative of a fact in issue.
    (D) inadmissible, because it seeks to put into evidence separate, unrelated transactions with a third party.
A

ii6. (B) One of the most commonly tested areas in Evidence comes under relevancy, where
evidence is offered to prove the existence of a particular physical condition, situa
tion, or defect. In a suit alleging fraud or breach of warranty, for example, the pLain
tiff must prove that the defendant knew of the defective product, or ought, in the
exercise of reasonable care, to have learned of it. Here, the plaintiff will want to
prove directly that the defendant had knowledge of other accidents, injuries, or
complaints as circumstantial evidence that the defendant was aware of the defect.
Choice (A) is incorrect because the evidence cannot prove negligence by itself. Choice (C) is incorrect; as explained above, the evidence is probative. Choice CD) is an incorrect statement of the law of evidence.

117
Q
  1. In a breach of contract action brought by a supplier against a grocery store for refusing to buy his artisanal bread and goat cheese, the supplier calls his ex-wife to testify about the business the supplier operated and the financial effect of the breach on his business and personal finances. On cross-examination she admits that she is the supplier’s former spouse. Thereafter, on re-direct, the supplier’s attorney seeks to have the ex-wife testify that she and the supplier have not spoken to each other since their bitter divorce proceeding three
    years ago.
    The ex-wife’s testimony is
    (A) admissible under the family history exception to the hearsay rule.
    (B) admissible, because the ex-wife’s answer might rebut the inference of bias.
    (C) inadmissible, because it relates to a collateral matter.
    (D) inadmissible, because it is irrelevant to any substantive issue in the case.
A
  1. (B) Certainly, the Law recognizes the slanting effect upon human testimony of the emo
    tions or feelings of the witness toward the parties or the self-interest of the witness
    in the outcome of the case. Partiality, or any acts, relationships, or motives reason
    ably likely to produce it, may be proved to impeach credibility. In most states the
    impeacher may inquire as to the facts of bias on cross-examination as the first step
    in impeachment. By the same token, the reply to bias (or any new matter) drawn
    out on cross-examination is the normal function of the re-direct, and examination
    for this purpose is a matter of right, though its extent is subject to the control of the
    judge’s discretion. Choice (A) is incorrect. The purpose of the re-direct is to attack
    the charge that the ex-wife’s testimony is biased, not to establish the fact of her
    former marriage. Choice (C) is wrong because witness motive and bias are never a
    collateral matter at trial. For similar reasons, choice (D) is also incorrect; bias and
    motive are always relevant.
118
Q
  1. A plaintiff sued a defendant for injuries that the plaintiff suffered when he was struck by the defendant’s car. At trial, the plaintiff testified that he was walking across the street inside a crosswalk at the time the defendant’s car hit him. This action takes place in a jurisdiction that has a statute in effect requiring all motorists to stop and permit pedestrians to cross streets at designated crosswalks.
    The defendant contended that the plaintiff ran into the street outside the crosswalk. Furthermore, the defendant testified that he immediately applied his brakes when he saw the plaintiff dart into the street, but couldn’t stop his car in time to avoid hitting him. In support of the defendant’s testimony, the defendant calls a neighbor to testify that she has observed the plaintiff frequently during the years that she has lived on that street, and that the plaintiff “always crosses the street outside the crosswalk.”
    The neighbor’s testimony is likely to be
    (A) admitted, because it tends to show that the plaintiff was careless.
    (B) admitted, because it is probative of the plaintiff’s habit of crossing the street outside the crosswalk.
    (C) excluded, because the neighbor was not an eyewitness to the accident.
    (D) excluded, because her testimony is impermissible evidence of character to show defendant’s plaintiff’s conduct on a particular occasion.
A
  1. (B) According to FRE 406, “Evidence of the habit of a person or of the routine practice
    of an organization, whether corroborated or not and regardless of the presence of
    eyewitnesses, is relevant to prove that the conduct of the person or organization on
    a particular occasion was in conformity with the habit or routine practice.” Words
    such as “aLways” or “invariably” are necessary to prove habit. Choice (A) is true,
    but it is a weaker answer than choice (B) because habit under FRE 406 is more spe
    cific to the fact pattern. Choice (C) is wrong because the neighbor is not testifying
    about the accident itself, but rather to the pLaintiff’s habits. Choice (D) is incorrect.
    Although one can argue that habit evidence is a form of character evidence, its use
    is permissible under FRE 406 if the foundational elements are met.
119
Q
  1. A husband and wife had a stormy relationship. The husband, who was an alcoholic, frequently beat his wife when he became drunk. One day, the husband had been drinking beer while watching football on television. After his favorite team lost a close game, the husband went into a violent rage. As he was storming around the apartment, he suddenly grabbed his wife and led her outside to their car. He then ordered her to get inside the vehicle.
    The husband was driving around aimlessly when he negligently collided with another car. The other driver, who was injured in the accident, brought suit against the husband to recover damages. Thereafter, the husband retained an attorney to represent him. At his first consultation with the attorney, the husband was accompanied by his wife. During the conference, the attorney’s secretary took notes of the meeting. Two weeks later, the wife separated from the husband. She then hired a lawyer to represent her in a suit against the husband for battery and false imprisonment. At the trial between the husband and the other driver, the other driver’s attorney calls the wife to testify. She proposes to testify that her husband was intoxicated at the time of the accident.
    Upon objection by the husband’s attorney, the wife’s proposed testimony will most likely be
    (A) admissible as a proper lay opinion. (B) inadmissible, under the attorney-client
    privilege.
    (C) inadmissible, under the marital privilege. (D) inadmissible, because the wife is biased.
A
  1. (C) At common law, under the marital privilege, a husband or wife shall not be required
    or, without consent of the other, if living, allowed to disclose a confidential communication made by one to the other during marriage. A majority of courts have construed “communications” to extend to facts, conditions, and transactions, as well. McCormick even points out that the marital privilege has been said to apply to “any information secured by the wife (or husband) as a result of the marital relation and which would not have been known in the absence of such relation.” Accordingly, “information secured by one spouse through observation during the marriage as to the health, or intoxication, habitual or at a particular time, or the mental condition of the other spouse would be protected by the privilege.” Evidence, pp. 163—164. Note: at the time of trial, even though they are separated, the husband and wife are still married, so the privilege still applies. Choice (A) is wrong. Although the wife’s observation about the husband’s intoxication is likely lay opinion testimony, the marital privilege will prohibit her from sharing her observation and opinion at trial. Choice (B) is incorrect. There is no indication that the information the wife wants to share occurred during an attorney-client consultation. Finally, choice (D) is incorrect. Biased witnesses frequently testify at trial, and it is the job of the advocate in the adversarial system to bring the bias to the attention of the fact-finder.
120
Q
  1. A husband visits an attorney seeking a divorce because of his wife’s infidelity. At the client consultation, the attorney’s secretary took notes. With the permission bf the husband, the attorney’s law clerk also sat in on the consultation to see how the attorney conducted these meetings. Shortly after this initial consultation with the attorney, the secretary quit. During the divorce trial, the wife’s attorney seeks to call the secretary to testify about what the husband told the attorney during the initial client consultation.
    Upon objection, the secretary’s proposed testimony will most likely be
    (A) admissible, because her presence during the meeting destroyed the attorney-client privilege.
    (B) admissible, because the law clerk’s presence during the meeting destroyed the attorney- client privilege.
    (C) inadmissible, because the attorney-client privilege disqualifies the secretary from testifying to such confidential communications.
    (D) inadmissible, because it is hearsay not within any recognized exception.
A
  1. (C) It is the essence of the attorney-client privilege that it is limited to those communications that the client either expressly made confidential or which she could reasonably assume, under the circumstances, would be understood by the attorney as so intended. In cases where the client has one of his agents attend the conference, or the lawyer calls in his clerk or confidential secretary, the presence of these intermediaries will be assumed not to militate against the confidential nature of the consultation. McCormick, pg. 189. Thus, choices (A) and (B) are incorrect answers. Choice (D) is a wrong answer because the fact pattern tells us nothing about the nature of the statements made during the consultation.
121
Q
  1. A husband and his wife are involved in a contested divorce and child custody battle. Ignorant of the adversarial system, they both visited a family law attorney together for an initial consultation. The attorney advised them that he could not represent them both. The wife found another attorney. During the trial, the wife’s attorney calls the wife to testify. She states that during the initial consultation she and her husband had with his attorney, she privately told the attorney, when her husband was taking a bathroom break, that her husband had a bad drinking problem, which was one of her major reasons for seeking a divorce and custody of the children.
    Upon objection by the husband’s attorney, the wife’s testimony should be
    (A) admitted, because the spousal privilege is inapplicable, since the parties are living in separation.
    (B) admitted, because the marital communication privilege is inapplicable in a lawsuit involving the parties.
    (C) excluded, because the wife is incompetent to testify to marital communications because she is not the holder of the privilege.
    (D) excluded, because confidential statements made during a joint consultation are privileged communications between clients and their attorneys.
A
  1. (B) When two or more persons, each having an interest in some problem or situation, jointly consult an attorney, their confidential communications with the attorney will be privileged in a controversy of either or both of the clients with the outside world. But McCormick notes that it will often happen that the two original clients will fall out between themselves and become engaged in a controversy in which the communications at their joint consultation with the attorney may be vitally material. In such a situation, it is clear “that the privilege is inapplicable.” Evidence, pg. 189. Furthermore, the marital privilege does not apply to either, (1) prosecutions for crimes committed by one spouse against the other, or (2) actions by one spouse against the other. Consequently, choice (B) is correct, and choice (D) is incorrect. Choice (A) is wrong because the spousal privilege applies, regardless of the current marital status of the parties (and separation is not divorce). Choice (C) is incorrect. Either party can claim the marital privilege.
122
Q
  1. A plaintiff brought an action against a defendant for personal injuries resulting from a car accident in which the defendant’s car, negligently driven by his cousin, struck the telephone booth in which the plaintiff was resting. Liability is based on a statute making owners of automobiles liable for the negligent actions of those driving with the consent of the owner of the vehicle. The plaintiff offered into evidence the testimony of his doctor that the plaintiff had said to the doctor, when consulting the doctor for treatment, that he felt pain in his back immediately after the accident, and that pain persisted.
    The trial court should rule this testimony
    (A) admissible as a spontaneous declaration.
    (B) admissible under the hearsay exception of declaration of present bodily condition.
    (C) inadmissible as hearsay not within any recognized exception.
    (D) inadmissible as conclusions.
A
  1. (B) Due to the fact that the statement was not made immediately following the occurrence, choice (A) is incorrect. Choice (C) is incorrect because, although hearsay, it falls within a recognized exception. Choice (D) is not accurate, since the plaintiff did not state a legal conclusion (e.g., the ailment was caused by the accident). But rather, he stated that “he felt” pain in his back following the accident. Choice (B) is correct, and the evidence would be admissible as a declaration of present bodily condition, a recognized exception to the hearsay rule.
123
Q
  1. During his lunch break one day, an employee took a company vehicle to a fast-food restaurant and got into an accident on the way back. The driver of the other car sued the company. The company, in its defense, claimed that the employee’s use of the vehicle was unauthorized. At trial, the company calls the employee’s boss. The boss offers to testify that the employee called him five minutes after the accident and stated, “I hope you won’t be sore, because I took the company car without permission.”
    The court should rule that this evidence is
    a. admissible as a declaration against interest.
    b. admissible as an admission.
    c. inadmissible, because the employee was not unavailable.
    d. inadmissible, because it violates the employee’s Fifth Amendment rights.
A
  1. (C) Choice (A) is incorrect because there is no showing that the dectarant is unavailable. Choice (B) is incorrect because the employee is not a party opponent and there is no showing of agency. Choice (D) is an incorrect statement of law. Choice (C) is the correct answer, since unavailability would be a requirement to bring the statement in as a declaration against interest.
124
Q
  1. Following their law school graduation party, a graduate and his roommate were driving home in an automobile that was owned and operated by the graduate. As they were approaching an intersection, their vehicle collided with a motorcycle whose rider was seriously injured in the accident. The rider sued the graduate for operating his motor vehicle in excess of the speed limit. In addition, the roommate brought suit against the rider for operating his motorcycle negligently and driving through a red light. The roommate subsequently reached an out- of-court settlement with the graduate and dismissed the lawsuit against him.
    At trial against the rider, the roommate testified that he observed the rider drive his motorcycle through a red light and then swerve into the path of the graduate’s automobile. In rebuttal, the rider offered into evidence the roommate’s complaint against the graduate alleging that the latter was driving “at an excessively high rate of speed at the time the accident occurred.”
    The complaint is
    (A) admissible as an admission.
    (B) admissible as a declaration against interest.
    (C) admissible under the former testimony exception to the hearsay rule.
    (D) inadmissible, because the suit was dismissed as part of a compromise.
A
  1. (A) A party’s pleadingin onecase, whether a final one orone laterwithdrawn, amended, or superseded, is freely usabLe against him as an evidentiary admission in any other litigation. Choice (B) is incorrect because there is no showing of unavailability. Choice (C) is incorrect because a pleading is not “former testimony.” Choice (D) is an incorrect statement of the law.
125
Q
  1. A driver was severely injured when his car burst into flames after he was rear-ended in a minor auto accident. The driver filed suit against the manufacturer. To prove negligent design by the manufacturer, the driver offers the portion of the transcript from a prior trial in which an expert witness testified that in his opinion the car was unsafe as designed.
    The testimony could qualify under the hearsay exception for former testimony
    (A) only if the manufacturer had been a party to the former proceeding.
    (B) whether or not the manufacturer had been a party to the former proceeding.
    (C) if this exception has been held incompatible with the confrontation clause of the Sixth Amendment.
    (D) if the manufacturer had an opportunity to cross-examine the expert at the former preceding.
A
  1. (B) Although the FRE make former (reported) testimony admissible when proffered against a party, such testimony is also admissible against a predecessor in interest to the former proceeding. Choice (A) is too limited an answer, while choices (C) and (D) are simply incorrect. Thus, choice (B) is the best answer.
126
Q
  1. In order to establish the unavailability of a witness at trial
    (A) it is sufficient to show that the witness is not presently in court.
    (B) it is necessary to show that he has died.
    (C) it is necessary to show that every reasonable effort has been made to procure his attendance.
    (D) if living, it is necessary to show that he is insane.
A
  1. (C) Absence from the courtroom is not enough; thus, choice (A) is not the best answer. Choice (B) is incorrect, since death is not the only circumstance for unavailability. Choice (D) is incorrect, as insanity is only one of the conditions that make a person unavailable. Therefore, the best alternative is choice (C).
127
Q
  1. While riding her bicycle along the street, a woman was struck by a vehicle that she didn’t see. Subsequently, the woman sued the driver of the vehicle to recover damages for her injuries.
    At trial, the woman calls a police officer to testify that a few minutes after the accident, a driver stopped him and said, “Hey, officer, I just saw an accident involving a red truck that hit this girl who was riding a bicycle. The truck left the scene of the accident and I followed it to a warehouse.” The police officer then testified that he immediately drove to the warehouse and saw the defendant sitting in a red truck that was parked in the lot. The driver is available to testify at trial.
    Upon objection by the defendant’s attorney, the police officer’s testimony regarding the driver’s statement should be
    (A) admissible as a statement of recent perception.
    (B) admissible as a present sense impression.
    (C) inadmissible, because the driver was available to testify at trial.
    (D) inadmissible as hearsay not within any recognized exception.
A
  1. (D) A favorite Multistate testing area deaLs with distinguishing between the hearsay rule and the rule requiring firsthand knowledge. The distinction is whether the witness purports to give the facts directly upon his own credit or whether he purports to give an account of what another has told him and this is offered to evidence the truth of the other’s report. Here, you must read the question carefully. You’re being asked to determine the admissibility of the police officer’s testimony regarding the driver’s statement. Although the police officer can properly testify to his own observations, he cannot testify to what the driver told him because it is being offered to prove the truth of the driver’s statement and, thus, inadmissible hearsay. Choices (A) and (B) are, therefore, incorrect because both of them discuss hearsay exceptions. Choice (C) is also incorrect. It suggests that the hearsay would be admissible if only one could establish the unavailability of the driver. The driver’s statement, however, doesn’t fit into any hearsay exceptions under FRE 803, 804, or 807.
128
Q
  1. In a prosecution of a defendant for receiving stolen property, an informer testified that the defendant sold him a stolen stereo, which the defendant knew had been stolen. During direct examination, the informer testified that he was unemployed but sometimes worked part-time as a substitute teacher.
    On cross-examination, the defendant’s attorney asked the informer if he had recently filed an application to become a police officer. The informer responded affirmatively. The defendant’s attorney then asked the informer the following question:
    “Isn’t it true that you hope that by acting as an undercover agent, the police department will overlook the fact that you had two misdemeanor convictions for possession of marijuana?” The prosecuting attorney immediately objected.
    The trial judge should rule that the defendant’s attorney’s inquiry concerning the informer’s hopes and misdemeanor convictions is
    (A) improper, as evidence of conduct not related to truthfulness.
    (B) improper, as relating to convictions of crimes not punishable by imprisonment in excess of one year.
    (C) proper, as tending to show the informer’s bad character for truthfulness.
    (D) proper, as relevant to the informer’s possible bias.
A
  1. (D) Partiality, or any acts, relationships, or motives reasonably likely to produce it, may be proved to impeach credibility. McCormick notes that self-interest may be shown in a criminal case when the witness testifies for the state and it is shown that an indictment is pending against him. By analogy, in this case, the defense attorney may show the informer’s seLf-interest, or bias, because he filed an application to become a police officer and had two misdemeanor convictions on his record. Choice (A) is incorrect because the issue here is bias and not necessarily truthfuLness. Choice (B) is a wrong answer, and somewhat deceptive. The rule it states is true if one is trying to accomplish an impeachment by proof of prior conviction under FRE 609. That is not the purpose of the impeachment by evidence of bias in this case. Choice (C) is incorrect. Evidence of motive or bias does not necessarily prove character for untruthfulness; it merely provides a vehicle for suggesting that the witness’s testimony on this occasion is less worthy of belief.
129
Q
  1. A wife is the beneficiary of a policy issued by an insurance company, insuring the life of her husband, now deceased. The policy contained a clause providing that double indemnity is payable in the event that death of the insured “results directly, and independently of all other causes, from bodily injury effected solely through external violent and unexpected means.”
    The husband was found dead in the chicken shed of his farm. His death resulted from wounds caused by a shotgun blast. The wife filed the necessary papers with the insurance company concerning proof of her husband’s death. The insurance company admitted liability for the face amount of the policy but rejected the wife’s claim for double indemnity.
    The wife then instituted suit against the insurance company demanding judgment according to the double indemnity provisions of the husband’s insurance policy.
    At trial, the wife was called to testify about the events on the day of her husband’s death. The wife said that she was in the kitchen when she heard a gunshot in the shed. As she rushed out of the house, she saw their neighbor running from the shed. The neighbor is present in court.
    As a witness, the wife was
    (A) competent, because she had personal knowledge of the matter.
    (B) competent, because the neighbor is available to testify.
    (C) incompetent, because she had a personal interest in the outcome of the lawsuit.
    (D) incompetent, because she was testifying to facts occurring after her husband’s death.
A
  1. (A) Under FRE 602, a witness must have personal knowledge of the matter he is to testify about. The requirement of “personal knowledge” means that the witness must have observed the matter and must have a present recollection of his observation. Choice (B) is incorrect, since the neighbor’s availability is irrelevant with respect to the wife’s competency to testify. Choice (C) is incorrect, since one having a personal interest is competent to testify. Choice (D) is wrong because, under the circumstances, the Dead Man’s Statute is inapplicable.
130
Q
  1. At a defendant’s trial for the murder of his former employer, the prosecutor offers into evidence a note that was found during a lawful search of the defendant’s apartment. The note states: “The demon compels me to kill my former employer. I thirst for his bloody spirit.” At trial, the defendant’s former girlfriend is called by the prosecution to testify that the handwriting was, in fact, the defendant’s. The defendant’s attorney objects.
    The trial judge should
    (A) sustain the objection on the grounds that the former girlfriend’s testimony would be inadmissible opinion evidence.
    (B) sustain the objection on the grounds that identification of handwriting requires expert testimony.
    (C) overrule the objection on the grounds that the former girlfriend qualifies as an authenticating witness.
    (D) overrule the objection on the grounds that the letter qualifies as a past recollection recorded exception to the hearsay rule.
A
  1. (C) In accordance with FRE 901 (a)(2), a lay person with sufficient familiarity with the handwriting of another person may be called upon to prove or disprove the genuineness of such handwriting. Here, one may properly infer that a former girlfriend would have sufficient familiarity with the handwriting of her boyfriend. Thus, the former girlfriend would be afforded a basis for identifying the defendant’s handwriting specimen. Given the authentication rules of FRE 901, choices (A) and (B) are both incorrect. Choice (D) is incorrect because none of the foundational elements for past recollection recorded apply to this fact pattern.
131
Q
  1. A customer is suing a car dealer for selling him a salvaged vehicle that the car dealer had represented as being brand new. A few weeks before trial, the car dealer approached his sister and said, “Sis, I need some sympathy. I sold a salvaged vehicle to a customer, and now he’s suing me. I didn’t mean any harm by it. I inspected the vehicle and everything.” Unknown to either the car dealer or his sister, the sister’s boyfriend was at the front door and overheard this conversation. When the time for trial came around, the car dealer left the country and refused to attend the trial, telling his attorney to handle it. The customer’s attorney attempted several times to secure the car dealer’s attendance at trial, but was unsuccessful. At trial, the sister’s boyfriend is called to testify about the conversation he overheard.
    On objection by the car dealer’s attorney, the court will most likely rule the testimony
    (A) admissible as a statement of then-existing mental or emotional condition.
    (B) admissible as a declaration against interest.
    (C) admissible as a statement of present sense impression.
    (D) inadmissible as hearsay not within any recognized exception.
A
  1. (B) In accordance with FRE 804(b)(3), the car dealer’s statement, “…I sold a salvaged vehicle to the customer,” may properly be admissible as a declaration against interest. Whenever you are dealing with a declaration against interest, there are two things to remember. First, it must be a declaration against one’s (i.e., the declarant’s) (1) penal, (2) pecuniary, or (3) proprietary interest(s). Second, the declarant must be unavailable to testify at trial. In the present case, the car dealer’s out-of- court statement subjected him to pecuniary liability. As a result, it would qualify as a declaration against interest. According to FRE 804, a person can be considered unavailable if he “is absent from the hearing and the proponent of a statement has been unable to procure [his] [attendance or testimony].” Choice (A) is wrong because the car dealer’s statement does not relate to his mental or emotional condition. Choice (C) is incorrect because the statement describes a past act and not a current impression. And finally, choice (D) is wrong because, even though the statement is hearsay, it fits within a recognized exception.
132
Q
  1. A defendant is on trial for false pretenses. He is charged with selling worthless stock in a dummy corporation to unwitting investors. The defendant is alleged to have masterminded a scheme wherein he set up a nonexistent corporation that never conducted business. The victims were sent prospectuses containing false financial data, which induced them to purchase stock in the phony corporation.
    At trial, the prosecution seeks to introduce into evidence proof that the defendant had set up 10 other so-called dummy corporations that never existed.
    This evidence is
    (A) admissible, to show defendant’s character trait for dishonesty.
    (B) admissible, to show his intent to defraud.
    (C) inadmissible, because character cannot be proved by specific instances of misconduct.
    (D) inadmissible, because the evidence is not relevant.
A
  1. (B) In accordance with FRE 404(b), evidence of other crimes, wrongs, or acts is not admissible to prove the character of a person in order to show that he acted in conformity therewith. It may, however, be admissible for other purposes, such as proof of motive, opportunity, intent, preparation, plan, knowledge, identity, or absence of mistake or accident. It may be helpful to remember this exception by the acronym MIMIC: M-motive, I -intent, M-mistake (absence of which), I-identity, and C-common scheme or plan. Choice (A) is wrong because the evidence is being offered for intent and not to show character. Choice (C) is a true statement, but the wrong answer here because the evidence is being offered for a non-character purpose. Choice (D) is incorrect. The evidence is relevant to show intent.
133
Q
  1. A husband and his passenger were killed in an auto collision involving a defendant. The husband’s wife now brings an action to recover damages for loss of society and companionship. The wife testified that she and her husband had a close relationship and that they were happily married for over 15 years. Then on cross-examination, defendant’s counsel asked her if she “was aware of the affair that her husband was having with his passenger?”
    The wife’s attorney immediately objected to this question. The judge should rule that the question is
    (A) objectionable, because the question is beyond the scope of direct examination.
    (B) objectionable, because it was highly prejudicial.
    (C) unobjectionable, because plaintiff’s knowledge of such an affair was probative of the value to be placed on her loss.
    (D) unobjectionable, because it was relevant on the issue of negligence.
A
  1. (C) Choice (C) is correct since, in plaintiff’s action for loss of society and companionship, the existence of her husband’s infidelity and plaintiff’s knowledge of it would be probative of the value to be placed upon husband’s loss. Moreover, the question would be directly related to, and in impeachment of, hertestimony on direct examination. Choice (A) is wrong; the statement is squarely within the scope of direct. Choice (B) is incorrect. Some of the most effective evidence at trial is highly prejudicial. FRE 403 only excludes prejudicial evidence if its prejudicial impact substantially outweighs its probative value. Choice (D) is wrong. The evidence is relevant for loss, but has nothing to do with negligence.
134
Q
  1. A defendant in an automobile accident case is being sued by the estate of the driver of the other car. At trial, the defendant calls an eyewitness to the collision. The eyewitness testifies that after the crash, he immediately ran to the other driver’s car to try to render assistance. The eyewitness observed the other driver covered with blood from the top of his head down to his toes. He was moaning, gasping, and crying out, “I did not see the other car coming!” The other driver died 10 minutes later. The estate’s attorney objects to the eyewitness’s testimony.
    The trial judge should rule that his testimony is
    (A) admissible as a declaration against interest.
    (B) admissible as a dying declaration.
    (C) inadmissible as hearsay not within any recognized exception.
    (D) inadmissible, because this testimony cannot be admitted in civil cases.
A
  1. (B) In accordance with FRE 804 (b), in a prosecution for homicide or in a civiL action or proceeding, a statement made by a declarant, while believing that his death was imminent, concerning the cause or circumstances of what he believed to be his impending death is not excluded by the hearsay rule. While the common law required that the “dying declaration” be that of the homicide victim, the Federal Rules of Evidence have expanded the theory of admissibility to apply equally in civil cases. Thus, choice (B) is correct, and choice (D) is incorrect. Choice (A) is incorrect because there is nothing to indicate that the statement was against a penal, pecuniary, or proprietary interest. Choice (C) is wrong because the statement does satisfy a recognized hearsay exception.
135
Q
  1. A defendant was arrested and charged with involuntary manslaughter for causing a vehicular homicide. A grand jury was convened, and various witnesses were subpoenaed. At the grand jury, a witness was asked the following question by the prosecuting attorney: “Is it not true that 20 minutes after the accident you told the police that the defendant’s car went through the red light?” He answered, “Yes.”
    The defendant was also subpoenaed to appear before the grand jury. The defendant was represented by counsel at the grand jury hearing. During questioning he was shown the witness’s statement from his testimony. No objection was made to the introduction of the witness’s statement.
    At the defendant’s trial, the prosecuting attorney seeks to introduce into evidence the witness’s statement from the deposition taken at the grand jury hearing. The defendant’s attorney objects to this proffer of evidence.
    The objection at trial will be
    (A) granted, because the statement is hearsay not within any recognized exception.
    (B) granted, because the questioning of the witness was leading.
    (C) denied, because the defendant’s attorney did not object at the grand jury hearing that the witness’s testimony was hearsay.
    (D) denied, because the defendant’s attorney did not object at the grand jury hearing that the questioning of the witness was leading.
A
  1. (A) Grand jury proceedings have traditionally been shrouded in secrecy. Substantial controversy has surrounded the issue of disclosure of testimony given by grand jury witnesses. Clearly, the defense has a right of access to such testimony under Fed. Rule Crim. Pro. Section 26.2(a) once a government witness has testified on direct. Here, the question presented is whether a defendant is required to make objections at the grand jury proceeding in order to preserve the issue at trial. As a general rule, objections that are “substantive” in nature (e.g., relevancy and hearsay) may be raised for the first time when the deposition is offered in evidence at trial. McCormick, pg. 127. Therefore, choice (A) is correct and, for the same reasons, choice (C) is incorrect. However, objections that go to the “manner and form” of questions! answers (e.g., nonresponsive or leading) must be made at the time the deposition is taken. Choice (B) is, therefore, incorrect because no objection was lodged at the proceeding to the form of the question. Although it is a true statement, choice (D) is not the best answer here. Choice (A) is a stronger answer because it provides a contingency (if the defendant’s attorney objects on hearsay grounds) that, if true, would ensure the objection was sustained.
136
Q
  1. A man was arrested and charged with robbery. Upon being taken into custody, he was given his Miranda rights and then taken to the police station for booking. At the stationhouse, the man told a police officer that he was prepared to make a confession. The police officer then turned on a video recorder and videotaped the man’s confession.
    At trial, the prosecution called the police officer to testify to the incriminating statements that the man made in his confession.
    Upon objection by the man’s attorney, the police officer’s proposed testimony is
    (A) inadmissible, because the videotape is the best evidence of the man’s confession.
    (B) inadmissible, because it is hearsay not within any recognized exception.
    (C) admissible, because the police officer had firsthand knowledge of the confession.
    (D) admissible, because the man was given his Miranda rights before the confession was elicited.
A
  1. (C) The best evidence rule applies only when the proponent is attempting to prove the contents or terms of a writing. Sometimes, a writing recites or records a perceivable event or condition, such as a marriage (marriage certificate), payment of money (receipt), or the utterance of certain words (transcript). Here, the proponent wishing to prove the underlying event may offer testimony as an observer. This does not involve the best evidence rule because the proponent is not attempting to prove the terms of a writing, but merely is presenting evidence of an event perceived by a witness with firsthand knowledge. Therefore, choice (A) is a wrong answer. In this question, the police officer’s testimony is not reliant on the writing orthe videotape. He had firsthand knowledge of the man’s confession because he was present and overheard it. Choice (B) is wrong because the police officer is testifying based on personal knowledge and not introducing a hearsay statement. Choice (D) is incorrect because the issue of proper Miranda warnings is not relevant to admissibility under these facts.
    Exam Tip: This distinction between the best evidence rule and firsthand knowLedge is frequently tested on the MBE. Remember, the best evidence rule applies only when the proponent is attempting to prove the contents or terms of a writing.
137
Q
  1. A heavyset man with long red hair robbed a liquor store. Thereafter, a man was arrested and charged with the armed robbery. At the man’s trial, the owner of the liquor store was called to testify. He admitted that he was unable to identify the man, who now had a shaven head, as the robber. The prosecuting attorney then handed the owner six photographs. He proposed to testify, over defense objections, that he had previously told the prosecuting attorney that picture #4, admittedly a picture of the man before he shaved his head, was a picture of the person who robbed his store.
    The owner’s proffered testimony should be adjudged
    (A) admissible as a prior identification by the witness.
    (B) admissible as past recollection recorded.
    (C) inadmissible, because it is hearsay not within any recognized exception.
    (D) inadmissible, because it is a violation of the man’s right of confrontation.
A
  1. (A) FRE 801(d)(1)(C) defines as non-hearsay a prior statement by a witness of identification of a person made after perceiving him, if the person making the identification is available to testify at the trial or hearing and subject to cross-examination concerning the statement. There is no requirement that the witness first be impeached. Nor does a prior identification require that the identifying witness make a positive in- court identification. Choice (A) is correct, since the owner had previously identified the man in picture #4. Choice (C) is incorrect, since prior identifications are defined as non-hearsay. Choice (D) is incorrect because the man’s right to confrontation is not violated, since the owner is available in court and subject to cross-examination. Choice (B) is incorrect because the past recoLlection recorded hearsay exception involves first, an attempted refreshing of the witness’s recollection that fails, followed by the witness reading the words of a memorandum or record into evidence. In this case, the only writing is a photograph.
138
Q
  1. A supermarket had just reopened after a two- week closing for renovations. On the morning of the reopening, a woman was shopping when she suddenly fell and suffered a broken leg. Thereafter, the woman sued the supermarket for personal injuries. In her suit, the woman claimed that she tripped over a paint bucket that had been lying in one of the aisles.
    At trial, the woman called another customer to testify that while he was shopping he heard a thud, turned around and saw the woman fall on the floor. He also testified that he saw an empty paint bucket lying a few feet from the woman’s leg. On cross- examination, the customer got into an argumentative exchange with the supermarket’s attorney. When asked his opinion of what caused the woman’s fall, the customer testified, “She fell because she tripped on the bucket.” He admitted on cross-examination, however, that he didn’t actually see the woman trip on the bucket. The supermarket’s attorney then asked the customer the following question:
    “If you didn’t see the woman trip on the bucket, how do you know that she fell because of it?” The customer answered, “Because my girlfriend who was shopping with me told me later that she saw the woman trip over the bucket.”
    The supermarket’s attorney moves to strike the customer’s last answer. If the trial judge overrules the defendant’s motion and permits the customer’s answer to stand, it will most likely be because
    (A) the customer’s answer was invited by the supermarket’s attorney’s question.
    (B) the customer’s answer was based on firsthand knowledge.
    (C) the customer’s answer was a statement of recent perception.
    (D) the customer’s answer was a present sense impression.
A
  1. (A) Choice (B) is wrong because the customer’s answer was not based upon firsthand knowledge. In order to have firsthand knowledge, a witness must have actuaLly observed the fact (to which he is testifying). The facts clearly indicate that the customer did not see the woman fall. Choice (C) is incorrect because the customer did not perceive the woman falling. Choice (D) is wrong because declarations concerning a present sense impression require that the declarant observe the event at the time the statement is made. In other words, the declaration must be made contemporaneously with the observation. By process of elimination, choice (A) is the best answer.
139
Q
  1. Four men entered into a lease as tenants of a five- bedroom house. They each had a separate bedroom and used the fifth bedroom as a laboratory to manufacture the illegal drug methamphetamine.
    One of the men was the so-called money man; he provided the funds to make the rental payments and to purchase the laboratory equipment. One of the other men was the chemist; he had both an undergraduate and graduate degree in chemistry and knew the formula and procedure for producing methamphetamine. Another of the men had sold drugs in the past and knew potential buyers of the methamphetamine. Finally, the fourth man was the enforcer; his role in the scheme was to make sure that no unauthorized persons entered onto the premises of the house.
    The four men had been involved in this drug venture for seven months when they were finally arrested and charged with conspiracy to manufacture and distribute methamphetamine, and distribution and manufacture of methamphetamine. During the trial, the prosecution wishes to introduce as evidence against the money man, a statement made by the enforcer to the landlord at the time of the signing of the lease, in which the enforcer said, “No matter what you charge us for rent, you better O.K. it with the money man, because he’s the one who will really be paying it.” The money man’s attorney objects.
    The judge’s ruling on admissibility will depend on
    (A) whether the enforcer actually paid all of the rent.
    (B) whether the enforcer had a legal obligation to pay the rent under the terms of the lease.
    (C) whether the landlord may be considered a party opponent.
    (D) whether the statement was made during the course of and in furtherance of the conspiracy.
A
  1. (D) If the enforcer’s statement was made during the course of and in furtherance of the conspiracy, the statement would be admissible as an admission. Under FRE 801(d) (2), “A statement is not hearsay if the statement is offered against a party and is (subsection E) a statement by a co-conspirator of a party during the course and in furtherance of the conspiracy.” The enforcer’s statement concerning the fact that the money man would be paying the rent for the premises where the methamphetamine was being manufactured would certainly fall within the “course of and furtherance of” the conspiracy to manufacture and distribute methamphetamine. All other statements would be incorrect as to prove the admissibility of the enforcer’s statement against co-conspirator, the money man.
140
Q
  1. A defendant has been charged with manufacturing illegal firearms in violation of federal firearms laws. In its case-in-chief, the government wants to introduce a copy of the defendant’s transcript from a technical college to show his certificate as gunsmith. The college is no longer in existence, though, and the whereabouts of the previous officials and employees are unknown. However, the transcript bears a seal above the registrar’s signature, and the court has taken judicial notice of the existence of the college and has found that it is normal for a college to compile a transcript. Finally, the information about the defendant on the transcript has been corroborated by a government witness. Assume the prosecution provided the defendants with a copy of the transcript before trial and informed them of its intent to introduce it as evidence.
    The transcript will be admitted
    (A) as a business record, because the custodian can testifS’ as to how the records of the college were kept.
    (B) under the general hearsay exception, as the transcript in these circumstances has sufficient circumstantial guarantees of trustworthiness.
    (C) only if the registrar’s signature can be authenticated by an expert witness.
    (D) only if the defendant waives his right to object to its admission.
A
  1. (B) The transcript would be admitted under FRE 807, which is the residual “catch-all” exception since, under the circumstances, the circumstantial guarantees of trustworthiness would be met. The court here took (1) judicial notice of the existence of the college and of the fact that colleges normally compile such records, (2) found that the copy of the transcript was an authentic copy because of the registrar’s seal, and (3) received corroborating testimony concerning the contents of the transcript. Therefore, the transcript was properly authenticated and reliable under the circumstances. Choice (A) is incorrect, since the custodian of the record was not available to testify as required by FRE 806 (6). Choice (C) is wrong, since an expert witness is not required under the FRE for authenticating a document. Choice (D) is incorrect, since it implies that the transcript is not admissible. Refer to United States v. Hitsman eta!., 604 F.2d 443 (5th Cir. 1979), concerning one of the first instances wherein a federal court has invoked the residual “catch-all” hearsay exception.
141
Q
  1. A victim was killed by a gunshot in the presence of three men. The fatal wound was inflicted by a single shot, and there was no suggestion of a conspiracy among those present. One of the three men was charged with the killing.
    The prosecution called one of the other men as its first witness. His testimony, if true, established that the defendant fired the shot that killed the victim. The defendant’s attorney then called the man’s longtime neighbor to impeach him.
    Which of the following offers of evidence is most likely to be admitted for the impeachment of the man?
    (A) Testimony that the witness is a cocaine addict.
    (B) Testimony that the witness embezzled money from his employer last year.
    (C) Testimony that the witness is not, in the neighbor’s opinion, worthy of belief.
    (D) Testimony that the witness is an atheist.
A
  1. (C) Without question, Evidence is one of the toughest areas on the Multistate exam. Although the Evidence fact patterns are rather short and simple, the questions themselves are very tricky. Here, for example, is an extremely difficult question dealing with impeachment. Most students will be tempted to choose choice (B). With regard to impeachment, FRE 609 provides, “For the purpose of attacking the credibility of a witness, evidence that he has been convicted of a crime shall be admitted but only if the crime (1) was punishable by death or imprisonment in excess of one year, or (2) involved dishonesty or false statement” Choice (B) is wrong, then, because the neighbor testified that the witness “embezzled money,” not that he was convicted of embezzlement; FRE 609 does not apply. Choice (C) is the best answer because FRE 608 states, “The credibility of a witness may be attacked or supported by evidence in the form of opinion or reputation, but subject to these limitations: (1) the evidence may refer only to character for truthfulness or untruthfulness.” You are being tested here on the distinction between impeachment by opinion and by conviction of a crime.
142
Q
  1. A man and a woman were passengers on a plane. They were seated next to one another in the first- class section of the plane. Midway through the flight, the woman excused herself and went to the restroom. While she was gone, the man rummaged through the woman’s purse and stole $100 in cash. Unknown to the man, a flight attendant saw him steal the money. As is customary in such situations, the flight attendant radioed ahead, and police officers arrested the man when the plane landed.
    The man was subsequently charged with violating an applicable federal larceny statute. During the trial, the prosecuting attorney called the man’s wife as a willing witness against her husband. She proposed to testify that the man confided to her that he did steal the woman’s money during the flight. The man’s attorney objected on the grounds of hearsay and privilege.
    The wife’s proposed testimony is
    (A) admissible, because it is neither hearsay nor privileged.
    (B) inadmissible, because it is hearsay not within any recognized exception, though it is not privileged.
    (C) inadmissible, because it discloses a privileged communication, though it is not hearsay.
    (D) inadmissible, both because it discloses a privileged communication and because it is hearsay not within any recognized exception.
A
  1. (C) This is an extremely tricky question because, in light of the holding in Trammel v. California, 445 U.s. 40 (1980), you might be tempted into choosing choice (A). This is incorrect, however, because although in federal courts one spouse may testify against the other in criminal cases, with or without the consent of the party spouse, there is one very important limitation. According to footnote 42 in Trammel, confidential communications between the spouses are nevertheless privileged. Since the facts indicated that “the man confided” to his wife, his communication is deemed to be confidential and, therefore, remains privileged. The statement is not hearsay because it is an admission under FRE 801(d). Choice (A) is wrong because the statement is privileged. Choice (B) is wrong because the statement is not hearsay within the meaning of FRE 801(d), but it is privileged. Choice (D) is incorrect because the statement is privileged, but is not hearsay.
143
Q
  1. A defendant was prosecuted for assault and battery after he admitted striking the victim with a pool cue during a barroom argument. The defendant claimed that he acted in self-defense after he was attacked by the victim, who was drunk and belligerent. At trial, the defendant called a witness who testified that the victim was the aggressor in the altercation. On cross-examination of the witness, the prosecuting attorney asked the witness the following question:
    “Isn’t it true that when you filed your federal income tax return last year, you failed to report the interest income from your savings accounts?” The prosecuting attorney was informed of this fact by the witness’s accountant.
    Upon objection by the defendant’s attorney, the prosecutor’s question is
    (A) improper, because it is not relevant to the issues in the case.
    (B) improper, because the defendant has not been convicted of any crime in connection with the tax return.
    (C) within the court’s discretion to allow, because filing a false income tax return is an act that bears on the witness’s truthfulness.
    (D) within the court’s discretion to allow, because federal income tax evasion is a crime punishable by imprisonment in excess of one year.
A
  1. (C) This question deals with the highly tested area of “bad act” impeachment. FRE
    608(b) states that “Specific instances of the conduct of a witness, for the purpose ofattackingorsupporting his credibility, otherthan conviction of crime as provided in Rule 609, may not be proved by extrinsic evidence. They may, however, in the discretion of the court, if probative of truthfulness or untruthfulness, be inquired into on cross-examination of the witness (1) concerning his character for truthfulness or untruthfulness, or (2) concerning the character for truthfulness or untruthfulness of another witness as to which character the witness being cross-examined has testified.” The prosecuting attorney’s impeachment of the witness, the defense witness, is proper in this case, since the questioning inquires into unconvicted acts bearing on untruthfulness — i.e., failure to report income interest on the witness’s tax return. Since admissions of such specific instances of conduct are discretionary, choice (C) is the correct answer. Choice (A) is incorrect because, as discussed above, the evidence could be admissible on the issue of credibility. Choice (B) is incorrect. Forthe proposed use of this evidence, a prior conviction is not necessary. Choice (D) is wrong. Allegations of serious offenses that have not yet resulted in a conviction cannot be used for impeachment unless they can be tied into an issue, such as truthfulness, as in this example.
144
Q
  1. A plaintiff sued a trucking company for injuries allegedly suffered when a speeding truck jackknifed and struck her car.
    Which of the following pieces of evidence, if offered by the plaintiff, is most likely to be admitted by the court?
    (A) Testimony concerning subsequent repairs to the truck paid for by the trucking company where they have stipulated to ownership.
    (B) Color pictures of a plastic surgeon operating on the plaintiff showing the incision and bloody surgical tools.
    (C) Testimony from the plaintiff’s boss concerning the amount of time she has missed work, when payroll records had already been admitted as evidence of the plaintiff’s lost wages.
    (D) Testimony of a surprise witness to the accident when the court is willing to grant the trucking company a continuance to prepare for crossexamination.
A

izt. (D) Choice (D) is the best choice. This has high probative value, and the prejudice that could result from a surprise witness can be overcome by the continuance. The easiest way to answer this question is by process of elimination. Choice (A) is wrong. Even though evidence of subsequent remedial measures is sometimes admissible to show ownership and control (though not for an inference of negligence), it is unnecessary here to prove that the defendant owned the truck, since that issue was stipulated. Choice (B) is incorrect. A court would most probably sustain an objection under FRE 403 on the grounds that the evidence is too inflammatory and would encourage the jury to decide the issue on an emotionaL basis. In other words, the pictures are unduly prejudicial, even though they may have some limited probative value as to the extent of injury. Note: The examiners often use gross, disgusting evidence to test you on FRE 403. Choice (C) is wrong. This evidence is cumulative, and a judge could exclude it under Rule 403 on the grounds that it is time-consuming and adds little to the case.

145
Q
  1. One night, a defendant went to play bingo. After losing at bingo, the defendant went on a violent rampage. He stole a car and then picked up a woman at a local bar. After she rejected his advances, the defendant stabbed her to death. The defendant was subsequently arrested and charged with felony- murder under an appropriate federal criminal statute.
    The defendant admitted committing the crime but pleaded not guilty by reason of insanity. At trial in federal court, the prosecuting attorney calls the defendant’s psychiatrist to testify as to the defendant’s mental state at the time of the killing. The defendant’s attomey objects, claiming that the testimony would violate the psychiatrist-patient privilege.
    The objection should be
    (A) sustained, provided the state law recognizes the psychiatrist-patient privilege.
    (B) sustained, provided the court concludes that the privilege should be recognized as part of modern common law.
    (C) overruled, because no such privilege is specifically provided in the Federal Rules of Evidence.
    (D) overruled, because the right to a fair trial overrides the use of a privilege to prevent full exploration of the facts in federal court.
A
  1. (B) The FRE do not contain a specific physician-patient (or any other) privilege. However, FRE 501, the onLy federal rule on privilege, is often tested on the MBE. It provides as follows: “Except as otherwise required by the Constitution of the United States or provided by Act of Congress or in rules prescribed by the Supreme Court pursuant to statutory authority, the privilege of a witness, person, government, State, or political subdivision thereof shall be governed by the principles of the common law as they may be interpreted by the courts of the United States in the light of reason and experience. However, in civil actions and proceedings with respect to an element of a claim or defense as to which State law supplies the rule of decision, the privilege of a witness, person, government, State, or political subdivision thereof shaLL be determined in accordance with State law.” The defense attorney’s objection to the testimony of the defendant’s psychiatrist may not be sustained under choice (A) because the defendant’s trial is a criminal case, and in accordance with FRE 501, the principles of common law (not state Law) wiLl govern. Choice (B) is correct because, under FRE 501, common law “as interpreted . . . in the Light of reason and experience,” wilL determine the privileges applicable in federal question and criminal cases. See McCormick, Evidence, pg. 182. Choice (C) is wrong for the reasons discussed above. Choice (D) is also incorrect. Frequently, privilege law operates to exclude relevant and valuable evidence, thereby interfering with the full exploration of the facts in court. For sound policy reasons, however, we have decided, as a society, that this is an acceptable price to pay.
146
Q
  1. A man and a defendant were college roommates. With the defendant’s permission, his roommate borrowed the defendant’s baseball bat to use in an intramural baseball game. During the course of the game, the roommate struck out with the bases loaded. Angry at himself, the roommate took his bat and flung it into the stands. The bat struck a fan in the face and fractured his nose.
    The fan sued the defendant for his injury, alleging that the defendant was negligent in lending his baseball bat to his roommate when he knew that his roommate was irresponsible with bats. At trial, the fan offers evidence that on four separate occasions during the past year the roommate had negligently thrown bats during other baseball games.
    The fan’s proffered evidence is
    (A) admissible to show that the roommate was negligent on the occasion when the fan was injured.
    (B) admissible to show that the roommate was irresponsible in the use of bats.
    (C) inadmissible, because it is evidence of character.
    (D) inadmissible, because character must be proved by evidence in the form of reputation or opinion.
A
  1. (B) FRE 404(a) states the general rule for character evidence in a civil case: “Evidence of a person’s character or a trait of his character is not admissible for the purpose of proving that he acted in conformity therewith on a particular occasion.” However, under FRE 405(b), ‘in cases in which the character or a trait of character of a person is an essential element of a charge, claim, or defense, proof may also be made of specific instances of his conduct.” In the fan’s suit against the defendant, he is alleging that the defendant was negligent in (ending his bat to his roommate, knowing that he was irresponsible with bats. Since the cause of action here is negligent entrustment, character is in issue. Therefore, FRE 405(b) applies, allowing evidence of the roommate’s specific acts to be admissible to show the character trait that is in issue, specifically, the roommate’s irresponsible use of bats. Evidence of this trait will then be relevant to the defendant’s negligent entrustment. Choice (B) is, thus, correct. Choice (A) is incorrect; the fan’s testimony is not being used to show the roommate’s negligence on this occasion, but rather to show the defendant’s negligence in lending his roommate his bat. Choice (D) is incorrect, since character evidence need not be limited to reputation or opinion evidence when character is in issue.
147
Q
  1. A husband and a wife are filing for divorce. At issue is the market value of the house they own. During the divorce proceedings, the husband’s lawyer calls a neighbor to testify that the market value of the home exceeds $100,000. She states that her testimony is based on an estimate contained in an appraisal report that she saw. The appraisal had been prepared at the husband’s request by a well-known real estate company in the area.
    Upon objection by the wife’s attorney, the neighbor’s testimony will be held
    (A) admissible as proper lay opinion. (B) admissible, because she had firsthand knowledge of the estimate contained in the appraisal report.
    (C) inadmissible, because it involves a privileged communication between husband and wife.
    (D) inadmissible, because the neighbor has not been shown to be an expert on real estate market values.
A
  1. (D) FRE 701 states that “if the witness is not testifying as an expert, his testimony in the form of opinion or inference is limited to those opinions or inferences which are (a) rationally based on the perception of the witness and (b) helpful to a clear understanding of his testimony or the determination of a fact in issue.” Lay opinion under FRE 701 must be based on personal knowledge. Generally, a property owner is qualified to give an opinion as to the value of his property. However, the facts here state that the witness is a neighbor, not the property owner. Moreover, she has no personal knowledge of the fact on which the appraisal report based the market value. Therefore, the neighbor’s testimony will be inadmissible. Choices (A) and (B) are incorrect, since FRE 602 requires evidence “sufficient to support a finding that the witness has personal knowledge of the matter.” The neighbor’s testimony as to real estate market value would require an expert opinion (since experts need not have personal knowledge of the subject matter of their testimony). Choice (D) thus provides the correct rationale.
148
Q
  1. A woman was driving to work when her car was struck by another car. At the time of the accident, the other driver had momentarily taken his eyes off the road while he placed a CD in his dashboard player. Following the collision, which resulted in the woman’s car being extensively damaged, the woman demanded that the other driver pay for the repair work. Whereupon, the other driver said to the woman, “If you will take $2,000, I’m certain my insurance company will pay for it.” The woman refused the other driver’s offer and sued him for damages.
    If, at trial, the woman seeks to testify to the other driver’s statement, this proffered evidence should be ruled
    (A) admissible as an admission by a party-opponent.
    (B) admissible as a statement against interest. (C) inadmissible as a statement made in conjunction with settlement negotiations.
    (D) inadmissible as a self-serving declaration.
A
  1. (C) Under FRE 408, “Evidence of (1) furnishing, offering, or promising to furnish, or (2) accepting, offering, or promising to accept, a valuable consideration in compromising or attempting to compromise a claim which is disputed as to either validity or amount, is not admissible to prove liability. . .“ Since the other driver’s statement to the woman offering $2,000 to pay for the damage to her car was an attempt to settle, it would be inadmissible for public policy reasons. Therefore, choice (C) is correct. Note: Be aware that any statement made in connection with a settlement offer will be inadmissible to prove fault or value of the claim. For example, where the defendant says, “I know I drove through the red light. Anyway, I’m willing to settle and pay for the damage to your car,” the entire statement is inadmissible to prove fault. Choice (A) is incorrect because the other driver’s offer to settle is not an admission. Choice (B) is incorrect. First, FRE 408 would preclude the statement and, second, declarations against interest require a showing of unavailability of trial, which is not present in this fact pattern. Choice (D) contains a rule that is nowhere to be found in evidence law and is, therefore, incorrect as a distracter answer choice.
149
Q
  1. Two cars were driving within the legal speed limit as they approached an intersection. There are no traffic lights, stop signs, or warnings posted at this intersection. Both vehicles entered the intersection simultaneously and collided. In the accident, one of the drivers suffered a broken pelvis and internal injuries. Although the other driver’s truck was slightly damaged, he did not suffer any physical injuries.
    Thereafter, the injured driver sued the city for negligence, claiming that the city failed to provide adequate warning at the intersection, which she contended was particularly dangerous and heavily traveled. At trial, the city attorney offers evidence that the intersection has been the same for 14 years, and no other accidents have occurred during that time.
    Upon objection by the injured driver’s attorney, this evidence should be
    (A) admitted as relevant evidence of a fact of consequence to the action.
    (B) admitted, provided that the jury is instructed about the danger of inferential negative evidence.
    (C) excluded, because it is improper negative evidence.
    (D) excluded, because the dangers of unfair prejudice and confusion of the issues may tend to mislead the jury.
A
  1. (A) A commonly tested area on both the MBE and state essays deals with absence of similar accidents. FRE 403 states that “although relevant, evidence may be excluded if its probative value is substantially outweighed by the danger of unfair prejudice, confusion of the issues, or misleading the jury, or by considerations of undue delay, waste of time, or needless presentation of cumulative evidence.” Application ofthis rule poses considerable difficulty for a defendant wishing to establish the nonexistence of an unduly dangerous condition through the lack of previous accidents. The key determination required to establish relevancy in this case is a showing of substantial identity of material circumstances. By showing fixed or stable conditions, safe use, and an extensive number of experiences, the similarity requirement may be satisfied and, on balance with the probative value, will justify admission. Lilly, Introduction to the Law of Evidence, p. 149. In the classic case of Erickson v. Wa!green Drug Co., 120 Utah 31 (1951), evidence that over 4,000 people a day, for 15 years, in all weather conditions, had used a terrazzo entranceway without anyone slipping, was held admissible negative evidence. Similarly, where a heavily traveled intersection has remained the same for 14 years without any accidents during that time, such evidence should be found relevant and admissible to show nonexistence of a dangerous condition. Choices (B) and (C) are, therefore, incorrect. Choice (A) is correct and a better answer than choice (B), since a jury instruction would not necessarily be required under these facts, but would be subject to the discretion of the court.
150
Q
  1. A relevant fact in a plaintiff’s suit is the magnitude of an explosion at an oil refinery one evening. At trial, the plaintiff was asked on direct examination if he remembered the explosion. He replied, “I recall my son running into the home and screaming that the car windows just shattered.” The son was available to testify at trial.
    The defendant’s attorney makes a motion to strike the plaintiff’s answer. The trial judge should rule the plaintiff’s testimony
    (A) admissible, because the son is available as a witness.
    (B) admissible, whether or not the son is available as a witness.
    (C) inadmissible as hearsay not within any recognized exception.
    (D) inadmissible under the excited utterance exception, because it can be a product of reflection and deliberation.
A
  1. (B) FRE 803(1) provides that a present sense impression is admissible as a recognized hearsay exception. Thus, a statement describing or explaining an event or condition made while the declarant was perceiving the event or condition or immediately thereafter is not excluded by the hearsay rule. Hearsay exceptions under FRE 803 apply regardless of the declarant’s availability at trial. Thus, choice (B) is correct, and choice (A) is incorrect. Choice (C) is incorrect because the statement of the son is, in fact, admissible as a present sense impression. Choice (D) misstates the excited utterance exception, which does not apply if the statement is the result of reflection and deliberation. Choice (D) is, therefore, incorrect.
151
Q
  1. A commissioner of a sports league was charged with embezzling funds by increasing the amount of his allocated travel expenses without the approval of a majority of the team owners. In accordance with the bylaws of the league’s charter, the commissioner was required to obtain the approval of at least half the owners before raising his expense allocation.
    At trial, the commissioner seeks to testify that his predecessor told him that he had authority under the league’s charter to raise his expense allocation.
    The commissioner’s testimony is
    (A) admissible, to show that the commissioner lacked criminal intent.
    (B) admissible as evidence of the routine practice of an organization.
    (C) inadmissible as hearsay not within any recognized exception.
    (D) inadmissible, because the league’s charter is the best evidence.
A
  1. (A) This is a Multistate example of non-hearsay because the predecessor’s statement or utterance is being offered to show its effect on the commissioner, the listener. That is to say, the commissioner is attempting to prove that he acted honestly (without criminal intent) in increasing his expense allocation based upon the information provided him by the predecessor. Consequently, the commissioner’s testimony is not hearsay because its value does not rest upon the truth of the predecessor’s statement. Therefore, choice (C) is incorrect. On the contrary, the predecessor’s utterance is being offered only to show the effect it had on the commissioner’s subsequent conduct (i.e., he reasonably believed that he had the power as commissioner to raise his expense allocation). Choice (B) is incorrect because the foundational elements of FRE 406 on habit and routine practice are not met here: there is no evidence of what the organization “always” or “invariably” did. Choice (D) is wrong because the contents of a writing are not at issue here.
152
Q
  1. A defendant was arrested at an airport when the small suitcase he was carrying was found to contain heroin. The defendant, who did not challenge the legality of the airport search, was subsequently prosecuted for possession of heroin.
    At trial, the defendant testified on his own behalf and said that the suitcase belonged to his girlfriend who was accompanying the defendant when he was arrested. The girlfriend died in a skydiving accident two weeks before the defendant’s trial. Moreover, the defendant testified that although he was a former heroin addict, he had not used any heroin in the past three years. On cross-examination, the prosecuting attorney asked the defendant to roll up the sleeves of his shirt and exhibit his arms to see if there were any needle marks.
    This request is
    (A) objectionable, because the defendant has a privilege against self-incrimination.
    (B) objectionable, because the probative value is substantially outweighed by the danger of unfair prejudice.
    (C) permissible, because such evidence is relevant to the defendant’s credibility.
    (D) permissible, because the defendant waived his privilege against self-incrimination by taking the stand.
A
  1. (C) Choices (A) and (D) are wrong because the privilege against self-incrimination applies only to evidence that is testimonial in nature. Choice (C) is correct because the prosecuting attorney is attempting to attack the defendant’s credibility. According to FRE 806, “When a hearsay statement, or a statement defined in Rule 801(d) (2) has been admitted in evidence, the credibility of the declarant may be attacked, and if attacked may be supported, by any evidence which would be admissible for those purposes if declarant had testified as a witness.” In our case, the defendant testified that he “had not used any heroin for the past three years.” Consequently, it is proper for the prosecution to attack his credibility by seeing if there are any needle marks on his arms. Choice (B) is wrong. For most criminal defendants, the unfortunate reality of trial includes the introduction in evidence against them of highly prejudicial evidence. This fact pattern contains insufficient information to balance probative value and prejudicial impact.
153
Q
  1. A professional football player was seriously injured when the football helmet he was wearing shattered as he was being tackled during a game. The player subsequently asserted a claim against the manufacturer of the helmet to recover damages for his injury.
    At trial, the player’s attorney calls a former employee of the manufacturer to testify that three weeks after the player suffered his injury, the manufacturer devised a new design for its football helmets.
    The former employee’s testimony is
    (A) admissible as an admission.
    (B) admissible as circumstantial evidence that the player’s injury was traceable to the defective design of the football helmet.
    (C) inadmissible, because of the public policy behind taking safety precautions.
    (D) inadmissible, because the former employee is no longer authorized to speak on behalf of the manufacturer.
A
  1. (C) In accordance with FRE 407, “When, after an event, measures are taken which, if taken previously, would have made the event less likely to occur, evidence of the subsequent measures is not admissible to prove negligence or culpable conduct in connection with the event.” Choice (B) is wrong because, although some courts permit evidence of subsequent repairs in strict liability cases, this is not the majority rule. Furthermore, you should not assume that this is a strict liability action, since the facts simply state that the player is asserting a claim against the manufacturer. Quite conceivably, the player may be bringing suit for negligent manufacture or breach of warranty, in which case evidence of subsequent repairs is clearly inadmissible. Choice (A) is wrong because the former employee is no longer an employee of the company and cannot make vicarious admissions on its behalf under FRE 801(d) (2)(D). Choice (D) is incorrect because the former employee is testifying as a fact witness and is not speaking on behalf of the manufacturer.
154
Q
  1. A large man with red hair robbed a liquor store. Thereafter, a defendant was arrested and charged with the armed robbery. At the defendant’s trial, several eyewitnesses testified that they had seen a large redheaded man pull out a gun and rob the owner of the liquor store. The defendant appeared at trial with a shaven head. The prosecution calls a corrections officer to testify that the defendant had red hair when he was first brought to jail. The defendant’s counsel objects.
    The trial judge should rule the correction officer’s testimony
    (A) admissible as a prior identification. (B) admissible, for the limited purpose of clarifying the discrepancy in the witnesses’ testimony.
    (C) inadmissible as hearsay not within any recognized exception.
    (D) inadmissible, because it is opinion testimony.
A
  1. (B) Choice (B) is a better answer than choice (A) because the corrections officer’s testimony does not qualify as a prior identification. A prior identification requires that on a prior occasion, a statement of identification be made. For example, if A testifies that on a prior occasion B pointed to the accused and said, “That’s the man who robbed me,” the testimony, even though it is hearsay, would be admissible as a prior identification if B is present in court and available for cross- examination. Similarly, if B has himself testified to the prior identification. In either case, a prior statement of identification was made before trial. Here, on the other hand, no prior statement was made. Rather, the corrections officer is being called to testify at trial (for the first time) regarding the defendant’s appearance when he was arrested. Choice (C) is incorrect because no out-of-court statements are involved in the corrections officer’s testimony. Choice (D) is wrong. The corrections officer is testifying about a factual observation and not an opinion.
155
Q
  1. In a false imprisonment action by a plaintiff against a defendant, the plaintiff calls a witness to testify that the incident occurred on particular date. The witness was not questioned about an affidavit he made for trial in which he stated that the incident occurred on a different date. After the witness left the stand, but before he was excused, the defendant’s attorney offers into evidence the affidavit.
    Assuming that the affidavit is properly authenticated, the trial judge should rule it
    (A) admissible as substantive evidence.
    (B) admissible for impeachment purposes only.
    (C) inadmissible, because the defendant’s attorney failed to question the witness about the affidavit while on the stand.
    (D) inadmissible as hearsay not within any recognized exception.
A
  1. (B) In accordance with FRE 607, the credibility of a witness may be attacked by any party calling him (to testify). According to McCormick, there are five main lines of attacking the credibility of a witness. The first, and probably the most effective and most frequently employed, is an attack by proof that the witness, on a previous occasion, has made statements inconsistent with his present testimony. Therefore, choice (B) is the best answer. The second is an attack by a showing that the witness is biased. The third is an attack upon the character of the witness. The fourth is an attack by showing a defect of capacity in the witness to observe, remember, or recount the matters testified about. The fifth is proof by other witnesses that material facts are otherwise than as testified to by the witness. Choices (A) and (0) are wrong because, under the traditional view of hearsay, previous statements are inadmissible as substantive evidence (i.e., as evidence of what they state) because they constitute hearsay and are not within any exceptions to the hearsay rule. Choice (C) is incorrect because, under these facts, the witness has not been excused. Therefore, the witness can be afforded an opportunity to explain or deny the inconsistence under FRE 613.
156
Q
  1. In which of these situations are the communications set forth privileged?
    (A) An attorney was a member of a seven-attorney law firm. On a busy day, three of his partners had clients waiting to see them, and all of the chairs in the office waiting room were filled. A woman had an appointment with the attorney, and the attorney came out to greet her in the crowded waiting room. As the woman saw the attorney she said, “Am I glad to see you! I need a will made up right away, because I don’t want my rotten son to get any of my money. He will just throw it away at the race track.” The attorney replied, “Let’s go back to my office to discuss this matter.” The attorney prepared a will for the woman.
    (B) A woman arrived at a law office for an appointment and was escorted to her attorney’s office. She sat down and said to the attorney, “You know, I pay you a large retainer every year for your valued counsel. I need some advice now. There was a fire in my house yesterday. It was minor, but I’m covered for $15,000 under my fire insurance policy. I didn’t have any furniture in the room where the fire took place, but I want to put in a claim for a sofa and love seat. How should I go about it?” The attorney refused to give her any advice.
    (C) A woman engaged an attorney to represent her in adoption proceedings. During the course of his representation, she told the attorney some very personal facts about her lifestyle that the attorney needed in order to prepare a petition for adoption. The adoption took place, but the woman never paid her agreed fee to the attorney. The attorney sued the woman to collect his fee.
    (D) A woman consulted an attorney regarding a sale of 70% of the stock of a closely held corporation in which the woman was the majority shareholder. She explained the reason for the sale and divulged the financial condition of the corporation and its prospects for the future. The attorney determined that the transaction would be complicated. Since he felt he was inexperienced in handling such a transaction, the attorney decided to decline to represent the woman. The woman nevertheless sold her stock with the assistance of another attorney.
A
  1. (D) Under the FRE, there are no specific privilege provisions. FRE 501 provides that the privilege of a witness shall be governed by the principles of common law as they may be interpreted by the courts of the United States in light of reason and experience. However, in civil actions and proceedings, with respect to an element of a claim or defense as to which state law applies the rule of decision, the privilege of a witness shall be determined in accordance with state law. Thus, the common Law attorney- client privilege is applicable in our factual situations. The client has a privilege to refuse to disclose and to prevent other persons from disclosing any confidential communications between himself and his attorney. Choice (D) is the best answer, since the woman intended that her disclosures to the attorney about the corporation were confidential and in furtherance of the rendition of legal services that she sought from the attorney. McCormick, in The Handbook on Evidence, p. 179, states that “Communications in the course of preliminary discussion with a view to employing the lawyer are privileged though the employment is in the ‘upshot’ not accepted by the attorney. . . . Payment, or agreement to pay a fee, however, is not essential.” In choices (A), (B), and (C), the communications between the attorney and his client are not confidential and, therefore, not protected by the attorney- client privilege. In choice (A), the communication is made publicly and, therefore, there is no expectation of confidentiality. In choice (B), the communication was made in connection with the woman’s attempt to get the attorney to assist her in a fraud. Such statements are not confidential. In choice (C), the statements are not confidential because they were integrated into the pleadings of the case.
157
Q
  1. One afternoon, a pilot was flying a small airplane when it suddenly ran out of gas. As he was coming in for an emergency landing, the plane crossed into a neighboring state at a very low altitude. At this time, a 9-year-old boy was walking to school when he was struck and injured by an object, which may have fallen from the plane.
    In federal court, a negligence suit was brought against the pilot by the father of the boy for his son. Accompanied by his father, the boy had visited an attorney for preliminary discussions regarding the case. However, the father did not retain the attorney to represent his son in the lawsuit. Instead, the father hired another lawyer to handle the case.
    At trial, the pilot’s attorney calls the consulting attorney to testify what the boy had said to him regarding his physical condition during the consultation that the attorney had had with the boy and his father.
    The attorney’s testimony is
    (A) admissible, because the attorney-client privilege was waived by the filing of the lawsuit.
    (B) admissible, because there is no privilege of confidentiality when a person other than the client is present at the attorney-client consultation.
    (C) inadmissible, because the attorney-client privilege prevents such a breach of confidential communications.
    (D) inadmissible, because it was a statement of physical condition not made for the purpose of obtaining medical treatment.
A
  1. (C) The privilege for communications of a client with his attorney hinges upon the client’s belief that he is consulting a lawyer in that capacity and has manifested intention to seek professional legal advice. According to the hoLding in the case of In re Dupont’s Estate, 140 R2d 866 (1943), communications in the course of preliminary discussion with a view of employing the attorney are privileged even though the attorney is never hired. Choice (A) is wrong; the privilege cannot be defeated by the filing ofa lawsuit. Choice (B) is incorrect. The father’s position as father and guardian ad 1/tern of his minor child puts him on the same footing as the child for purposes of determining the existence of the privilege. Choice CD) is incorrect. Choice (0) misstates the medical hearsay exception, which permits the admissibility of information provided for the purpose of obtaining medical treatment.
158
Q
  1. A taxpayer was notified by the government that her individual income tax was underpaid by $1,012.69. The taxpayer retained an attorney to represent her in contesting the assessment. During the preparation of his client’s case, the attorney suggested that it might be a good idea if the taxpayer hired an accountant to organize her records and prepare a financial statement for the year in question. Following the attorney’s advice, the taxpayer consulted an accountant, an old family friend, who prepared a financial statement, which the attorney referred to at trial.
    During the trial, the attorney representing the government called the accountant to testif’ about statements that the taxpayer made to him.
    The accountant’s proposed testimony is
    (A) inadmissible, because it would violate the attorney-client privilege.
    (B) inadmissible, because it would violate the taxpayer’s privilege against self-incrimination.
    (C) inadmissible as violative of the work-product rule. (D) admissible as an admission.
A
  1. (0) In Fishery. United States, 425 U.S. 391 (1976), the court held that compliance with a subpoena for an accountant’s records (that defendant had given his attorney in preparation for trial) did not violate the defendant’s 5th Amendment privilege against self-incrimination. For this same reason, choice (B) is incorrect. Choices (A) and (C) are incorrect because the accountant was not a representative of the attorney. The facts do not state that the attorney hired the accountant. Rather, the accountant was employed by the taxpayer. A “representative of the lawyer” is one employed by the attorney to assist the attorney in the rendition of professional Legal services.
159
Q
  1. Late one afternoon, a woman was hitchhiking when she was picked up by a man. Shortly thereafter, the man stopped and parked his car in a roadside rest area. They were smoking marijuana when another car skidded and crashed into the man’s car. The collision damaged the man’s car and inflicted personal injuries upon him. The woman was likewise injured. In fact, the impact produced a state of unconsciousness in her that lasted several minutes.
    The woman sued the other driver seeking to recover for his alleged negligence. At trial, the woman testified to her injuries and to the other driver’s negligence. In defense, the other driver called an attorney who lived next door to the woman. The attorney proposed to testify that after the accident, the woman consulted him about her claim and asked the attorney in confidence how she could falsely testify that she wasn’t smoking marijuana at the time of the accident.
    Upon objection by the woman’s attorney, the attorney’s testimony is
    (A) admissible as an admission.
    (B) admissible as a statement against interest.
    (C) inadmissible, because it violates the woman’s privilege against self-incrimination.
    (D) inadmissible, because it violates the attorney-client privilege for confidential
    communications.
A
  1. (A) Under the attorney-client privilege, a client has a priviLege to refuse to disclose and to prevent any other person from disclosing confidential communications made for the purpose of facilitating the rendition of professional legal services to the client between himself (or his representative) and his attorney (or his attorney’s representative). However, there are exceptions when there is not privilege. For example, the privilege is lost if the services of the attorney were sought or obtained to enable or aid anyone to commit or plan to commit what the client knew or reasonably should have known to be a crime or fraud. Thus, choice (A) is correct, and choice (D) is incorrect because the woman sought the attorney’s advice about committing perjury or falsely testifying about the incident. Choice (B) is incorrect because there has been no showing of unavailability in this case. Choice (C) is incorrect. Voluntary statements made to others outside the courtroom are not covered by the privilege against self-incrimination.
160
Q
  1. A shopper sued a supermarket for injuries allegedly suffered from a collision with the store’s automatic doors. The shopper contended that the doors, which were programmed to swing inward, swung outward and injured him as he attempted to enter the store one afternoon. The shopper’s brother-in- law, who was an eyewitness to the accident, was not called to testify at trial. Moreover, the shopper’s attorney failed to depose the brother-in-law, who redomiciled out of state shortly after the accident.
    With respect to the shopper’s failure to offer the brother-in-law’s testimony at trial, on request by the supermarket’s attorney, the court should
    (A) instruct the jury that it raises the presumption that the brother-in-law’s testimony would have been unfavorable to the shopper.
    (B) instruct the jury that it constitutes an adoptive admission that the brother-in-law’s testimony would have been unfavorable to the shopper.
    (C) permit the supermarket’s attorney to argue that it raises a presumption that the brother-in-law’s testimony would have been unfavorable to the shopper.
    (D) neither instruct the jury on the matter nor permit the supermarket’s attorney to argue the matter.
A

i6o. (0) A popular Multistate testing area deals with whether the failure of a party to call a particular witness (or the failure to take the stand himself as a witness) allows his adversary to use this failure as a basis for invoking an adverse inference. Although there are a large number of cases supporting the inference, under the prevailing view, if the witness is “equally available” to both parties, no inference springs from the failure of either to call him. McCormick, Law of Evidence, pg. 657. McCormick points out that “the possibility that the inference may be drawn invites waste of time in calling unnecessary witnesses or in presenting evidence to explain whythey were not called.” Most important, McCormick notes that “the availability of modern discovery procedures serves to diminish both the justification and the need for the inference.” Law of Evidence, pg. 657. For this reason, answers (A), (B), and (C) are all incorrect.

161
Q
  1. During the murder trial of a defendant, the prosecution presented four witnesses to the brutal slaying of the victim. The evidence pointed to the fact that the defendant beat her about the head and neck with a baseball bat, causing severe injuries to her brain and her ultimate death.
    The prosecution rested, and the defendant presented two witnesses, his brother and his girlfriend, who testified that the defendant was dining at an elegant restaurant on the other side of town at the time of the alleged murder. The defendant presented no other witnesses.
    During his closing argument to the jury, the assistant district attorney called attention to the fact that the prosecution witnesses had no apparent reason to have any bias toward the prosecution or against the defendant. He then noted that the defendant’s witnesses had clear motives to falsify their testimony and favor the defendant. The assistant district attorney added, “If the defendant was on the other side of town, why didn’t he tell us himself? Why didn’t he get on the stand? What was he hiding? Those are questions for you, the jury, to answer.”
    The defendant was convicted of first-degree murder and sentenced to life imprisonment.
    On appeal, his conviction should be
    (A) reversed, because the prosecutor improperly referred to the possible motives or interests of the defense witness.
    (B) reversed, because the defendant’s constitutional rights were violated during the closing argument.
    (C) reversed, because the assistant district attorney referred to the defendant’s failure to testify.
    (D) reversed, because the assistant district attorney’s argument violated the defendant’s rights under the Fifth and Fourteenth Amendments.
A

i6i. (D) In Griffin v. California, 380 U.S. 609 (1965), the U.S. Supreme Court held that the self-incrimination guarantee of the 5th Amendment, as applicable to the states under the 14th Amendment, forbids either comment by the prosecution of an accused’s silence or instructions by the court that such silence is evidence of guilt. The comments by the prosecutor during his closing argument that the defendant failed to take the stand would be violative of the defendant’s right against self- incrimination. Choices (B) and (C) are also correct, but choice (D) is a better answer because it is more specific and on point to the facts of the question. Choice (A) is incorrect. The motives and bias of a party’s witnesses are fair game in closing argument.

162
Q
  1. A woman sued a man for personal injuries that she suffered when she was struck by the man’s car. The man’s car hit the woman as she was walking across the street. Immediately after the accident, the man ran over to the woman and said, “I know I was driving fast, but you weren’t paying attention where you were walking. Anyhow, I’m willing to pay all your medical expenses.”
    At trial, the woman calls an eyewitness to the accident. The eyewitness proposes to testify that he heard the man tell the woman, “I know I was driving fast.” The man’s attorney objects.
    If the eyewitness’s testimony is admitted, it will most likely be because the proffered evidence is
    (A) admissible as an opinion.
    (B) admissible as an admission.
    (C) admissible as a present sense impression.
    (D) admissible as a declaration against interest.
A
  1. (B) If the eyewitness’s testimony is admitted, it will most likely qualify as an admission. According to FRE 409, “Evidence of furnishing or offering or promising to pay medicaL, hospital, or similar expenses occasioned by an injury is not admissible to prove liability for the injury.” As is so often the case on the MBE, an admission may be made in connection with an offer or promise to pay medical or hospital expenses. In this instance, the admission of liability is admissible, but evidence of offering or promising to pay medical (or similar) expenses is inadmissible, in accordance with public policy considerations. Choice (A) is incorrect. The statement, even if it is in the form of an opinion, is an admission by a party opponent. Choice (C) is wrong because the statement does not “describe or explain an event or condition made while the decLarant was perceiving the event or condition, or immediately thereafter.” Choice (D) is wrong because there is no indication of the man’s unavailability at trial.
163
Q
  1. One day, while visiting a plastic surgery clinic for a routine Botox treatment, a patient received an excessive dose of Botox from a new medical assistant on the staff. Her face frozen in a grotesque grimace, she demanded to speak to the doctor. The doctor said, “Listen, why don’t you go over to the emergency room and send the bills to me? I’ll take care of your medical expenses.” The patient subsequently sued the doctor for negligent hiring of the medical assistant and for damages. At trial, she proposes to call the doctor’s administrative assistant, who was present during her conversation with the doctor. The patient wants the administrative assistant to testify to the doctor’s offer to pay her medical bills.
    The doctor’s attorney objects. The trial judge should
    (A) sustain the objection, because the patient’s medical records are the best evidence of the doctor’s negligence.
    (B) sustain the objection as an offer to pay the medical bills.
    (C) overrule the objection, because the evidence is relevant as to the question of the doctor’s negligent hiring.
    (D) overrule the objection, because an offer to pay medical bills is an inferential admission.
A
  1. (B) As noted previously, evidence of payment (or offering or promising to pay) of medical, hospital, or similar expenses of an injured party by the opposing party, is not admissible. The reason often given is that such payment or offer is usually made from human impulses and not from an admission of Liability, and that to hold otherwise would tend to discourage assistance to the injured person. Choice (B) is correct because the aforesaid rule is often phrased in terms of “humanitarian motives.” See FRE 409 Advisory Committee’s Note. For the same reasons, choice (D) is incorrect. Choice (A) is wrong because it incorrectly states the best evidence rule. Choice (C) is incorrect. The doctor’s offerto pay medical bills might be relevant to the issue of negligent hiring, but this tenuous connection will be trumped by the policy considerations of FRE 409.
164
Q
  1. A surgeon performed an appendectomy on a patient in an operating room at the local hospital. The surgeon was assisted by a nurse who was assigned to the operation by the hospital. During the patient’s early convalescence, he complained of pain not explicable as an ordinary post-operative symptom. On investigation, it turned out that the surgeon, who had bandaged him following the operation, had done so in such a manner as to constrict certain blood vessels. The faulty bandaging had caused acute pain and retarded the patient’s recovery.
    After the patient’s eventual recovery, he sued the surgeon for malpractice, claiming $25,000 in damages. In his case-in-chief, the patient called the nurse to testifS’ that shortly after the surgery, she saw the surgeon destroy the postoperative x-rays of the patient’s abdomen.
    Upon objection by the surgeon’s attorney, the trial judge should rule the nurse’s testimony
    (A) admissible, provided that the judge determines that the surgeon destroyed the x-rays as a cover-up.
    (B) admissible, but leave the weight of the nurse’s testimony to be determined by the jury.
    (C) inadmissible, because the probative value is substantially outweighed by the danger of unfair prejudice.
    (D) inadmissible, because it is extrinsic evidence of a collateral matter.
A
  1. (B) The nurse’s testimony as to seeing the doctor destroy the postoperative x-rays of the patient’s abdomen should be admissible. Since the patient’s suit for malpractice is based on the faulty bandaging following the operation, the postoperative x-rays would be relevant, not collateral, to show the effects of the constricted blood vessels on the patient’s retarded recovery, under FRE 401, relevant evidence is that which tends to make the existence of a fact of consequence more (or less) probable than it would otherwise be. ReLevant evidence is generally admissible under FRE 402. Once admitted, the jury then determines the weight of the evidence. Choice (B) is the correct answer, since the relevance of the postoperative x-rays goes to the malpractice issue being litigated, namely the patient’s postoperative pain and slow recovery. Proof of a cover-up is not a foundational prerequisite to admissibility. Choice (C) is incorrect, since the probative value of the nurse’s testimony far outweighs the danger of unfair prejudice; testimony regarding destruction of the x-rays will not confuse the issues, mislead the jury, or cause undue delay. FRE 403. Choice CD) is incorrect because the destruction of the x-rays is not a collateral matter.
165
Q
  1. A plaintiff is the beneficiary of a policy issued by an insurance company insuring the life of his wife, now deceased. The policy contains a clause providing that double indemnity is payable in the event that death of the insured “results directly, and independently of all other causes, from bodily injury effected solely through external violent and accidental means.”
    The plaintiff’s wife met her death in the silage shed of her farm. The death resulted from wounds caused by the discharge of a double-barreled shotgun. The plaintiff was arrested and prosecuted for the murder of his wife. After a lengthy trial, the plaintiff was acquitted of the charge.
    After the insurance company refused to pay the plaintiff’s insurance claim, the plaintiff instituted this civil action. The complaint sets forth the policy, alleges the facts surrounding the death of the insured, avers that the death was within the policy provisions for double indemnity payment, and demands judgment accordingly.
    At trial, the insurance company’s attorney proposes to introduce excerpts from the plaintiff’s murder trial. The plaintiff’s attorney objects to the introduction of such evidence.
    The trial judge should rule the proffered evidence
    (A) admissible as a declaration against interest.
    (B) admissible as former testimony.
    (C) inadmissible, because of collateral estoppel.
    (D) inadmissible, because of double jeopardy.
A
  1. (B) This is an extremely tricky Evidence example. Many students will recognize that unavailability (as a witness) is a requirement for the former testimony exception to the hearsay rule. Although this is true, former testimony may often be given in evidence without meeting the requirements of unavailability and confrontation. McCormick states that these requirements are applicable only when admission of the evidence is sought under the hearsay exception. However, when the former testimony is offered for some non-hearsay purpose, as to show the commission of the act of perjury, to refresh recollection, or impeach a witness at the present trial by proving that he testified differently on a former occasion, the restrictions of the hearsay exception do not apply. See McCormick, pg. 615. Since all other choices are clearly inapplicable, choice (B) is the only conceivably correct answer.
166
Q
  1. A rescuer was driving on an isolated portion of a country road. His headlights caught a figure lying at the side of the road. The rescuer stopped to investigate and found a victim, who was bleeding from head wounds and appeared to have been severely beaten.
    The rescuer then lifted the victim into his car and drove her to the hospital, a half-hour trip. When they arrived at the hospital, the rescuer carried the victim into the emergency room. He left her with a nurse and then returned home. Although the victim recovered from her injuries, she sued the hospital for malpractice, claiming that she was not promptly given medical attention.
    At trial, the nurse proposes to testify that when the victim was first brought to the hospital, she was unconscious. The victim’s attorney objects and moves to strike the nurse’s testimony.
    The trial judge should
    (A) sustain the objection, because it goes to an ultimate issue in the case.
    (B) sustain the objection, because the nurse is not qualified to render an expert opinion.
    (C) overrule the objection, because it is a shorthand rendition of what she observed.
    (D) overrule the objection, because there are independent grounds to show a present sense impression.
A

i66. (C) According to FRE 701, “If the witness is not testifying as an expert, his testimony in the form of opinions or inferences is limited to those opinions or inferences which are (a) rationally based on the perception of the witness and (b) helpful to a clear understanding of his testimony or the determination of a fact in issue.” Choice (C) is correct because the federal rules have adopted the so-called shorthand rendition rule, which incorporates the more liberal notion of sanctioning the admission of opinions on grounds of “expediency” and “convenience,” rather than “necessity.” Choice (A) is incorrect because an opinion witness can testify to the ultimate issue in a case under FRE 704. Choice (B) is incorrect. The nurse is not offering an expert opinion but, rather, an opinion within the province of lay witnesses. Choice (D) is incorrect. The nurse is not testifying to an out-of-court statement by a hearsay declarant, but rather to her own observations and conclusions.

167
Q
  1. A doctor is charged with the murder of his wife. The prosecution alleges that he murdered his wife by giving her a massive injection of succinylcholine while she was asleep. Succinylcholine is a drug used in small quantities by anesthesiologists as a muscle relaxant. The prosecution claims that the fatal dose given to the wife so totally relaxed her lung muscles that she suffocated.
    During the trial, a toxicologist, who had no previous knowledge of the case, sat in court and heard all of the evidence about the wife’s death and autopsy.
    As part of the doctor’s defense, his attorney calls the toxicologist to give his opinion on the cause of the wife’s death.
    May the toxicologist so testify?
    (A) Yes, because he can identify the data upon which his opinion is based.
    (B) Yes, because an expert may base his opinion on facts made known to him at the trial.
    (C) No, because he has no personal knowledge of the wife’s death.
    (D) No, because the cause of death is an issue to be decided by the jury.
A
  1. (B) FRE 703 provides that the “facts or data in the particular case upon which an expert bases an opinion or inference may be those perceived by or made known to him at or before the hearing.” Since an expert may base his opinion on facts made known to him at the trial, choice (B) is correct. Choice (A) is incorrect because FRE 705 states that an expert can testify to her opinion or inferences without having to first testify to the underlying facts or data. Choice (C) is incorrect. Under FRE 703, an expert is not required to have firsthand knowledge of the case. Finally, FRE 704 permits expert opinion on the ultimate issue in a case. Therefore, choice (D) is an incorrect answer.
168
Q
  1. For nearly three months, a supermarket underwent extensive renovations. The store was temporarily closed during the renovation period. The day the supermarket reopened, the store manager noticed that small fragments of plaster had fallen from a section of the ceiling. He promptly posted signs warning shoppers of the hazardous condition. The signs, which were printed in bold letters, read: “ATTENTION SHOPPERS - BE ON THE LOOKOUT FOR FALLiNG PLASTER.”
    That same afternoon, a shopper was shopping in the supermarket and noticed the signs. She looked at the ceiling but didn’t see any plaster falling. Moments later, she was placing some squash in a bag when a section of the ceiling suddenly fell on her head. She suffered a concussion and head lacerations.
    Thereafter, the shopper brought a tort action against the supermarket to recover for the injuries she suffered. Her attorney hired a physician to examine the shopper in order to assist the attorney in preparing the case.
    At trial, the supermarket’s attorney calls the physician that the shopper’s attorney hired as a witness and seeks to ask the physician about statements concerning the injuries that the shopper had made to the physician in confidence and that the physician had in turn communicated to her attorney.
    The physician’s testimony should be
    (A) admitted, because the shopper waived the physician-patient privilege by placing her physical condition in issue.
    (B) admitted, because the shopper’s statements are deemed admissions of a party-opponent.
    (C) excluded, because the shopper’s statements are protected by the physician-patient privilege.
    (D) excluded, because the shopper’s statements are protected by the attorney-client privilege.
A

i68. (D) Choices (A) and (C) are both incorrect, as the physician is an attorney representative and not the shopper’s physician. Choice (D) is the best answer because the physician is a “representative” of the shopper’s attorney and, therefore, the communications between the shopper and the physician will be privileged under the attorney-client privilege. Choice (B) is wrong because the shopper’s statements describe her injuries and are not admissions.

169
Q
  1. A defendant was charged with killing a victim during a barroom fight. The defendant claimed that he acted in self-defense when he was attacked by the victim. At trial, the defendant called a witness who testified that he witnessed the altercation and that the victim was the aggressor. The witness further testified that he has known the defendant for 10 years and that the defendant is a peaceable man who was acting reasonably in self-defense. The prosecution then offered the testimony of the victim’s brother-in-law, who proposed to testify that the victim was a peaceable and law-abiding person.
    Upon objection by the defendant’s attorney, the brother-in-law’s proposed testimony is
    (A) admissible, for the purpose of rebutting the contention that the defendant was a peaceable person.
    (B) admissible, for the purpose of repudiating the defendant’s claim of self-defense.
    (C) inadmissible, because the victim’s character cannot be proved by the brother-in-law’s testimony.
    (D) inadmissible, because the defendant did not directly attack the victim’s character.
A
  1. (B) When the accused has produced evidence that the deceased attacked him, thus grounding a claim of self-defense, this, when met by counter-evidence, raises an issue of conduct: Was the deceased or the accused the first aggressor? It is universally held that when such evidence has been produced, the accused may introduce testimony of the reputation of the deceased for turbulence and violence. Moreover, McCormick points out that “it is equally well settled of course that the prosecution may meet this by rebutting testimony of his (the victim’s) good reputation for peacefulness.” Evidence, pp. 460—461. Thus, choice (B) is correct. Choice (A) is incorrect because evidence of the victim’s peaceful character does not rebut the testimony that the defendant is a peaceful person. Choice (C) is incorrect. According to FRE 405, reLevant character evidence can be established by reputation or opinion testimony, as was done in this case. Choice (D) is incorrect. In order for the prosecution to introduce evidence of the victim’s peaceful character in a homicide case, there is no requirement that the defense directly attack the victim’s character. Instead, it is enough for the defense to introduce evidence that the victim was the first aggressor.
170
Q
  1. A man claims to have inherited property from a wealthy businessman. The man’s birth certificate shows that he was born out of wedlock. The record of a proceeding for a support order shows that the businessman was adjudged to be father of the man. However, the man can establish his status as heir of the businessman only if he was legitimated by the marriage of his mother and the businessman subsequent to his birth. Thus, the man must prove that this marriage occurred.
    The man’s attorney has not been able to discover any marriage license or certificate. However, the attorney does have a photograph that shows a couple dressed in wedding clothes. The scene is the front of a church. Bystanders are seen to be throwing rice at the couple and they seem to be responding by smiling and bowing down. The attorney was given the photograph by the man. He found it in his mother’s effects following her death. The man believes that the bride in the picture is his mother. He cannot identify the groom. The attorney was informed by a former acquaintance of the businessman who has seen the snapshot that he thinks the groom is the businessman.
    If the attorney seeks to introduce the photograph as proof that the mother and the businessman were married, the trial judge should rule the photograph
    (A) admissible, only if the photographer is available to testify concerning the circumstances under which the photograph was taken.
    (B) admissible, only if a witness verifies that it is a correct and accurate representation of the relevant facts.
    (C) inadmissible as non-verbal hearsay not within any recognized exception.
    (D) inadmissible as not the best evidence.
A
  1. (B) McCormick instructs thatthe principle upon which photographs are most commonly admitted into evidence is the same as that underLying the admission of illustrative drawings, maps, and diagrams. Under this theory, a photograph is viewed merely as a graphic portrayal of oraL testimony, and becomes admissible only when a witness has testified that it is a correct and accurate representation of relevant facts personally observed by the witness. Under this theory, the witness need not be the photographer; therefore, choice (A) is incorrect. Choice (C) is an incorrect statement of law and is, therefore, a wrong answer. Choice (D) is the wrong answer because the photograph itself is being introduced into evidence. Therefore, the best evidence rule is not implicated.
171
Q
  1. A defendant was charged with attempting to possess and distribute narcotics. The defendant was arrested after allegedly trying to purchase several kilos of heroin from an undercover government agent. At trial, the undercover agent testified about the defendant’s efforts to arrange the drug transaction. The defendant’s defense, on the other hand, consisted entirely in attacking the undercover agent’s credibility.
    During his summation, the prosecuting attorney, over defendant’s objections, stated that he had produced the only witness who was willing to take the stand and testify to what occurred at the meeting when the undercover agent arrested the defendant. Based on the undercover agent’s uncontradicted testimony, the prosecuting attorney urged the jury to convict the defendant. The defendant was convicted and, on appeal, argued that the judge erred in permitting the prosecutor’s statement.
    The defendant’s conviction should be
    (A) reversed, because the prosecutor’s remarks constituted improper comment about the defendant’s right to testify.
    (B) reversed, because the prosecutor’s remarks were not proper rebuttal to defense attacks on the undercover agent’s credibility.
    (C) affirmed, because it is immaterial; the probative value of the defendant’s failure to testify is outweighed by the danger of unfair prejudice.
    (D) affirmed, because the prosecutor had the right to express the strength of the evidence he had pursued.
A
  1. (A) Under the prevailing view, the failure of a defendant to take the stand himself as
    a witness does not allow his adversary to use this failure as a basis for invoking
    an adverse inference. See McCormick, Law of Evidence, pg. 656. Choice (B) is the
    wrong answer because the issue here concerns improper comment on the defendant’s failure to testify and not the proper scope of rebuttal evidence. Choice (C)
    is incorrect as a matter of Law. Choice (D) is incorrect. Prosecutors may strike hard
    blows, but they must be fair ones.
172
Q
  1. A national association brought suit in federal court against a city. The suit charged the city with violating several federal statutes. During the trial, the association’s attorney called the city attorney as an adverse witness. The city attorney asserted the attorney-client privilege and objected that the matters communicated to him were intended to be confidential.
    The issues raised by the objections of the city attorney should be resolved under
    (A) federal privilege law.
    (B) the privilege law of the forum state.
    (C) either federal privilege law or the privilege law of the forum state as determined by the court.
    (D) either federal privilege law or the privilege law of the forum state, whichever the court determines is more likely to admit the evidence.
A
  1. (A) FRE 501 states that “except as otherwise required by the Constitution of the United
    States or provided by Act of Congress or in ruLes prescribed by the Supreme Court
    pursuant to statutory authority, the privilege of a witness, person, government, State
    or political subdivision thereof shall be governed by the principles of the common
    law as interpreted by the courts of the United States in the light of reason and experience.” In other words, on matters governed by federal substantive law—and this
    will generally be true in criminal cases, civiL actions brought by the United States, and private federal question cases—federal courts are to apply and develop a federal common law of privilege. Since this question deals with a federal question, the issues should be resolved under federal privilege law. Choice (A) is, therefore, correct. Choices (B) and (C) are wrong because they state the wrong rule about the choice of privilege law. Choice (D) is incorrect in its suggestion that a court should select evidentiary law based on its greater likelihood to ensure the admission of evidence at trial.
173
Q
  1. Two defendants were prosecuted for conspiracy to rob a bank. An undercover officer of the local police department was the most important government witness. The defendants’ principal defense was entrapment.
    The undercover officer testified for the government that he was present at a meeting with the defendants during which they plotted to rob the bank. He further testified that the idea of robbing the bank had first been suggested by one of the defendants, and that afterward, the undercover officer stated that he thought it was a good idea.
    Thereafter, the defendants’ counsel called a witness who testified that the undercover officer had a bad reputation for truthfulness. The defense then called a second witness who proposed to testify that the undercover officer once perpetrated a hoax on the police department and was reprimanded.
    The second witness’s proposed testimony is
    (A) admissible, because the hoax resulted in a reprimand of the undercover officer.
    (B) admissible, because a hoax is probative of the undercover officer’s untruthfulness.
    (C) inadmissible, because it is essentially cumulative impeachment.
    (D) inadmissible, because it is extrinsic evidence of a specific instance of misconduct.
A
  1. (D) FRE 608(b) provides that “specific instances of the conduct of a witness, forthe purpose of attacking or supporting his credibility, otherthan conviction of crime as provided in FRE 609, may not be proved by extrinsic evidence.” Choice (A) is incorrect. The fact pattern tells us nothing about the maximum penalty for a hoax. Furthermore, according to FRE 609(a)(1), evidence of an accused’s convictions for serious offenses must be more probative than prejudicial to be admitted. The exception for crimes involving deception and false statement is narrowly interpreted, and it is not at all clear whether perpetrating a hoax would qualify. Choice (B) is also incorrect because it is not necessarily true that a hoax is probative of untruthfulness. Finally, choice (C) is the wrong answer because it is not likely that two witnesses would be considered cumulative. Under FRE 403, a judge can exclude the needless presentation of cumulative evidence.
174
Q
  1. One morning, a woman telephoned her next-door neighbor and asked if she could borrow her car. The woman explained that her car was being serviced and would not be ready for a couple of days. The woman told her neighbor that she had a doctor’s appointment that afternoon and would return the car immediately afterwards. The neighbor agreed and gave the woman permission to use her car. As the woman was driving to her doctor’s office, she collided with another car.
    As a result of the accident, the other driver brought suit against the woman and the neighbor to recover for her personal injuries. The other driver asserted a claim against the neighbor for negligent entrustment of an automobile and charged the woman with negligent operation of a motor vehicle.
    In her case-in-chief, the other driver called a witness to testify to three incidents of careless driving on the woman’s part during the past six months.
    The trial judge should rule the testimony
    (A) admissible as circumstantial evidence that the woman was negligent on this occasion.
    (B) admissible, because the witness had personal knowledge of the woman’s poor driving record.
    (C) admissible against the neighbor as evidence of the woman’s lack of fitness.
    (D) inadmissible, because specific acts are not admissible except to rebut evidence of good character.
A
  1. (C) According to FRE 405(b), “In cases in which character or a trait of character of a person is an essential element of a charge, claim, or defense, proof may also be made of specific instances of his conduct.” Therefore, choice (D) is incorrect. In connection with FRE 405 are situations where the owner of a motor vehicle or any dangerous object is charged with liability for the acts of a person using it on grounds of negligent entrustment of the vehicle or object to an incompetent or unfit person. In such cases, the character of the custodian is in issue and his acts come in to show
    it. Thus, the witness’s testimony is admissible to show the neighbor’s negligence (namely, that she was aware or should have been aware that the woman was a careless driver) in entrusting her vehicle to the woman. Therefore, choice (C) is the correct answer. Choice (A) is incorrect. Character evidence is propensity evidence, not circumstantial evidence. Choice (B) is incorrect. Personal knowledge alone is insufficient to admit character evidence at trial.
175
Q
  1. A college student owns an expensive sports car. His friend called him up one afternoon and asked to borrow his car for a date he had that night. The college student generously lent the car to his friend. On his way home from the date, basking in the memories of a beautiful evening, the friend lost control of the car and hit a pedestrian.
    As a result of the accident, and after learning that the friend had no money and no insurance, the pedestrian brought suit against the college student to recover for his injuries. He asserted a claim against the college student for negligent entrustment of a vehicle. During his case, the pedestrian introduced evidence that the friend had been in several previous accidents. In his defense, the college student offered the testimony of his girlfriend that the college student frequently lent his car to his friend. The girlfriend further testified that the college student gave his friend permission to use his car only after determining that the friend was a careful and attentive driver.
    The girlfriend’s testimony is
    (A) admissible as evidence of habit.
    (B) admissible as a present sense impression.
    (C) inadmissible, because it goes to the college student’s character.
    (D) inadmissible, because she is biased.
A
  1. (C) This is a highly tested Multistate area dealing with negligent entrustment. Where the owner of a motor vehicle or any dangerous object is charged with Liability for the acts of a person using it on grounds of negligent entrustment, the character of the custodian is in issue, and his acts come in to show it. Such character evidence is probative on the issue of whether the entrustor was negligent in lending the instrumentaLity to the custodian or entrustee. Note, however, that the entrustor’s character is not in issue. Therefore, the college student’s character is not in issue, since he is the person making the entrustment. That’s why character evidence is not admissible in this question. Choice (A) is wrong because habit evidence applies to a person’s regular response to a repeated specific situation. The words “always,”“invariably,”“automatically,” or “without fail” will tip off a habit issue. Conversely, “frequently” or “generaLly” will not. Choice (B) is wrong because the testimony describes past acts directly observed by the witness and is not a hearsay statement. Finally, choice (D) is incorrect. Biased evidence is not per se inadmissible. In an adversarial system, it is up to the advocate to convince the fact- finder that a biased witness is less worthy of belief.